ARDMS Abdomen MOCK exam Questions and Answers (2022/2023) (Verified Answers)

Progression of which of the following abnormalities flattens the portal veins?
Biliary obstruction

The wall thickness in a normal fasting gallbladder should not exceed
3 mm

This color Doppler sonogram is most likely demonstrating which of the following abnormalities?
Pseudoaneurysm

This color Doppler image demonstrates turbulent swirling blood flow within a fluid collection, classic sonographic findings of a common femoral artery pseudoaneurysm.

A patient presents with sudden onset of upper abdominal pain. Ultrasound demonstrates prominence in the stomach rugae. These findings are most suspicious for which of the following conditions?
Gastritis

Prominence of the stomach rugae in a patient with upper abdominal pain is most suspicious for gastritis. Hypervascular, thick gastric walls are sonographic findings associated with gastric ulcers.

Which of the following is a complication of acute pancreatitis?
Duodenal obstruction

Complications of acute pancreatitis may include abscess formation, duodenal obstruction, hemorrhage, phlegmon, and pseudocyst formation. Cholecystitis is a possible etiology of acute pancreatitis.

A patient presents with a history of hematuria. The findings in this duplex image are most suspicious for which of the following pathologies?
Carcinoma

A vascular echogenic mass is identified protruding from the posterior wall of the urinary bladder. Bladder carcinoma commonly presents with a history of painless hematuria. Based on the clinical history, the sonographic findings are suspicious for a malignant mass.

A sagittal sonogram medial to the porta hepatis is demonstrating which of the following abnormalities?
Dilated common bile duct

A hypoechoic mass identified by the calipers is obstructing the common bile duct resulting in dilatation. The mass is most likely a malignancy in the head of the pancreas.

The pathology in this sonogram is most likely a/an
Pseudocyst

A complex fluid collection is identified posterior to the tail of the pancreas. This is most likely a pancreatic pseudocyst. Phlegmons and islet cell tumors appear as hypoechoic masses on ultrasound. A pancreatic hemorrhage is a differential consideration but not the most likely pathology.

Which of the following abnormalities is demonstrated in this transverse sonogram?
Stones in the duct of Wirsung

Multiple stones are located in main pancreatic duct (duct of Wirsung).

Which of the following is a clinical symptom of hypothyroidism?
Muscle cramps

Muscles cramping is a symptom of hypothyroidism. Other symptoms may include weight gain, mental and physical lethargy, arthritis, skin dryness, feeling cold, slow metabolic rate, and decreased heart rate. Symptoms commonly associated with hyperthyroidism include weight loss, palpitations, nervousness, exophthalmos, constant hunger, tremors, increased heart rate, and intolerance to heat.

A 20-year-old patient presents with a palpable left scrotal mass. The sonographic findings are most suspicious for which of the following pathologies?
Malignant neoplasm

A malignant neoplasm is the most likely diagnosis in a young adult demonstrating a hypoechoic intratesticular mass. The patient is afebrile excluding a testicular abscess from the differential considerations.

A patient presents with a history of a palpable neck mass. Which of the following terms best describes the sonographic findings?
Heterogeneous thyroid gland

The sonographer’s technical report should describe the right thyroid lobe as demonstrating an irregular and heterogeneous echo texture.

A patient presents with a history of cirrhosis. The arrows are identifying the
coronary ligament

A hyperechoic linear structure is identified dividing the right subphrenic space from the subhepatic space. This is consistent with the right coronary ligament. The right coronary ligament serves as a barrier between these two peritoneal spaces.

This color Doppler sonogram is most likely demonstrating which of the following abnormalities?
Patent umbilical vein

The sonogram is demonstrating a patent paraumbilical vein. Note the liver parenchyma of the left lobe superior to the patent umbilical vein. Gastric varices are generally located to the left of midline.

Splenomegaly is a consistent finding in which of the following pathologies?
Portal hypertension

Splenomegaly is a consistent finding in cases of portal hypertension.

A duplex image of the splenic vein is demonstrating
retrograde flow.

That’s correct! The splenic vein normally flows toward the transducer, displaying a waveform above the baseline (antegrade). In this case, the blood is flowing away from the transducer back toward the spleen (retrograde).

Which of the following structures separates the subphrenic space into two compartments?
Falciform ligament

The falciform ligament divides the subphrenic space into right and left compartments. The left coronary ligament suspends the left lobe of the liver from the diaphragm.

An asymptomatic patient presents with a history of elevated liver function tests. Based on this clinical history, the sonogram most likely demonstrates
fatty infiltration

Fatty infiltration is the most likely diagnosis in an asymptomatic patient demonstrating diffusely echogenic liver parenchyma. In addition, the portal veins are still clearly visible, consistent with fatty infiltration.

The subhepatic space communicates with the lesser sac through the foramen of
Winslow

The foramen of Winslow allows communication between the subhepatic space and lesser sac.

Which vascular structure courses posterior to the superior mesenteric artery and anterior to the abdominal aorta?
Left renal vein

The left renal vein courses posterior to the superior mesenteric artery and anterior to the abdominal aorta.

The celiac axis is located at
superior to the body of the pancreas

The celiac axis is located superior to the body of the pancreas, superior mesenteric artery, and splenic vein; posterior to the left gastric vein; inferior to the gastroesophageal junction.

A postsurgical patient presents with abdominal tenderness and leukocytosis. A sonogram demonstrates a superficial, ill-defined mass beneath the surgical incision. Based on the clinical history, the sonographic findings are most suspicious for
abscess

An ill-defined superficial mass beneath a recent surgical incision in a patient with leukocytosis most likely represents a postsurgical abscess.

Secretion of thyroid-stimulating hormone is controlled by the
pituitary gland

The anterior pituitary gland controls secretion of thyroid-stimulating hormone.

The coronary vein enters the venous system near the
superior border of the portosplenic confluence

The coronary vein enters the superior border of the portosplenic confluence and may be a collateral source in cases of portal hypertension. The inferior mesenteric vein enters the inferior border of the portosplenic confluence.

A complication generally occurring within days of a renal transplant is
renal vein thrombosis

Renal vein thrombosis generally occurs within days of a renal transplant. Renal artery stenosis typically occurs months to years after transplantation.

The sonogram is most suspicious for which of the following abnormalities?
Varicocele

Multiple anechoic tubular structures are identified posterior to the left testicle, most suspicious for a varicocele. Duplex imaging of these tubular structures should be included in this examination.

The solitary echogenic focus most likely represents a(n)
floating gallstone

Multiple small stones are present in the dependent portion of the gallbladder. The solitary focus most likely represents a floating gallstone.

Clinical findings associated with an ileus may include all of the following EXCEPT
hyperactive bowel sounds

Clinical findings in cases of ileus or bowel obstruction may include absent or hypoechoic bowel sounds, fever, constipation, abdominal pain, nausea, and vomiting.

A cake kidney is generally located in which of the following regions?
Pelvis

A cake kidney is a variant of a horseshoe kidney. The entire medial aspects of the kidneys are fused. A cake kidney is typically located in the pelvis.

Focal dilatation of the anterior wall of the abdominal aorta describes a
saccular aneurysm

Focal outpouching of one arterial wall is characteristic of a saccular aneurysm. They are often a result of trauma or infection.

Which of the following structures is most likely mistaken for the splenic artery?
Pancreatic duct

The pancreatic duct and the splenic artery are most likely to be confused with each other. Color Doppler imaging is a quick method of verifying a vascular structure.

This transverse sonogram of the right testicle is most likely demonstrating which of the following abnormalities?
Intratesticular calcifications

Multiple small hyperechoic foci (microcalcifications) are demonstrated in the right testis. Intratesticular calcifications may be idiopathic or associated with chronic orchitis.

The pathology in this sonogram is most consistent with
metastatic lesions

The liver demonstrates a heterogenous parenchyma most suspicious for metastatic lesions.

A palpable buttock mass identified is most consistent with which of the following?
Lipoma

A well circumscribed mass that does not demonstrate posterior acoustic enhancement is most consistent with a lipoma.

Extended use of oral contraceptives is a predisposing factor in developing which of the following hepatic neoplasms?
Adenoma

Extended use of oral contraceptives is linked to development of liver adenomas. Cavernous hemangiomas and cystadenomas have a female prevalence but are not related to the use of oral contraceptives.

Which of the following neoplasms is commonly located in the tail of the pancreas?
Islet cell tumor

Islet cell tumors are most commonly located in the body and tail of the pancreas. Adenocarcinoma involving the pancreas commonly develops in the head of the pancreas.

Which of the following vascular structures is the arrow identifying?
Internal jugular vein

The anechoic structure is located lateral to the thyroid lobe and anterior to the common carotid artery. This is most consistent with the internal jugular vein. The external jugular vein is located in the lateral neck.

Bile pigments are produced by the
liver

The liver breaks down red blood cells, producing bile pigments.

Which of the following anatomic variants involves the fundus of the gallbladder?
Phrygian cap

Folding of the gallbladder fundus describes a phrygian cap. Hartmann pouch and junctional fold are anatomic variants in the region of the gallbladder neck. A multiseptated gallbladder is a congenital anomaly and fetal lobulation involves the kidney.

Renal dialysis patients have an increased risk of developing a renal
carcinoma

Renal dialysis patients have an increased risk of developing a renal carcinoma, cyst, or adenoma.

This sonogram of the right kidney is most likely demonstrating
junctional parenchymal defect.

A triangular, echogenic cortical defect present in the anterior wall of the right kidney most likely represents a junctional parenchymal defect. An angiomyolipoma is a differential consideration but is not as likely the diagnosis.

An endocrine function of the pancreas includes secretion of
glucagon

Endocrine functions of the pancreas include secretion of glucagon, insulin, and somatostatin. Exocrine functions include secretion of amylase, lipase, and trypsin.

Which of the following demonstrates diffuse enlargement of the abdominal aorta without distal tapering?
Arteriomegaly

Arteriomegaly (aortic ectasia) demonstrates as a diffuse enlargement of the abdominal aorta without distal tapering on ultrasound.

A patient presents with a history of severe upper abdominal pain increasing in intensity over the last 12 hours. Based on this clinical history, the sonogram is most likely demonstrating
acute pancreatitis.

An enlarged hypoechoic pancreas in a patient with severe upper abdominal pain is most consistent with acute pancreatitis. Laboratory values will likely demonstrate elevation in serum amylase and lipase levels.

A round anechoic mass adjacent to the renal pelvis is most likely a(n)
parapelvic cyst

A round anechoic mass (cyst) adjacent to the renal pelvis is most likely a parapelvic cyst. An extrarenal pelvis and renal vein demonstrate an oval contour medial to the renal pelvis.

A small hyperechoic pancreas is most suspicious for which of the following abnormalities?
Chronic pancreatitis

Chronic pancreatitis is associated with atrophy of the pancreas and demonstrates a hyperechoic parenchyma. Fatty infiltration generally does not affect the size of the pancreas.

An 11-year-old presents with a history a palpable thyroid gland, fatigue, and dysphagia. Based on this clinical history, the color Doppler image of the right thyroid lobe is most likely demonstrating
thyroiditis

The duplex sonogram is demonstrating a hypervascular, hypoechoic enlarged thyroid gland, most suspicious for thyroiditis. Clinical symptoms of thyroiditis may include fatigue, fever, leukocytosis, neck pain, dysphagia, and hyperthyroidism followed by hypothyroidism.

A patient presents with a history of a painful lump on her posterior wrist. Based on this clinical history, the sonogram is most likely demonstrating which of the following abnormalities?
Ganglion cyst

An anechoic smooth walled mass is identified in the area of concern. This most likely represents a ganglion cyst. A Baker cyst is located in the posterior medial portion of the knee joint.

The anechoic structure is most likely arising from which of the following structures?
Kidney

The cyst is most likely arising from the renal parenchyma. Possible differential considerations include hepatic or adrenal cyst.

The sonographic findings in this infant kidney are most consistent with which of the following anomalies?
Renal duplication

Two individual renal sinuses are present. The lower sinus demonstrates pelviectasis (Grade 1 hydronephrosis). This is most suspicious for renal duplication. Hypertrophied column of Bertin is a common anatomic variants.

The arrow is identifying which of the following vascular structures?
Common hepatic artery

The arrow identifies a structure branching from the celiac axis and coursing toward the liver. This is most consistent with the common hepatic artery. The common hepatic artery becomes the proper hepatic artery after the origin of the gastroduodenal artery.

Cases of gallbladder carcinoma frequently demonstrate coexisting
cholelithiasis

In approximately 90 percent of cases, cholelithiasis coexists with gallbladder carcinoma

Sonographic findings associated with pyelonephritis include
well-defined renal pyramids.

Generalized or focal swelling of the kidney demonstrating well-defined renal pyramids is the typical sonographic finding associated with pyelonephritis.

Which benign liver neoplasm is composed of large, blood-filled cystic spaces?
Cavernous hemangioma

The cavernous hemangioma is composed of large, blood-filled cystic spaces and is the most common benign hepatic neoplasm.

Which of the following arteries are termed the hypogastric arteries?
Internal iliac arteries

The internal iliac arteries are also known as the hypogastric arteries.

A transverse image of the urinary bladder is demonstrating which of the following?
Ureteroceles

Bilateral hyperechoic septations within the posterior bladder wall are most consistent with ureteroceles. Ureteroceles are focal dilatations of the distal ureter expanding as urine enters the bladder.

Normal sonographic findings of a renal transplant include
prominent renal pyramids.

Normal sonographic findings in a renal transplant include prominent renal pyramids, hyperechoic renal sinus, and hypoechoic renal cortex.

Which of the following structures aids in locating the gallbladder fossa?
Main lobar fissure

The main lobar fissure is a sonographic landmark used in locating the gallbladder fossa. The right hepatic vein divides the right lobe of the liver into anterior and posterior segments.

A 25-year-old female patient presents with a history of mild right upper quadrant pain and normal liver function tests. Based on the sonogram, the sonographer should ask the patient
Do you use oral contraceptives?

A 25-year-old female demonstrating a hypoechoic liver mass and a peripheral hypoechoic halo is suspicious for a hepatic adenoma. The sonographer should inquire whether the patient is taking oral contraceptives.

The “water lily” sign is associated with which of the following abnormalities?
Echinococcal cyst

The “water lily” sign describes a collapsed cyst within a cyst, characteristic of an echinococcal cyst.

This sagittal image of the left kidney is most likely demonstrating
a normal neonatal kidney.

A 5.0 cm kidney (using sidebar measurements) inferior to the spleen demonstrates an echogenic renal cortex, anechoic medullary pyramids, and a discrete renal sinus. This is most consistent with a normal neonatal kidney.

Which of the following most accurately describes the location of the adrenal glands?
Medial to the kidneys

The adrenal glands are located anterior, medial, and superior to each kidney and lateral to the crura of the diaphragm.

Cryptorchism is a predisposing risk factor for developing which of the following abnormalities?
Malignancy

Undescended testis (cryptorchism) is a risk factor for developing testicular torsion, malignancy, and infertility.

The hyperechoic structure in this sonogram represents
a hepatic stent

The hyperechoic structure in the right portal vein is most consistent with a hepatic stent. Patients with cirrhosis may require a hepatic stent or shunt to decrease pressure within the portosplenic venous system. Recanalization demonstrates as an echogenic thickening to the intraluminal hepatic walls.

Which of the following vascular structures normally demonstrates this waveform?
A. Portal vein
B. Hepatic vein
C. Splenic vein
D. Superior mesenteric vein
Hepatic vein

Pulsatile bidirectional venous flow is characteristic of a normal hepatic vein.

A 6-week-old female infant presents with a history of left hip clicking. Based on this clinical history, the sonogram is most likely demonstrating a
dysplastic hip.

The Graf alpha angle is abnormal. To be considered within normal limits the alpha angle should be 60 degrees or greater. The alpha angle in this case measures 46 degrees. The beta angle is normal measuring under 55 degrees ( 48 degrees). This is consistent with developmental dysplasia of the hip. In addition the femoral head is too lateral but still partially lies within the acetabulum consistent with subluxation.

The sonographic findings are most suspicious for a(n)
angiomyolipoma

A well-defined hyperechoic mass is present in the anterior cortex. Angiomyolipomas are common incidental findings.

Which of the following laboratory values increases in Addison disease?
Serum potassium

Laboratory values associated with Addison disease demonstrate an elevation in serum potassium and a decrease in serum sodium and glucose.

Which of the following structures divides the left lobe into two segments?
Left hepatic vein and ligamentum of Teres

The left hepatic vein and ligamentum of Teres divide the left lobe of the liver into left and right segments. The left lobe is separated from the right lobe by the middle hepatic vein and the main lobar fissure.

A contracted gallbladder is a common sonographic finding in which of the following conditions?
Chronic cholecystitis

A small or contracted gallbladder is a common sonographic finding in cases of chronic cholecystitis. Cholesterolosis, adenomyomatosis, and metastatic gallbladder disease demonstrate as intraluminal masses

Which of the following laboratory values is associated with malignancy in the elderly population?
Alpha-fetoprotein

Alpha-fetoprotein is an abnormal finding associated with an underlying malignancy in nonpregnant patients.

A patient with a history of splenomegaly demonstrates multiple tubular structures posterior to the left lobe of the liver. These tubular structures are most suspicious for
gastric varices.

Splenomegaly is associated with portal hypertension. Multiple anechoic tubular structures in the epigastric region are most suspicious for gastric varices.

The sonographic findings are most consistent with which of the following abnormalities?
Cholelithiasis

Multiple small echogenic foci layer in the dependent portion of the gallbladder. Attenuation of sound by these foci produces posterior acoustic shadowing.

Which of the following abnormalities is demonstrated in this color Doppler image?
Fatty infiltration with focal fat sparing

The liver parenchyma demonstrates a diffuse increase in echogenicity (fatty infiltration). Hypoechoic areas are identified anterior and posterior to a hepatic vessel (fat sparing). Incidentally the right renal cortex has a scalloped appearance typical of fetal lobulation.

What is the first step in a sterile procedure technique?
Wash your hands thoroughly

The first step in a sterile procedure technique is to wash your hands thoroughly, followed by placing a sterile wrapped package on a clean and stable surface.

Hemochromatosis is a rare disorder that may lead to
cirrhosis

Hemochromatosis is characterized by excessive iron deposits throughout the body and may cause cirrhosis.

Which of the following abnormalities is present in this sagittal sonogram?
Adenomyomatosis

Immobile hyperechoic foci demonstrating a “comet-tail” reverberation artifact are characteristic findings of adenomyomatosis.

The gastroduodenal artery arises from which of the following arteries?
Hepatic artery

The gastroduodenal artery arises from the common hepatic artery. The common hepatic artery is a branch of the celiac axis.

Which of the following liver pathologies is associated with glycogen storage disease?
Adenoma

Glycogen storage disease is an autosomal recessive disorder resulting in excessive deposition of glycogen in the liver, kidneys, and gastrointestinal tract. It is associated with development of a hepatic adenoma, focal nodular hyperplasia, and nephromegaly.

Regulation of serum electrolytes is a function of the
kidneys

The urinary system regulates serum electrolytes.

Which of the following abnormalities are demonstrated in this sonogram?
Hydrocele and complex epididymal cyst

An anechoic fluid collection is identified in the right scrotal sac. The scrotal wall appears thin. This is most consistent with a hydrocele. A complex cyst is also identified in the head of the epididymis.

The arrow is identifying which of the following structures?
Falciform ligament

A hyperechoic linear structure extends anterior to the left lobe of the liver consistent with the falciform ligament. The falciform attaches the liver to the anterior abdominal wall and separates the subphrenic space into right and left compartments.

Elevation in creatinine levels is associated with
renal failure, chronic nephritis, or urinary obstruction

Elevation in creatinine levels is associated with renal failure, chronic nephritis, or urinary obstruction.

Prominence of the portal veins is a sonographic characteristic of which of the following abnormalities?
Hepatitis

The contour of the liver is generally unaffected in cases of acute hepatitis. Acute hepatitis may demonstrate a hypoechoic liver parenchyma, hepatomegaly, associated splenomegaly, and prominence of the portal veins (star effect).

A positive Phalen’s sign is associated with an abnormality of which of the following joints?
Wrist

A positive Phalen’s and/or Tinel’s sign is associated with carpal tunnel syndrome (wrist).

The arrow is identifying which of the following structures?
Right renal artery

The right renal artery coursing posterior to the inferior vena cava is a common sonographic landmark.

The main lobar fissure is a sonographic landmark used for locating the
gallbladder fossa

The main lobar fissure is a sonographic landmark used to locate the gallbladder fossa.

Dehydration increases the risk for developing
renal vein thrombosis

Dehydration increases the risk for developing renal vein thrombosis

Spectral analysis of the suprarenal portion of the abdominal aorta should demonstrate a
high resistance waveform with a low diastolic component

The abdominal aorta demonstrates a high resistance waveform with a low diastolic component proximal to the renal arteries. A triphasic waveform is found distal to the renal arteries.

Which of the following ligaments is present in this image of the liver?
Venosum

The sonogram demonstrates the left and caudate lobes of the liver separated by the hyperechoic ligament of venosum.

Glisson’s capsule surrounds which of the following organs?
Liver

Glisson’s capsule surrounds the liver and Gerota’s fascia surrounds the kidneys.

A patient presents with abdominal pain and fever following an appendectomy. Based on this clinical history, the sonographic findings are most suspicious for a(n)
abscess

An irregular bordered, complex hepatic mass is present in a febrile post appendectomy patient most likely representing a hepatic abscess.

Which of the following gallbladder abnormalities is not always pathological in origin?
Biliary sludge

Biliary sludge may develop in cases of prolonged fasting or biliary disease.

A Klatskin tumor is located near which of the following structures?
Hepatic hilum

A Klatskin tumor is a malignant neoplasm located at the junction of the right and left hepatic ducts near the hepatic hilum.

A normal lymph node will likely demonstrate
a hyperechoic fatty center

The normal lymph node demonstrates a smooth, hypoechoic, oval shape and a hyperechoic fatty center on ultrasound.

This sonogram is most likely demonstrating which of the following?
Gallbladder adenoma

An immobile (decubitus position), nonshadowing, echogenic intraluminal focus is most consistent with a gallbladder adenoma (polyp).

Which of the following pathologies is more commonly associated with severe abdominal pain?
Portal vein thrombosis

Severe abdominal pain is more commonly associated with portal vein thrombosis.

The color Doppler image is most suspicious for which of the following abnormalities?
A. Epididymitis
B. Testicular torsion
C. Testicular abscess
D. Testicular carcinoma
Testicular torsion

The color Doppler image demonstrates an absence of testicular flow in the right testicle and the presence of blood flow within the left testicle. This is most suspicious for testicular torsion.

The sonographic appearance of candidiasis is described as
uniform hypoechoic masses

Candidiasis is a fungal infection associated with immune-suppressed patients. Sonographic findings include uniform hypoechoic masses, hyperechoic masses demonstrating posterior shadowing, or target lesions.

Which laboratory value reveals cell injury or damage?
Aspartate aminotransferase

Aspartate aminotransferase (AST) is an enzyme present in many kinds of tissues releasing when cells are injured or damaged. Levels are proportional to the amount of damage and the time between cell injury and testing

An ultrasound is requested to rule out Budd-Chiari syndrome. The sonographer should thoroughly evaluate which of the following abdominal veins?
Hepatic veins

Budd-Chiari syndrome is a life-threatening condition associated with thrombosis of the hepatic veins. The sonographer should thoroughly evaluate the liver vasculature.

In this case which of the following laboratory values would be of benefit?
Hematocrit

A fluid-fluid level is demonstrated within the liver parenchyma most suspicious for a hematoma. Hematocrit levels would aid in determining internal bleeding.

A patient presents with a history of abdominal pain. A sonogram of the liver is most likely demonstrating
polycystic liver disease

Multiple contiguous hepatic cysts are most suspicious for polycystic liver disease.

Which anatomic variant is present in this sonogram of the gallbladder?
A. Duplication
B. Phrygian cap
C. Hartmann pouch
D. Intrahepatic location
Phrygian cap

A fold in the gallbladder fundus is consistent with a phrygian cap. Hartmann pouch or a junctional fold is anatomic variants located near the gallbladder neck.

Which of the following accurately describes the portal veins?
Portal veins are intralobar in location

Portal veins are intralobar or intrasegmental in location. The portal veins supply approximately 70 to 75 percent of the liver’s total blood volume. The main portal vein divides into the right and left portal veins. The walls of the portal veins contain fibrin.

Which of the following is an abnormal flow characteristic of the hepatic veins?
Hepatopetal

Hepatic veins course away from the liver toward the inferior vena cava. This is termed hepatofugal flow. Hepatic veins demonstrate spontaneous multiphasic (pulsatile) flow.

The sonographic finding in this sonogram of the common bile duct is most consistent with
A. ascariasis.
B. cholangitis.
C. clonorchiasis.
D. cholangiocarcinoma
cholangitis

The common duct demonstrates hypoechoic wall thickening. This is most consistent with cholangitis. Cholangitis is a common complication in patients with autoimmune deficiency syndrome.

Production of prothrombin depends on the amount of vitamin: D, A, K, B ?
Vitamin K

Production of prothrombin depends on the amount of vitamin K in the circulatory system.

A hepatic abscess is most likely to develop within the
right lobe

Hepatic abscesses form within the right lobe in approximately 80 percent of cases.

In which of the following laboratory tests is marked elevation associated with obstructive jaundice?
Alkaline phosphatase

Marked elevation in alkaline phosphatase is associated with obstructive jaundice. Elevation of direct or conjugated bilirubin is associated with biliary obstruction.

Which of the following pathologies is most likely demonstrated in this sonogram?
Hepatitis

An increase in echogenicity in the walls of the portal veins is a sonographic finding associated with hepatitis (star effect). Biliary obstruction demonstrates parallel channeling and flattening of the portal veins.

When assessing for mesenteric ischemia, which of the following arteries should be evaluated?
Celiac, superior, and inferior mesenteric arteries

Duplex imaging of the celiac, superior mesenteric, and inferior mesenteric arteries should be evaluated when assessing for mesenteric ischemia.

Which of the following ligaments forms the anterior border of the epiploic foramen?
Hepatoduodenal ligament

The hepatoduodenal ligament surrounds the portal triad just proximal to the porta hepatis and forms the anterior border of the epiploic foramen.

Which of the following pathologies is most likely identified in this sonogram of the liver?
A. Abscess
B. Adenoma
C. Hepatoma
D. Cavernous hemangioma
Hepatoma

A solitary, smooth walled, hypoechoic mass is identified in the left lobe of the liver most suspicious for a hepatoma. The mass does not demonstrate a hypoechoic halo characteristic of a hepatic adenoma or irregular wall margins characteristic of an abscess.

A beaded appearance to the intrahepatic ducts is a sonographic finding in which of the following conditions?
Caroli disease

Caroli disease demonstrates a segmental, saccular, or beaded appearance to the intrahepatic biliary ducts.

Sonographic findings in cases of mesenteric lymphomatous are described by the
A. olive sign.
B. doughnut sign.
C. sandwich sign.
D. keyboard sign
sandwich sign

Sandwich sign describes an anechoic lesion with a hyperechoic center found in mesenteric lymphomatous. This type of lesion is more frequent in non-Hodgkin’s lymphoma cases. The jejunum and ileum demonstrate small folds in the wall termed the keyboard sign. The doughnut sign is a sonographic finding associated with intussusception.

Which of the following pathologies is most consistent with these sonographic findings?
Portal vein thrombosis

Altered blood flow within the portal vein, in conjunction with a hypoechoic intraluminal mass, is most suspicious for portal vein thrombosis. Severe abdominal pain and loss of appetite are common clinical symptoms connected with thrombosis of the portal venous system.

A transplanted kidney is commonly placed in the
right iliac fossa

A renal transplant is commonly placed in the anterior right iliac fossa.

An obese patient presents with a history of elevated liver function tests. The arrows are identifying which of the following structures?
Perinephric fat

Increase in perinephric and peritoneal fat are common sonographic findings in obese patients. The arrows are identifying a thick amount of perinephric fat.

To obtain an optimal evaluation of the gallbladder, the adult patient should fast for
A. 2 to 4 hours.
B. 4 to 6 hours.
C. 6 to 8 hours.
D. 8 to 10 hours
6 to 8 hours

To obtain an optimal evaluation of the gallbladder, the adult patient should fast for 6 to 8 hours prior to the examination.

Which of the following conditions is associated with secondary subacute thyroiditis?
A. Grave’s disease
B. Mirizzi syndrome
C. Hashimoto disease
D. de Quervain syndrome
de Quervain syndrome

de Quervain syndrome is associated with subacute thyroiditis secondary to a viral infection. Hashimoto disease is a progressive autoimmune inflammatory disorder of the thyroid gland.

Portal veins appear prominent on ultrasound because the walls contain
A. lipids.
B. fibrin.
C. calcium.
D. plaque.
Fibrin

Portal walls contain fibrin, resulting in a prominent hyperechoic appearance on ultrasound.

The protective connective tissue surrounding each kidney is termed
A. renal fascia.
B. renal capsule.
C. Gerota’s fascia.
D. perinephric fat
renal capsule

The renal capsule is a protective connective tissue surrounding each kidney. Gerota’s or renal fascia is the fibrous covering of tissue surrounding each kidney.

The caudate lobe is located
A. medial to the lesser sac.
B. anterior to the porta hepatis.
C. anterior to the ligamentum venosum.
D. medial to the inferior vena cava
medial to the inferior vena cava

The caudate lobe lies anterior and medial to the inferior vena cava, posterior to the ligamentum venosum and porta hepatis, and lateral to the lesser sac.

The most common sonographic appearance of the mediastinum testis is described as a(n)
A. complex linear structure.
B. anechoic tortuous structure.
C. hyperechoic linear structure.
D. hyperechoic tubular structure.
hyperechoic linear structure

A hyperechoic linear structure in the posterior medial aspect of the testis is the most common sonographic appearance of the mediastinum testis.

A transverse sonogram of the liver is likely demonstrating which of the following?
A. Hepatomegaly resulting from biliary dilatation
B. Hepatic vein thrombosis and biliary dilatation
C. Prominent portal veins resulting from hepatitis
D. Biliary dilatation secondary to metastatic disease
Biliary dilatation secondary to metastatic disease

The most likely diagnosis of this sonogram is biliary dilatation resulting from metastatic disease. Multiple discrete masses are present in the liver parenchyma.

Prothrombin is an enzyme produced by the
A. heart.
B. liver.
C. spleen.
D. pancreas
Liver

The liver produces prothrombin. Elevation is associated with cirrhosis, malignancy, malabsorption of vitamin K, and clotting failure.

The renal cortex is bound by the renal capsule and
A. major calyces.
B. perinephric fat.
C. arcuate vessels.
D. column of Bertin
arcuate vessels

The renal cortex is bound by the renal capsule and the arcuate vessels.

How many centimeters below the superior mesenteric artery do the main renal arteries arise?
A. 0.5 to 1.0 cm
B. 1.0 to 1.5 cm
C. 1.5 to 2.0 cm
D. 2.0 to 2.5 cm
1.0 to 1.5 cm

The main renal arteries are located 1.0 to 1.5 cm inferior to the superior mesenteric artery. Knowing this is useful when determining whether an abdominal aortic aneurysm includes the renal arteries.

The inferior mesenteric artery is best visualized in which of the following body planes?
A. Transverse
B. Sagittal
C. Coronal
D. Sagittal oblique
Sagittal oblique

That’s correct! The inferior mesenteric artery is best visualized in the sagittal oblique plane, to the left of midline, approximately 1 cm superior to the abdominal aortic bifurcation.

A patient presents with a history of vague right upper quadrant pain. Laboratory values are within normal limits. The sonogram is most likely demonstrating a(n)
A. hepatoma.
B. metastatic lesion.
C. angiomyolipoma.
D. cavernous hemangioma
cavernous hemangioma.

A well-defined hyperechoic mass is present in a patient with normal laboratory values. This is most consistent with a cavernous hemangioma.

Which of the following enzymes will most likely affect blood pressure?
A. Renin
B. Trypsin
C. Somatostatin
D. Aldosterone
Renin

Renin, also known as angiotensinogenase, is an enzyme secreted by the kidneys to help regulate blood pressure.

Enlarged lymph nodes demonstrating smooth wall margins are most consistent with an underlying
A. infection.
B. obstruction.
C. malignancy.
D. hemorrhage
infection

Lymphadenopathy demonstrating a normal oval shape with smooth wall margins is most consistent with an underlying infection. Irregular wall margins or decreases in the fatty hilum are findings suspicious for an underlying malignancy.

Extension of the right lobe of the liver inferior and anterior to the right kidney is termed a(n)
A. accessory lobe.
B. Riedel’s lobe.
C. succenturiate lobe.
D. pyramidal lobe.
Riedel’s lobe.

Extension of the right lobe of the liver anterior and inferior to the kidney is termed a Riedel’s lobe. This is a common finding in the female population.

A patient presents with a history of right upper quadrant pain since returning from an overseas vacation. Based on this clinical history, the sonogram is mostly likely displaying a(n)
A. hepatic abscess.
B. hemorrhagic cyst.
C. echinococcal cyst.
D. hepatocellular carcinoma.
echinococcal cyst

A cystic structure within a cystic structure (honeycomb appearance) in a patient recently returning from an overseas vacation is characteristic of an echinococcal cyst. A hepatic abscess demonstrates as a heterogeneous complex mass.

Pelvic lymph nodes course along the
A. psoas muscles.
B. iliac vessels.
C. inguinal canal.
D. broad ligament
iliac vessels

The pelvic lymph nodes course along the iliac vessels.

Which of the following most accurately describes the location of the common hepatic duct?
Anterior to the main portal vein and lateral to the proper hepatic artery

The common hepatic duct lies anterior to the main portal vein and lateral to the proper hepatic artery.

All of the following structures attach to the liver by a ligament EXCEPT
A. stomach.
B. diaphragm.
C. gallbladder.
D. anterior abdominal wall.
gallbladder

Ligaments attach the diaphragm, stomach, retroperitoneum, and anterior abdominal wall to the liver. The gallbladder lies within a hepatic fossa.

A transjugular intrahepatic portosystemic shunt (TIPS) generally connects which of the following?
A. Right portal vein and the inferior vena cava
B. Right hepatic vein and right portal vein
C. Middle hepatic vein and the left portal vein
D. Left hepatic vein and left portal vein
Right hepatic vein and right portal vein

TIPS generally connects the right hepatic vein and right portal vein.

Inferiorly, the right hepatic lobe is separated from the left lobe by the
A. right portal vein.
B. left hepatic vein.
C. main lobar fissure.
D. middle hepatic vein.
main lobar fissure

The right hepatic lobe is separated from the left lobe by the main lobar fissure inferiorly and the middle hepatic vein superiorly.

Retroperitoneal fibrosis is most commonly located near large blood vessels in the region of the
A. pancreas.
B. stomach.
C. lumbar spine.
D. urinary bladder.
lumbar spine

Retroperitoneal fibrosis is a chronic inflammatory process wherein fibrotic tissue surrounds large blood vessels in the lumbar area. Fibrotic tissue rarely extends above the second lumbar vertebra.

Which of the following laboratory values reflects the balance between production and excretion of bile?
A. Bilirubin
B. Alkaline phosphatase
C. Alanine aminotransferase (ALT)
D. Aspartate aminotransferase (AST)
Bilirubin

Bilirubin reflects the balance between production and excretion of bile. Serum albumin evaluates protein synthesis.

Sharp, severe flank pain, radiating to the groin describes
A. dysuria.
B. renal colic.
C. renal failure.
D. dyspareunia
renal colic

Sharp, severe flank pain radiating to the groin describes renal colic. Renal colic is associated with nephrolithiasis. Pain or burning during urination describes dysuria. Abnormal pain during sexual intercourse is termed dyspareunia.

Which of the following is the optimal technique for evaluating the kidneys?
A. Harmonic imaging
B. Patient lying supine
C. Empty urinary bladder
D. Phased array linear transducer
Empty urinary bladder

Utilizing an empty urinary bladder is optimal when assessing the kidneys. Optimal patient position depends on several different factors. Prone imaging is great for pediatric cases. Adult exams typically utilize a coronal scanning plane.

Stenson’s duct is located in which of the following glands?
A. Thyroid gland
B. Parotid salivary gland
C. Sublingual salivary gland
D. Submandibular salivary gland
Parotid salivary gland

The parotid salivary gland contains Stenson’s duct. Submandibular salivary gland contains Wharton duct.

An inward extension of the renal cortex describes a
A. medullary pyramid.
B. fetal lobulation.
C. dromedary hump.
D. column of Bertin
column of Bertin

An inward extension of the renal cortex between the medullary pyramids describes a column of Bertin. Dromedary hump describes an outward cortical bulge.

A patient with a history of cirrhosis presents with a history of a painful supraumbilical mass. Based on this clinical history, the sonogram is most suspicious for which of the following abnormalities?
A. Abdominal wall hematoma
B. Meckel diverticulum
C. Abdominal wall hernia
D. Recanalized paraumbilical vein
Abdominal wall hernia

A small defect is demonstrated in the anterior abdominal wall with extension of the omentum through the defect.

The expected sonographic appearance of a subacute hematoma is described as a(n)
A. anechoic mass.
B. complex mass.
C. hypoechoic mass.
D. hyperechoic mass
complex mass.

A subacute hematoma is composed of both hypoechoic blood and a hyperechoic clot appearing as a complex mass on ultrasound. A hypoechoic mass is more commonly associated with acute cases and an anechoic appearance in chronic cases.

Acute tubular necrosis is the most common cause of
A. renal failure.
B. nephrolithiasis.
C. sinus lipomatosis.
D. renal cell carcinoma
renal failure

Ischemic necrosis of the tubular cells is the most common cause of renal failure.

A true abdominal aortic aneurysm is defined as dilatation of the aorta
A. inferior to the renal arteries.
B. when compared with a previous study.
C. with a diameter measuring 3 cm or greater.
D. when compared with a proximal segment
with a diameter measuring 3 cm or greater

A true abdominal aortic aneurysm (AAA) measures 3 cm or greater in diameter. The majority of AAA are located inferior to the renal arteries.

A toddler presents to the ultrasound department with a history of a left upper quadrant mass. Based on this clinical history, the coronal sonogram is most suspicious for which of the following abnormalities?
A. Splenomegaly
B. Adrenal adenoma
C. Nephroblastoma
D. Intussusception
Nephroblastoma

A complex solid mass is identified inferior to the spleen. A normal left kidney is not identified. In a toddler the most likely diagnosis would be nephroblastoma. Differential consideration may include neuroblastoma if a normal left kidney is identified. An adrenal adenoma appears as a hypoechoic homogeneous mass.

Sialolithiasis most commonly occurs in which of the following salivary glands?
A. Parotid
B. Sublingual
C. Submandibular
D. Sublingual and submandibular
Submandibular

60 – 90% of cases of sialolithiasis occur in the submandibular salivary glands with 10 to 20 percent occurring in the parotid salivary glands.

The gallbladder lies posterior and inferior to the
A. inferior vena cava.
B. coronary ligament.
C. right hepatic vein.
D. main lobar fissure
main lobar fissure.

The gallbladder lies posterior and inferior to the main lobar fissure, lateral to the inferior vena cava, and anterior medial to the right kidney.

Inferiorly, the right hepatic lobe is separated from the left lobe by the
A. right portal vein.
B. left hepatic vein.
C. main lobar fissure.
D. middle hepatic vein
Main Lobar Fissure

The right hepatic lobe is separated from the left lobe by the main lobar fissure inferiorly and the middle hepatic vein superiorly.

The celiac axis is located at
A. anterior to the left gastric vein.
B. inferior to the splenic vein.
C. superior to the gastroesophageal junction.
D. superior to the body of the pancreas
superior to the body of the pancreas

The celiac axis is located superior to the body of the pancreas, superior mesenteric artery, and splenic vein; posterior to the left gastric vein; inferior to the gastroesophageal junction.

A 25-year-old female patient presents with a history of mild right upper quadrant pain and normal liver function tests. Based on the sonogram, the sonographer should ask the patient:
A. Are you diabetic?
B. Do you use oral contraceptives?
C. Are you taking medication for hypertension?
D. Have you undergone any abdominal surgery
Do you use oral contraceptives?

A 25-year-old female demonstrating a hypoechoic liver mass and a peripheral hypoechoic halo is suspicious for a hepatic adenoma. The sonographer should inquire whether the patient is taking oral contraceptives.

The caudate lobe is located
A. medial to the lesser sac.
B. anterior to the porta hepatis.
C. anterior to the ligamentum venosum.
D. medial to the inferior vena cava
medial to the inferior vena cava

The caudate lobe lies anterior and medial to the inferior vena cava, posterior to the ligamentum venosum and porta hepatis, and lateral to the lesser sac.

Which of the following structures are included in an ultrasound protocol of the pancreas?
A. Liver and spleen
B. Liver and biliary system
C. Inferior vena cava and abdominal aorta
D. Liver and right kidney
Liver and biliary system

Due to the association of the liver and biliary tree with the pancreas, sagital and transverse images of these structures should be included in an ultrasound protocol of the pancreas.

Which of the following is the most common form of renal fusion?
A. Renal duplication
B. Cake kidney
C. Cross-fused ectopia
D. Horseshoe kidney
Horseshoe kidney

The horseshoe kidney is the most common form of renal fusion

Which of the following anatomic variants involves the fundus of the gallbladder?
A. Junctional fold
B. Phrygian cap
C. Hartmann pouch
D. Multiseptated gallbladder
Phrygian cap

Folding of the gallbladder fundus describes a phrygian cap. Hartmann pouch and junctional fold are anatomic variants in the region of the gallbladder neck. A multiseptated gallbladder is a congenital anomaly and fetal lobulation involves the kidney.

Which of the following conditions is most likely to result in acute pancreatitis?
A. Cirrhosis
B. Diverticulitis
C. Cholelithiasis
D. Portal hypertension
Cholelithiasis

Biliary disease is the most common cause of acute pancreatitis followed by alcohol abuse.

To obtain an optimal evaluation of the gallbladder, the adult patient should fast for
A. 2 to 4 hours.
B. 4 to 6 hours.
C. 6 to 8 hours.
D. 8 to 10 hours
To obtain an optimal evaluation of the gallbladder, the adult patient should fast for 6 to 8 hours prior to the examination.

Visceral lymph nodes are located in which of the following regions?
A. Around the aorta
B. In the peritoneum
C. Near the adrenal glands
D. Along the prevertebral vessels
In the peritoneum

Visceral lymph nodes are located in the peritoneum. They follow the course of the vessels supplying the major organs. Parietal lymph nodes are located in the retroperitoneum and course along the prevertebral vessels.

A transplanted kidney is commonly placed in the
A. right flank.
B. left iliac fossa.
C. right iliac fossa.
D. paraumbilical region
A renal transplant is commonly placed in the anterior right iliac fossa.

Which complication is most likely associated with an annular pancreas?
A. Hepatitis
B. Pancreatitis
C. Bowel obstruction
D. Urinary tract obstruction
Bowel obstruction

Annular pancreas is a rare congenital anomaly where the head of the pancreas surrounds the duodenum. This anomaly may result in obstruction of the duodenum or biliary tree.

Which of the following should be included in a kidney ultrasound protocol when hydronephrosis is encountered?
A. Color Doppler imaging of the renal artery and vein
B. Pre and post void bladder volumes
C. Color Doppler imaging of the ureteral jet(s)
D. Gray-scale images of the liver and spleen
Color Doppler imaging of the ureteral jet(s)

When hydronephrosis is encountered, color Doppler imaging of the ureteral jets should be included in the examination protocol.

Which of the following congenital anomalies demonstrates a “dipping effect” to inferior poles of the kidneys?
A. Cake kidney
B. Renal ptosis
C. Renal duplication
D. Horseshoe kidney
Horseshoe kidney

Demonstration of bilateral “dipping” of the inferior poles should raise suspicion of a horseshoe anomaly. Crossed fused ectopia demonstrates one single large kidney within the same quadrant.

An asymptomatic patient presents with a history of elevated liver function tests. Based on this clinical history, the sonogram most likely demonstrates
A. hepatitis.
B. cirrhosis.
C. fatty infiltration.
D. focal nodular hyperplasia
fatty infiltration

Fatty infiltration is the most likely diagnosis in an asymptomatic patient demonstrating diffusely echogenic liver parenchyma. In addition, the portal veins are still clearly visible, consistent with fatty infiltration.

The sonographic findings are most consistent with which of the following abnormalities?
A. Cholangitis
B. Cholecystitis
C. Cholelithiasis
D. Adenomyomatosis
Cholelithiasis

Multiple small echogenic foci layer in the dependent portion of the gallbladder. Attenuation of sound by these foci produces posterior acoustic shadowing.

Which of the following is the optimal technique for evaluating the kidneys?
A. Harmonic imaging
B. Patient lying supine
C. Empty urinary bladder
D. Phased array linear transducer
Empty urinary bladder

Utilizing an empty urinary bladder is optimal when assessing the kidneys. Optimal patient position depends on several different factors. Prone imaging is great for pediatric cases. Adult exams typically utilize a coronal scanning plane.

Patients with diabetes mellitus have an increased risk for developing
A. ascariasis.
B. cholangitis.
C. gangrenous cholecystitis.
D. adenomyomatosis
gangrenous cholecystitis

Patients with diabetes mellitus have an increased risk for developing cholelithiasis, gangrenous cholecystitis, gallbladder enlargement, and perforation.

Presence of a palpable ”thrill” over a vascular structure is highly suggestive of a(n)
A. occlusion.
B. stenosis.
C. arteriovenous fistula.
D. abdominal aortic aneurysm
arteriovenous fistula

The presence of a ”thrill” over a vascular structure is highly suspicious and characteristic of an arteriovenous fistula.

Renal dialysis patients have an increased risk of developing a renal
A. abscess.
B. calculus.
C. infection.
D. carcinoma
carcinoma

Renal dialysis patients have an increased risk of developing a renal carcinoma, cyst, or adenoma.

Paralytic ileus is associated with which of the following conditions?
A. Cirrhosis
B. Acute pancreatitis
C. Acute cholecystitis
D. Chronic pancreatitis
Acute pancreatitis

In cases of acute pancreatitis, amylase and lipase levels are generally elevated. Patients complain of an abrupt onset of epigastric pain. Other symptoms include paralytic ileus and nausea/vomiting.

Which of the following is a complication of acute pancreatitis?
A. Cholecystitis
B. Hepatomegaly
C. Splenomegaly
D. Duodenal obstruction
Duodenal obstruction

Complications of acute pancreatitis may include abscess formation, duodenal obstruction, hemorrhage, phlegmon, and pseudocyst formation. Cholecystitis is a possible etiology of acute pancreatitis.

Production of prothrombin depends on the amount of vitamin
Vitamin D.
Vitamin A.
Vitamin K.
Vitamin B.
Production of prothrombin depends on the amount of vitamin K in the circulatory system.

How many centimeters below the superior mesenteric artery do the main renal arteries arise?
A. 0.5 to 1.0 cm
B. 1.0 to 1.5 cm
C. 1.5 to 2.0 cm
D. 2.0 to 2.5 cm
The main renal arteries are located 1.0 to 1.5 cm inferior to the superior mesenteric artery. Knowing this is useful when determining whether an abdominal aortic aneurysm includes the renal arteries.

A round anechoic mass adjacent to the renal pelvis is most likely a(n)
A. hydroureter.
B. renal vein.
C. extrarenal pelvis.
D. parapelvic cyst
parapelvic cyst.

A round anechoic mass (cyst) adjacent to the renal pelvis is most likely a parapelvic cyst. An extrarenal pelvis and renal vein demonstrate an oval contour medial to the renal pelvis.

A metabolic disorder resulting from excessive production of cortisol describes
A. Conn syndrome.
B. Addison disease.
C. Marfan syndrome.
D. Cushing syndrome
Cushing syndrome

The cortex of the adrenal gland secretes cortisol. Cushing disease is associated with an excessive production of cortisol. Partial or complete failure of the adrenocortical function is associated with Addison disease. Conn syndrome is associated with elevated aldosterone levels.

Progression of which of the following abnormalities flattens the portal veins?
A. Ascariasis
B. Portal vein thrombosis
C. Portal hypertension
D. Biliary obstruction
Biliary obstruction

Progression of biliary obstruction will inadvertently flatten the portal venous system.

Extension of the right lobe of the liver inferior and anterior to the right kidney is termed a(n)
A. accessory lobe.
B. Riedel’s lobe.
C. succenturiate lobe.
D. pyramidal lobe
Riedel’s lobe

Extension of the right lobe of the liver anterior and inferior to the kidney is termed a Riedel’s lobe. This is a common finding in the female population.

A patient presents with a history of a Whipple procedure. The sonographer should expect to visualize
A. splenomegaly.
B. a small gallbladder.
C. pancreatic tissue attached to the liver.
D. common bile duct attached to the duodenum
common bile duct attached to the duodenum

A Whipple procedure is a surgical resection of the pancreatic head or periampullary area to relieve obstruction of the biliary tree often due to a malignant tumor. The remaining normal pancreatic tissue is anastomosed to the duodenum. The common bile duct is attached to the duodenum distal to the pancreas and stomach.

The coronary vein enters the venous system near the

A. medial border of the main portal vein.
B. inferior border of the right hepatic vein.
C. superior border of the portosplenic confluence.
D. inferior border of the superior mesenteric vein
superior border of the portosplenic confluence

The coronary vein enters the superior border of the portosplenic confluence and may be a collateral source in cases of portal hypertension. The inferior mesenteric vein enters the inferior border of the portosplenic confluence.

An inward extension of the renal cortex describes a

A. medullary pyramid.
B. fetal lobulation.
C. dromedary hump.
D. column of Bertin
column of Bertin.

An inward extension of the renal cortex between the medullary pyramids describes a column of Bertin. Dromedary hump describes an outward cortical bulge.

An abrupt increase in alkaline phosphatase levels is suggestive of

A. acute cholecystitis.
B. acute pancreatitis.
C. obstructive jaundice.
D. portal hypertension
obstructive jaundice.

That’s correct! Alkaline phosphatase is an enzyme produced primarily by the liver. A marked increase is suggestive of obstructive jaundice.

A complication generally occurring within days of a renal transplant is

A. pyelonephritis.
B. hydronephrosis.
C. renal vein thrombosis.
D. renal artery stenosis.
renal vein thrombosis

Renal vein thrombosis generally occurs within days of a renal transplant. Renal artery stenosis typically occurs months to years after transplantation.

In a renal transplant patient, a lymphocele is generally located

A. medial.
B. lateral.
C. anterior.
D. posterior
Medial

Lymphoceles are usually found medial to a renal transplant and will frequently contain debris.

Proteinuria is a common clinical finding in which of the following conditions?

A. Hydronephrosis
B. Glomerulonephritis
C. Nephrocalcinosis
D. Medullary sponge disease
Glomerulonephritis

Proteinuria is a finding in cases of glomerulonephritis, nephrolithiasis, renal carcinoma, polycystic disease, hypertension, and diabetes mellitus.

Which vascular structure courses posterior to the superior mesenteric artery and anterior to the abdominal aorta?

A. Splenic vein
B. Left renal vein
C. Left gastric artery
D. Superior mesenteric vein
Left renal vein

The left renal vein courses posterior to the superior mesenteric artery and anterior to the abdominal aorta.

The most common sonographic appearance of the mediastinum testis is described as a(n)

A. complex linear structure.
B. anechoic tortuous structure.
C. hyperechoic linear structure.
D. hyperechoic tubular structure.
hyperechoic linear structure

A hyperechoic linear structure in the posterior medial aspect of the testis is the most common sonographic appearance of the mediastinum testis.

Which of the following structures separates the subphrenic space into two compartments?

A. Falciform ligament
B. Left coronary ligament
C. Gastroesophageal junction
D. Hepatoduodenal ligament
Falciform ligament

The falciform ligament divides the subphrenic space into right and left compartments. The left coronary ligament suspends the left lobe of the liver from the diaphragm.

A positive Phalen’s sign is associated with an abnormality of which of the following joints?

A. Knee
B. Ankle
C. Wrist
D. Shoulder
A positive Phalen’s and/or Tinel’s sign is associated with carpal tunnel syndrome (wrist).

Which of the following laboratory values is associated with malignancy in the elderly population?

A. Bilirubin
B. Serum albumin
C. Alpha-fetoprotein
D. Alkaline phosphatase
Alpha-fetoprotein is an abnormal finding associated with an underlying malignancy in nonpregnant patients.

A cake kidney is generally located in which of the following regions?

A. Pelvis
B. Left flank
C. Paraumbilical
D. Right flank
Pelvic

A cake kidney is a variant of a horseshoe kidney. The entire medial aspects of the kidneys are fused. A cake kidney is typically located in the pelvis.

In which of the following laboratory tests is marked elevation associated with obstructive jaundice?

A. Indirect bilirubin
B. Alpha-fetoprotein
C. Alkaline phosphatase
D. Alanine aminotransferase
Alkaline phosphatase

Marked elevation in alkaline phosphatase is associated with obstructive jaundice. Elevation of direct or conjugated bilirubin is associated with biliary obstruction.

A decrease in serum amylase levels can be associated with

A. cirrhosis.
B. pancreatic carcinoma.
C. biliary obstruction.
D. peptic ulcer disease
cirrhosis

Decreases in serum amylase may occur in cases of cirrhosis or hepatitis. Elevation in serum amylase is associated with acute pancreatitis and peptic ulcer disease.

Which of the following laboratory values increases in Addison disease?

A. Cortisol
B. Serum sodium
C. Aldosterone
D. Serum potassium
Serum potassium

Laboratory values associated with Addison disease demonstrate an elevation in serum potassium and a decrease in serum sodium and glucose.

Which of the following accurately describes the spleen?

A. The spleen is a retroperitoneal organ.
B. The peritoneum completely covers the spleen.
C. The spleen is located medial to the pancreas.
D. The spleen lies posterior and lateral to the stomach
The spleen lies posterior and lateral to the stomach within the peritoneum. Peritoneum covers the spleen except at the hilum.

Sharp, severe flank pain, radiating to the groin describes

A. dysuria.
B. renal colic.
C. renal failure
D. dyspareunia
renal colic

Sharp, severe flank pain radiating to the groin describes renal colic. Renal colic is associated with nephrolithiasis. Pain or burning during urination describes dysuria. Abnormal pain during sexual intercourse is termed dyspareunia.

A “time out” is best described as

A. verifying the proper equipment and medication are available for a specific invasive procedure.

B. time set aside for the patient to ask questions about the invasive procedure being performed.

C. time after the procedure is performed to evaluate the patient and document the area of needle path for any evidence of post procedural bleeding.

D. verifying with the patient the correct invasive procedure is being performed, on the correct patient and at the correct site.
Verifying with the patient the correct invasive procedure is being performed, on the correct patient and at the correct site.

“time out” is performed before beginning the invasive procedure to verify with the patient the site of procedure, the correct invasive procedure is being performed, and the patient’s name and birthdate

Which of the following laboratory values reflects the balance between production and excretion of bile?

A. Bilirubin
B. Alkaline phosphatase
C. Alanine aminotransferase (ALT)
D. Aspartate aminotransferase (AST)
Bilirubin

Bilirubin reflects the balance between production and excretion of bile. Serum albumin evaluates protein synthesis.

Which of the following vascular landmarks is used to differentiate an indirect from a direct inguinal hernia?

A. Internal iliac artery
B. Superior epigastric artery
C. Inferior epigastric artery
D. External iliac artery
Inferior epigastric artery

The inferior epigastric artery is the vascular landmark used to differentiate an indirect from a direct inguinal hernia.

The gastroduodenal artery arises from which of the following arteries?

A. Gastric artery
B. Splenic artery
C. Hepatic artery
D. Superior mesenteric artery
Hepatic artery

The gastroduodenal artery arises from the common hepatic artery. The common hepatic artery is a branch of the celiac axis.

Glisson’s capsule surrounds which of the following organs?

A. Liver
B. Spleen
C. Kidney
D. Pancreas
Glisson’s capsule surrounds the liver and Gerota’s fascia surrounds the kidneys.

Which of the following demonstrates diffuse enlargement of the abdominal aorta without distal tapering?

A. Pseudoaneurysm
B. Mycotic aneurysm
C. Ectatic aneurysm
D. Arteriomegaly
Arteriomegaly (aortic ectasia) demonstrates as a diffuse enlargement of the abdominal aorta without distal tapering on ultrasound.

Which of the following liver pathologies is associated with glycogen storage disease?

A. Hepatitis
B. Adenoma
C. Hepatoma
D. Cavernous hemangioma
Adenoma

Glycogen storage disease is an autosomal recessive disorder resulting in excessive deposition of glycogen in the liver, kidneys, and gastrointestinal tract. It is associated with development of a hepatic adenoma, focal nodular hyperplasia, and nephromegaly.

An endocrine function of the pancreas includes secretion of

A. lipase.
B. pepsin.
C. glucagon.
D. amylase.
glucagon

Endocrine functions of the pancreas include secretion of glucagon, insulin, and somatostatin. Exocrine functions include secretion of amylase, lipase, and trypsin.

Development of which of the following is associated with recurrent urinary tract infections?

A. Angiomyolipoma
B. Renal cell carcinoma
C. Staghorn calculus
D. Multicystic dysplasia
Staghorn calculus

Development of staghorn calculus is associated with chronic urinary tract infections and chronic history of kidney stones.

Which of the following vascular structures is located directly posterior to the neck of the pancreas?

A. Inferior vena cava
B. Gastroduodenal artery
C. Superior mesenteric vein
D. Superior mesenteric artery
Superior mesenteric vein

The superior mesenteric vein and portosplenic confluence lie directly posterior to the neck of the pancreas.

Which of the following neoplasms is commonly located in the tail of the pancreas?

A. Cyst
B. Adenoma
C. Islet cell tumor
D. Adenocarcinoma
Islet cell tumors are most commonly located in the body and tail of the pancreas. Adenocarcinoma involving the pancreas commonly develops in the head of the pancreas.

When empty, bladder wall thickness should not exceed

A. 2 mm.
B. 3 mm.
C. 5 mm.
D. 7 mm.
Bladder wall thickness should not exceed 5 mm when empty or 3 mm when distended.

Bile pigments are produced by the

A. liver.
B. pancreas.
C. gallbladder.
D. biliary ducts
The liver breaks down red blood cells, producing bile pigments.

Which laboratory value reveals cell injury or damage?

A. Bilirubin
B. Alkaline phosphatase
C. Aspartate aminotransferase
D. Alanine aminotransferase
Aspartate aminotransferase

Aspartate aminotransferase (AST) is an enzyme present in many kinds of tissues releasing when cells are injured or damaged. Levels are proportional to the amount of damage and the time between cell injury and testing.

A beaded appearance to the intrahepatic ducts is a sonographic finding in which of the following conditions?

A. Caroli disease
B. Bouveret syndrome
C. Budd-Chiari syndrome
D. Courvoisier syndrome
Caroli disease demonstrates a segmental, saccular, or beaded appearance to the intrahepatic biliary ducts.

A transjugular intrahepatic portosystemic shunt (TIPS) generally connects which of the following?

A. Right portal vein and the inferior vena cava
B. Right hepatic vein and right portal vein
C. Middle hepatic vein and the left portal vein
D. Left hepatic vein and left portal vein
Right hepatic vein and right portal vein

TIPS generally connects the right hepatic vein and right portal vein.

Which of the following conditions is associated with secondary subacute thyroiditis?

A. Grave’s disease
B. Mirizzi syndrome
C. Hashimoto disease
D. de Quervain syndrome
de Quervain syndrome is associated with subacute thyroiditis secondary to a viral infection. Hashimoto disease is a progressive autoimmune inflammatory disorder of the thyroid gland.

Hyperparathyroidism may lead to which of the following renal pathologies?

A. Multicystic dysplasia
B. Pyelonephritis
C. Nephrolithiasis
D. Acute tubular necrosis
Nephrolithiasis

Excessive function of the parathyroid glands may lead to osteoporosis and nephrolithiasis.

Fifty percent of neuroblastomas appear prior to what age?

A. 6 months to 1 year
B. 1 to 2 years
C. 2 to 3 years
D. 5 years or less
1 – 2 years

Fifty percent of neuroblastomas occur prior to 1 to 2 years of age. A nephroblastoma (Wilms’ tumor) occurs at 5 years of age or less.

Acute tubular necrosis demonstrates on ultrasound as

A. a unilateral small hypoechoic kidney.
B. bilateral renal enlargement with hypoechoic renal pyramids.
C. unilateral renal enlargement with absent renal pyramids.
D. bilateral renal enlargement with hyperechoic renal pyramids
Bilateral renal enlargement with hyperechoic renal pyramids is the classic sonographic finding associated with acute tubular necrosis.

The protective connective tissue surrounding each kidney is termed

A. renal fascia.
B. renal capsule.
C. Gerota’s fascia.
D. perinephric fat
The renal capsule is a protective connective tissue surrounding each kidney. Gerota’s or renal fascia is the fibrous covering of tissue surrounding each kidney.

Sonographic findings associated with pyelonephritis include

A. hydronephrosis.
B. hypoechoic renal sinus.
C. ill-defined renal capsule.
D. well-defined renal pyramids.
well-defined renal pyramids

Generalized or focal swelling of the kidney demonstrating well-defined renal pyramids is the typical sonographic finding associated with pyelonephritis.

Extended use of oral contraceptives is a predisposing factor in developing which of the following hepatic neoplasms?

A. Adenoma
B. Hepatoma
C. Cystadenoma
D. Cavernous hemangioma
Adenoma

Extended use of oral contraceptives is linked to development of liver adenomas. Cavernous hemangiomas and cystadenomas have a female prevalence but are not related to the use of oral contraceptives.

Sonographic findings in cases of mesenteric lymphomatous are described by the

A. olive sign.
B. doughnut sign.
C. sandwich sign.
D. keyboard sign.
sandwich sign

Sandwich sign describes an anechoic lesion with a hyperechoic center found in mesenteric lymphomatous. This type of lesion is more frequent in non-Hodgkin’s lymphoma cases. The jejunum and ileum demonstrate small folds in the wall termed the keyboard sign. The doughnut sign is a sonographic finding associated with intussusception.

A postsurgical patient presents with abdominal tenderness and leukocytosis. A sonogram demonstrates a superficial, ill-defined mass beneath the surgical incision. Based on the clinical history, the sonographic findings are most suspicious for

A. seroma.
B. abscess.
C. hematoma.
D. lymphocele
abscess

An ill-defined superficial mass beneath a recent surgical incision in a patient with leukocytosis most likely represents a postsurgical abscess.

Splenomegaly is a consistent finding in which of the following pathologies?

A. Hepatic steatosis
B. Biliary obstruction
C. Portal hypertension
D. Portal vein thrombosis
Portal hypertension

Splenomegaly is a consistent finding in cases of portal hypertension

Enlarged lymph nodes demonstrating smooth wall margins are most consistent with an underlying

A. infection.
B. obstruction.
C. malignancy.
D. hemorrhage
infection

Lymphadenopathy demonstrating a normal oval shape with smooth wall margins is most consistent with an underlying infection. Irregular wall margins or decreases in the fatty hilum are findings suspicious for an underlying malignancy.

Which of the following ligaments forms the anterior border of the epiploic foramen?

A. Falciform ligament
B. Coronary ligament
C. Gastrohepatic ligament
D. Hepatoduodenal ligament
Hepatoduodenal ligament

The hepatoduodenal ligament surrounds the portal triad just proximal to the porta hepatis and forms the anterior border of the epiploic foramen.

A true abdominal aortic aneurysm is defined as dilatation of the aorta

A. inferior to the renal arteries.
B. when compared with a previous study.
C. with a diameter measuring 3 cm or greater.
D. when compared with a proximal segment.
with a diameter measuring 3 cm or greater.

A true abdominal aortic aneurysm (AAA) measures 3 cm or greater in diameter. The majority of AAA are located inferior to the renal arteries.

Which of the following is NOT a sonographic finding associated with Achilles’ tendonitis?

A. Irregular margins
B. Hypervascular tendon
C. Thickness exceeding 7 mm
D. Prominent hyperechoic areas interspersed within the tendon
Prominent hyperechoic areas interspersed within the tendon

Tendonitis of the Achilles’ tendon demonstrates hypoechoic areas interspersed between the fibrous tissues of the tendon. Additional sonographic findings may include hypervascular tendon, irregular margins, and a thickness exceeding 7 mm.

The wall thickness in a normal fasting gallbladder should not exceed

A. 2 mm.
B. 3 mm.
C. 5 mm.
D. 7 mm
The normal fasting gallbladder demonstrates smooth hyperechoic walls measuring 3 mm or less.

If cholelithiasis is discovered on an ultrasound, the sonographer must determine the

A. size of the stone(s).
B. contour of the stone(s).
C. composition of the stone(s).
D. mobility of the stone(s)
mobility of the stone(s).

The mobility of the gallstones must be evaluated. Lodged or immobile stones can change the patient’s course of treatment.

Which of the following vessels receives blood from the gallbladder, pancreas, spleen, and gastrointestinal tract?

A. Hepatic veins
B. Main portal vein
C. Inferior vena cava
D. Superior mesenteric vein
The main portal vein receives blood from the gastrointestinal tract, gallbladder, pancreas, and spleen.

Cases of gallbladder carcinoma frequently demonstrate coexisting

A. cholecystitis.
B. cholelithiasis.
C. pleural effusion.
D. adenomyomatosis.
cholelithiasis

In approximately 90 percent of cases, cholelithiasis coexists with gallbladder carcinoma.

Regulation of serum electrolytes is a function of the

A. liver.
B. kidneys.
C. adrenal glands.
D. thyroid glands
kidneys

The urinary system regulates serum electrolytes.

Which of the following arteries are termed the hypogastric arteries?

A. Suprarenal arteries
B. Common iliac arteries
C. External iliac arteries
D. Internal iliac arteries
The internal iliac arteries are also known as the hypogastric arteries.

All of the following structures attach to the liver by a ligament EXCEPT

A. stomach.
B. diaphragm.
C. gallbladder.
D. anterior abdominal wall.
gallbladder

Ligaments attach the diaphragm, stomach, retroperitoneum, and anterior abdominal wall to the liver. The gallbladder lies within a hepatic fossa.

Which of the following most accurately describes the location of the adrenal glands?

A. Medial to the kidneys
B. Inferior to the kidneys
C. Anterior to the pancreas
D. Medial to the diaphragmatic crura
Medial to the kidneys

The adrenal glands are located anterior, medial, and superior to each kidney and lateral to the crura of the diaphragm.

Hemochromatosis is a rare disorder that may lead to

A. hepatitis.
B. cirrhosis.
C. cholangitis.
D. schistosomiasis.
cirrhosis

Hemochromatosis is characterized by excessive iron deposits throughout the body and may cause cirrhosis.

A contracted gallbladder is a common sonographic finding in which of the following conditions?

A. Cholesterolosis
B. Metastatic gallbladder disease
C. Portal hypertension
D. Chronic cholecystitis
Chronic cholecystitis

A small or contracted gallbladder is a common sonographic finding in cases of chronic cholecystitis. Cholesterolosis, adenomyomatosis, and metastatic gallbladder disease demonstrate as intraluminal masses

Prominence of the portal veins is a sonographic characteristic of which of the following abnormalities?

A. Hepatitis
B. Candidiasis
C. Hepatic steatosis
D. Biliary obstruction
Hepatitis

The contour of the liver is generally unaffected in cases of acute hepatitis. Acute hepatitis may demonstrate a hypoechoic liver parenchyma, hepatomegaly, associated splenomegaly, and prominence of the portal veins (star effect).

Secretion of thyroid-stimulating hormone is controlled by the

A. cerebellum.
B. thyroid gland.
C. hypothalamus.
D. pituitary gland
pituitary gland

The anterior pituitary gland controls secretion of thyroid-stimulating hormone.

Which of the following structures is located posterior to the superior mesenteric vein and anterior to the aorta?

A. Splenic vein
B. Uncinate process
C. Body of the pancreas
D. Caudate lobe of the liver
Uncinate process

The uncinate process lies directly posterior to the superior mesenteric vein and anterior to the aorta and inferior vena cava. The splenic vein courses posterior to the pancreatic body and anterior to the aorta.

The “water lily” sign is associated with which of the following abnormalities?

A. Hydronephrosis
B. Echinococcal cyst
C. Hepatic cystadenoma
D. Polycystic liver disease
Echinococcal cyst

The “water lily” sign describes a collapsed cyst within a cyst, characteristic of an echinococcal cyst.

Stenson’s duct is located in which of the following glands?

A. Thyroid gland
B. Parotid salivary gland
C. Sublingual salivary gland
D. Submandibular salivary gland
Parotid salivary gland

The parotid salivary gland contains Stenson’s duct. Submandibular salivary gland contains Wharton duct.

The gallbladder lies posterior and inferior to the

A. inferior vena cava.
B. coronary ligament.
C. right hepatic vein.
D. main lobar fissure
main lobar fissure

The gallbladder lies posterior and inferior to the main lobar fissure, lateral to the inferior vena cava, and anterior medial to the right kidney.

Clinical findings associated with an ileus may include all of the following EXCEPT

A. fever.
B. constipation.
C. nausea and vomiting.
D. hyperactive bowel sounds
hyperactive bowel sounds

Clinical findings in cases of ileus or bowel obstruction may include absent or hypoechoic bowel sounds, fever, constipation, abdominal pain, nausea, and vomiting

A Klatskin tumor is located near which of the following structures?

A. Adrenal gland
B. Hepatic hilum
C. Head of the pancreas
D. Gastroesophageal junction
Hepatic hilum

A Klatskin tumor is a malignant neoplasm located at the junction of the right and left hepatic ducts near the hepatic hilum.

Which of the following pathologies is more commonly associated with severe abdominal pain?

A. Portal hypertension
B. Cirrhosis
C. Hepatoma
D. Portal vein thrombosis
Portal vein thrombosis

Severe abdominal pain is more commonly associated with portal vein thrombosis.

The subhepatic space communicates with the lesser sac through the foramen of

A. Ovale.
B. Monro.
C. Winslow.
D. Santorini
The foramen of Winslow allows communication between the subhepatic space and lesser sac

Which of the following terms describes the sections of the epididymis?

A. Head, body, and tail
B. Right, left, and posterior
C. Superior, midline, and inferior
D. Medial, posterior, and lateral
The sections of the epididymis include the head (posterior and superior portion), body (posterior portion), and tail (posterior and inferior portion).

bulls eye sign
Ultrasound appearance of gastroesophageal junction?

rugae
irregular folds in stomach formed by mucosa and submucosal

5 mm’s
the undistended wall should not exceed how many mm’s

haustrae
what are the large sacculations of the colon called?

thickened loops with no change with compression
slow graded transducer pressure is applied with evaluation of gi tract, what would these findings reveal?

target sign and pseudo kidney sign
what is the classic appearance of adenocarcinoma?

6mm
normal appendix has a target appearance and should not measure more than—————mm in diameter?

adhesions
what is most common cause of intestinal obstruction?

Increased activity
Evaluation of motor activity with obstruction will reveal?

Chronic
Is hashimoto’s thryroiditis an acute or chronic condition?

calcium levels
the parathyroid glands secrete PTH which is responsible for maintaining what levels?

Papillary
what is most common type of thyroid cancer?

Increased PTH levels, hypophosphatemia and hypercalcemia
The lab values with parathyroid adenoma’s will reveal what?

Longus coli muscle
what is the muscle located posterior to the thyroid and adjacent to the trachea?

External carotid artery
The superior thyroid artery branches off what artery?

Thyroid enlargement, protruding eyeballs
Two features of Grave’s disease?

Multinodular Goiter
What is the most common type of thyroid abnormality?

Innominate artery
The right common carotid artery arises from which artery?

Adventitia
What is the outermost layer of the arterial wall called?

Systemic hypertension
What is most common cause of arterial dissection?

distal to stenosis
maximum turbulence is usually detected distal or proximal to stenosis?

hyperechoic, hypoechoic, and free floating in vessel
Characteristics of acute venous thrombosis?

tendon
Fibrous cord that attaches muscle to bone is what?

Enhancement
An artifact that occurs when sound strikes a curved or cylindrical structure at less than 90 degrees is referred to as what?

Perimysium
A group of muscle fibers are bound together by a connective tissue layer called?

Capillaries and nerves
The endomysium consists of a network of?

Superior to the bladder in the lower abdomen
The usual location for a urachal cyst is where?

Popliteal Space
Where would a bakers cyst be located?

Anechoic mass with good through transmission
What is the most sonographic appearance of an acute hematoma?

Lymphoma
Multiple lobulated masses located around the porta hepatis, celiac trunk or SMA would most likely represent what?

Fusiform
What is most common type of abdominal aortic aneurysm?

Saccular
Type of abdominial aneurysm that has outpouching which is usually the result of trauma or infection is?

Pseudoaneurysm
The patient has a mass located in the groin region adjacent to the artery, which has mixed echogenicity and visable swirling blood, the most likely diagnosis would be what?

Cancer of the GI tract and ovarian cancer
The most common source of metastatic disease to the peritoneum and mesentery is from?

Trauma and catheterization
What is the most common cause of pseudoaneurysm?

Dermoid
What is the most common benign mesenteric neoplasm?

50%
The one year mortality rate for a 6 cm abdominal aortic aneurysm is about?

Exudative ascites
Peritoneal fluid collection due to an inflammatory or malignant process, which exhibits fine or coarse internal echoes is referred to as?

Fine needle aspiration
Type of biopsy that uses a capillary action technique is?

Subcostal
Core biopsy of the liver is best performed from which approach?

Lower pole in prone position
Renal biopsies are obtained from which pole of the kidney?

Upright
Thoracentesis is best performed with the patient in what position?

Matching layer
The material located in front of the transducer that helps to improve sound transmission by reducing ultrasound reflection at the surface is called what?

Decrease pulse duration, decrease SPL, improve axial resolution
Purpose of Damping material?

one quarter of a wavelength
The optimal matching layer thickness is?

bandwidth
Range of frequencies present in a pulse is called?

mechanical
What is the type of transducer that has single or multiple moving elements is called?

Linear switched and linear sequenced array
Another name for a linear array transducer is?

Linear arrays
Transducers that have multiple elements that are pulsed with slight time delays are called?

Annular array
Type of transducer that has concentric elements is?

Sliced thickness
An artifact that appears as echoes filling a fluid filled structure or vessel is?

Clean shadows
Shadowing produced from calcific structures are referred to as?

Side lobe occur with single element transducers and grating lobes occur with multiple element transducers.
What is the key difference between side lobe artifacts and grating lobe artifacts?

9-12 cm
What does the normal adult kidney measure?

True capsule
What is the fibrous capsule surrounding the kidney referred to as?

Columns of Bertin
The cortical tissue extending into the space between adjacent pyramids appearing as a hypoechoic mass area is?

Cortex
What is outer portion of kidney referred to as?

Nephron
Structural and functional unit of kidney?

5 mm or less
The renal cortex is considered abnormal if measures what mm?

Dromedary Hump
Normal renal anatomical variant which has a bulging appearance to the contour of the kidney is referred to as a?

Angiomyolipoma or renal hematoma
A renal mass composed of fat, blood vessels and smooth muscle tissue is called?

Renal Cell Carcinoma
Mr M is a 60 y/o man at the u/s dept with hematuria, pain and palpable mass, the u/s reveals a primarily solid renal mass with areas of cystic degeneration. Scattered calcifications were also present, what would the most likely diagnosis be?

Nephroblastoma
What is a Wilm’s tumor also know as?

Increased
End stage renal disease wil reveal __ echogenicity of the renal cortex

High resistance characteristics
Doppler evaluation of a pt with renal parenchymal disease will reveal what?

Ureter and bladder
What is the most common site of obstruction by kidney stones?

Iliac fossa
What is the normal location of the renal transplant?

hydronephrosis, fluid collections around kidney, abnormal renal size and ATN
Sonographic signs of renal transplant rejection would include?

Lymphocele
Mr B was referred to the U/S dept several mths after and renal transplant and the U/S exam revealed a lg fluid collection with internal echoes arising medial to the lower pole of the kidney, what is the most likely diagnosis?

200cm/sec
A significant renal artery stenosis would be considered in a renal transplant pt when the peak systolic velocity exceeds?

Supernumery kidney
A completed duplication of the kidney and ureter is referred to as?

Horseshoe Kidney
Fusion of the upper or lower poles of the kidney during fetal development is called?

Ligamentum Venosum and left portal vein
The dividing landmarks for the left lobe and caudate lobe are the ??

PHA, MPV, and CBD
What structures are located within the Portal Triad?

Cystic and Common Hepatic ducts
The CBD is formed by the confluence what two ducts?

Right Hepatic vein
The right lobe of the liver is divided into anterior and posterior segments by the ?

AST, ALT, and Alkaline phosphatase
What lab values will increase with Hepatocellular adn obstructive diseases?

Caput Madusal Sign
Distended collateral veins visualized on the abdominal wall have been referred to as the what?

Schistosomiasis
A condition where the worm punctures the skin and migrates into the general circulation via the lymphatics or venous system is??

cavernmatous Transformation
Portal vein occlusion that occurs in infancy or the neonatal period will demonstrate a collection of collateral vessels in the region of the porta hepatis, this is referred to as?

Thick walled cysts, hepatomegaly, calcification of cyst walls and possible thrombosis and infarction of blood vessels
Sonographic characteristics seen with hydatid disease?

Liver cell adenoma
Solitary hypoechoic to hyperechoic benign liver mass frequently seen in women taking oral contraceptives is referred to as?

Hepatic artery thrombosis, and PV thrombosis
Most common post-op liver transplant complications?

Liver metastasis
Multiple target lesions found withing the liver would most likely represent?

Hepatoma
A positive fetal antigen is found in over 50% of patients who have?

Varicocle
Enlargement of veins of the spermatic cord commonly found on the left side in males less than 25 y/o is what?

Tunica albuginea
The dense fibrous capsule surrounding the testicles is referred to as the ?

Hydrocele
Abnormal accumulation of serous fluid between the layers of the tunica vaginales which results in painless scrotal swelling is referred to as?

Epididiymitis
The most frequent cause of an acute scrotum in the post puberty male is?

Enlarged testicle, Normal testicle echogenicity, decrease flow in testicle and epididymis.
U/S findings of testicular torsion 4 to 6 hours after torsion occurs will reveal?

Vas Deferens
What structure carries the sperm from each testis to the prostatic urethra?

Less than 3 mm
What is the average size of testicular microlithiasis?

Diferential
What artery provides the primary blood supply to the epididymis?

Main lobar fissure and right portal vein
The two anatomical landmarks for the GB location are?

3 mm
The normal distended GB should not exceed?

Phrygian cap
A fold in the GB toward the fundus is referred to as?

Spiral valves of heister
A shadowing effect may be seen arising from the cystic duct region, what is the sources of the shadow?

The cystic duct and common hepatic duct
Common bile duct is formed by the confluence of?

3mm
Gallstones will produce a shadow when what size?

Inspissated bile
Sludge is also known as?

Hydrops
Male pt presents with epigastric pain, nausea and vomiting, the u/s exam reveals a markedly dilated GB with thin walls, the most likely diagnosis would be?

Acalculous Cholecystitis
Pt presents with RUQ pain, nausea, and vomiting, positive murphys sign, enlarged thick walled GB containing echoes that did not layer or shadow, Pericholecystic fluid was also demonstrated, most likely diagnosis would be?

Adenomyomatosis
Small echogenic mass like areas with internal cysts are seen in the fundus of the GB that produces a comet tail artifact, what is this?

Cholangitis
Definition of stones in ducts?

Confluence or right and left hepatic ducts
A klatskins tumor occurs in the?

Choledochal Cyst
Anomalous insertion of the CBD into the pancreatic duct allowing reflux of pancreatic juice into the bile duct leading to dilatation and cholangitis is referred to as?

4mm
Normal average size of the common hepatic duct of a pt under the age of 40 would be?

true
stones impacted in the gb neck will not move with changing patient positions, true or false?

Perirenal Compartment
The kidneys and adrenal glands are located in which compartment?

SMA
Enlarged lymph nodes in the para-aortic region may cause displacement of the?

Sandwich sign
When lymphoma involves the mesentery, there may be a layered appearance, this is called?

rounded appearance, hypoechoic mass, anechoic mass
Usual appearance of enlarged lymph nodes?

Phenochromocytoma
Large solid mass with well defined borders arising from the adrenal region, there are also areas of cystic degeneration, no other abnormalities in the abdomen found, most likely diagnosis would be?

Neuroblastoma
A heterogenous childhood adrenal tumor which frequently invades surrounding structures and presents with weight loss, malaise, abdominal protrusion and fever describes?

Retroperitoneal fibrosis
A pt presents with abdominal pain, nausea, vomiting, and hypertension, the u/s exam reveals a bulky midline mass, most likely the diagnosis is?

Peritoneal or intraperitoneal
Location of spleen?

Hilum
Accessory spleens are usually detected near the?

Hemangioma
Most common primary neoplasm of the spleen?

Solid focal lesions, calcifications, bright echogenic lesions.
U/S characteristics of Histoplasmosis and tuberculosis?

Superior mesenteric vein and inferior mesenteric vein
Splenic vein exits hilum of spleen and merges with the and __ to form the portal vein.

Left Hemidiaphragm
What is at the superior border of the spleen?

Subcapsular hematoma
Post MV accident with LLQ pain, crescent shaped fluid collection that conforms to the spleen is identified, what is this?

peripheral zone
70 % of prostatic tissue is located in which zone?

40
In older men prostate considered enlarged if weighs more than how many grams?

BPH
A prostate exam reveals an enlarged hypoechoic inner zone and calcifications in the transition zone, this is suggestive of?

Anterior
The common hepatic artery courses________________
to the portal vein.

Anterolateral
The gastroduodenal artery is located in the_________________ aspect of the head of pancreas

Splenic, left gastric and common hepatic
Which 3 arteries arise from the celiac trunk?

Posterior
The right renal artery usually courses__________ to the IVC

below
The renal arteries arise _____ the SMA

SMA
Artery that supplies blood to the small bowel, portions of the large intestine and pancreatic head is?

Segmental
After entering the hilum, the main renal artery divides into the ?

15-20%
What percent of people have duplicated renal arteries?

2.5
The IVC is considered dilated when its dimensions exceeds how many cm’s?

Low resistance
Normal renal artery will demonstrate what type of resistance?

Splenic Vein
The inferior mesenteric vein drains into the ?

AV fistula
Pt presents with a palpable thrill post op, doppler evaluation reveals an abnormal turbulent waveform with a mixture of arterial and venous flow signals, most likely represents?

Bud Chiari Syndrome
Pt presents with ascites, hepatomegaly, serpigenous collaterals lead to IVC and absent hepatic vein flow, findings are suggestive of?

Intussucseption
A condition where one part of the intestine slips into another part just below it represents what?

6 mm
Normal appendix has a target appearance and should not measure more than_________mm in diameter.

Abnormal LFT’S, increased total billirubin and alkaline phosphatase.
Lab values associated with Metastatic Lesions?

Abnormal LFT”S and increased AFP
Hepatoblastoma, usually found during infancy or childhood, lab values?

Increased AST, ALT, alkaline phosphatase, 70% of the time AFP will be present
Lab values with malignant tumors? HCC?

Normal AFP, excludes mass being malignant
Lab Values with Infantile Hemangioendothelium?

Normal
With liver cell adenomas? How are LFTS/ Normal or abnormal?

Normal LFT’s
FNH lab values? LFT’S?

Normal
Benign solid tumors? LFT’s normal or abnormal?

decreased hematocrit, leukocytosis
Hematoma lab values?

Varied depending on cause
Fungal infection lab values?

Leukocytosis, increased LFT’s and anemia
Infection abscess (Pyogenic) Lab Values?

Jaundice and increased alkaline phosphatase if cysts cause billiary obstruction
Acquired Cysts Echinococcal cysts( Hydatid cause by a parasite) Lab Values?

Normal
How are LFT’s with Polycystic disease?

Normal
Cysts-congenital, how are LFT’S?

Decreased glucose-6 phosphatase
Glycogen Storage Disease, Von Gierkes disease most common type, Lab Values?

Normal or slightly abnormal LFT”S
Chronic Hepatitis Congestion-Lab Values?

LFT’S depend upon stage, increased ALT-AST-Alkaline Phosphatase-serum and urine conjugated billirubin values.
With Cirrhosis, how are lab values effected and which ones?

Increase in billirubin, ALT, AST, and alkaline phosphatase
With Chronic Viral Hepatitis, what lab values increase?

Increased billirubin, ALT higher levels than AST, alkaline Phosphatase
With acute viral hepatitis, what lab values effected?

Increased LFTS due to hepatocellular disease
Lab values with Fatty liver?

Serum amylase twice the normal values
Lab values with acute pancreatitis?

lymphocele
With a two week old renal transplant there are fluid collections with septations and internal debris adjacent to kidney, what is this?

interlobar
what intrarenal arteries course alongside renal pyramids

arcuate
what arteries course on top of the renal pyramids and give rise to the tiny interlobar arteries?

anterior
right renal vein lies____________ to the renal artery

posterior
left renal artery normally located __ to left renal vein

external iliac artery
where is the renal artery anastomosed to in renal transplant?

iliac fossa
where do you look for pancreas after transplant of?

phlegmon
what is a nonencapsulated collection of necrotic and edematous peripancreatic tissue called?

obstruction of pancreatic duct by biliary calculi
most common cause of acute pancreatitis?

SMV
which vessel courses anterior to the ucinate process?

anechoic acoustic enhancement, sharply defined smooth wall, round or ovoid shape
sonographic criteria for a simple cyst in kidney?

superior mesenteric and splenic
what vessels form the portal vein?

celiac trunk
Vessel to study with median arcuate syndrome?

right atrium
where does ivc empty into?

postprandial abd pain and weight loss
symptoms of chronic mesenteric ischemia

PV and hepatic vein
with a tips shunt, it connects what two vessels?

posterior to ivc
location of rt renal artery?

splenic
smv joins which vein?

orchitis
lg complex hydrocele commonly associated with?

testicular
capsular artery of teste is a branch of what artery?

abdominal aorta
testicular artery is a branch of?

peripheral
which zone of prostate involved with cancer?

peripheral zone
with transrectal prostate exam, the most lateral images of the gland show what tissue?

ejaculatory duct cyst
what cyst of the prostate is associated with infertility?

splenic vein
what structure will you detect at splenic hilum?

inferomedial
relationship of pancreatic tail to spleen?

intraperitoneal
location of spleen?

portal vein
what does splenic vein drain into?

abd aorta
what structure do we image to identify retroperitoneal fibrosis?

adrenal glands
what retroperitoneal structure is a common site for metastasis from lungs?

lateral to aorta and diaphragmitic crus
location of adrenal gland?

post to ivc and right renal artery
location of right diaphragmatic crus?

1cm
lymph nodes > than _ considered to be abnormal

anterior perirenal space
what retroperitoneal compartment is frequently involved with pseudocyst?

anterior para renal space
what retroperitoneal compartment contains the aorta?

5
how many layers are seen with good imaging of the gut?

adenocarcinoma
most common malignant tumor of the gi tract?

cecum
orfice of appendix opens up into the ?

increased conjugated billirubin, alkaline phosphatase, amylase and lipase and GGT
lab values for pancreatic carcinoma?

increased ALT,AST and conjugated and unconjugated billirubin.
lab values with jaundice?

BHCG
what lab is elevated with seminoma?

AST (SGOT)
with viral hepatitis highest level of what?

HCG
with choriocarcinoma have high levels of what?

hepatocellular and obstructive disease
AST increases with what disease> (SGOT)?

increase AST(SGOT) ALT (SGPT) and billirubin
lab values with hep a, b, or c?

AST, ALT and billirubin
with cirrhosis what lab values increase?

schistosomiasis
when you see echogenic bands while scanning along intrahepatic pv, what is this?

Increased billirubin and alkaline phosphatase
lab values increase with metastasis?

fetal antigen
what is present with 50% of pts with hematomas?

intrahepatic ducts
which ducts are associated with Caroli disease?

ucinate process
portion of pancreas that lies posterior to the sma and vein is the ?

retroperitoneum
location of pancreas?

anterior and superior to the pancreas
location of lesser sac?

indirect billirubin
hemolytic disorders may result in increase of what?

adrenal hemmorrhage
what is the most common neonatal adrenal mass?

neuroblastoma
what is most common childhood adrenal mass?

lung
what is the most common primary carcinoma associated with adrenal metastasis?

crus of diaphragm
what structure lies directly medial to both adrenal glands?

perirenal space
adrenal glands lie within the _

anterior pararenal space
ivc and aorta are located in which retroperitoneum space?

posterior parietal peritoneum and posterior abd wall muscles.
definition of retroperitoneal space?

hydronephrosis
most common secondary finding with retroperitoneal fibrosis?

left renal vein
which vessel is imaged between the SMA and aorta?

left gonadal vein
which vessel does not drain directly into IVC?

Superior vena cava
the azygos vein connects the proximal abd ivc to the?

adenoma’s
the halo sign is defined as a sonolucent rim surrounding a thyroid mass, commonly seen in?

anterior to trachea
location of thyroglossal duct cysts?

lymphoma
the most common primary source of testicular metastasis is?

intussusception
a 4 month old presents with abd pain, rectal bleeding,and palpable abd mass, what is most likely the pathology>

alanine amniotransferase
SGPT is same lab value as?

hepatic metastasis
what is most likely the diagnosis in a pt with elevation of CEA, alkaline phosphatase and GGT?

posterior superior
seminal vesicles are located on which surface of the prostate?

hepatitis
what is most likely the diagnosis with a pt with elevated AST and ALT?

Gastric Hypersecretion and peptic ulcer disease
Islet cell tumors are related to what two diseases?

hyperlipidemia and hypercalcemia
Predisposing factors to chronic pancreatitis?

fluid filled stomach, dilated pancreatic duct or tortuous dilated left portal vein.
What can mimic and pacreatic psuedocyst/

spreading diffuse inflammatory edema of subcutaneous connective tissue that can lead to necrosis
what is phlegmonous pancreatitis?

increased serum amylase lipase, decreased hematocrit and serum calcium levels also seen.
what are the lab values for hemmorhagic pancreatitis?

acute adult respiratory distress syndrome and acute tubular necrosis
what two things will often be associated with pancreatitis?

insulin glucagon and gastrin and do not forget it
what are the endocrine secretions of the pancreas?

less than two millimeters
what is the normal pancreatic duct measurement?

anterior to portal vein and to the left of the bile duct
proper hepatic artery courses__________ to portal vein and to the _____of bile duct

anterior
splenic artery is in what relation to the splenic vein?

posterior border
splenic vein is at the_________ border of tail of pancreas

portal vein
superior mesenteric vein and splenic vein join to form what vein?

anterior to
antrum of the stomach lies where to the neck and body region of pancreas?

9-15 centimeters
what are the normal liver measurements?

3 by 2 by 2 centimeters
what are the normal teste measurements?

2 to 8 millimeters
what is normal scrotal wall thickness?

7 to 10 centimeters in length
what is normal length of the gallbladder?

2.5 to 4.0 centimeters
what is normal diameter of the gallbladder?

12.5 to 15 centimeters in length, head is 2 to 3.5 centimeters and body and tail is 2 to 3 centimeters.
what are the normal measurements of the pancreas?

amylase and lipase
what lab values are elevated with pancreatic disease?

right middle and left hepatic arterys
after the proper hepatic artery enters the liver at the porta hepatis, it then divides into what?

anterior to portal vein and posterior to bile duct
right hepatic artery is generally seen lying anterior and posterior to what?

chronic cholecystitis
milk of calcium bile is associated with what?

ulcerative colitis
what is portal vein gas associated with?

necrotizing entercolitis
in infants what is portal vein gas due to?

pathology in distant common bile duct
what is biliary obstruction due to?

gallstones and carcinoma of head of pancreas
two most common lesions associated with biliary obstruction?

anterior surface of psoas muscles
location of ureters?

the splenic vein
what does the inferior mesenteric vein drain into?

renal corpuscle
what do we call the glomerulus and bowmans capsule together?

calculi, benign prostatic hypertrophy, prostate cancer, pelvic malignancies, pregnancy, and uteropelvic junction obstructions.
what are some of the causes of hydronephrosis?

on the right
where are the azygos veins located?

on the left
where are the hemiazygos veins located?

dense fibrous tissue proliferation confined to paravertebral region, Also known as inflammatory aneurysm or ormonds disease.
what is retroperitoneal fibrosis?

aorta to renal artery to segmental artery to interlobar arteries to arcuate arteries to interlobar arteriole to afferent arteriole
what are the branches of the renal artery?

on the top of the pyramids
what is the location of the arcuate arteries?

between the pyramids and then curves around the top of the pyramids.
what is the location of the interlobar arteries?

the interlobar arteries course through the cortex
what do the interlobar arteries course through?

it is a tube connecting the renal pelvis to the calyx steven and don’t forget this for the test
what is the infundibulum or major calyx?

bilateral renal agenisis
hello steven, what is classic potters syndrome?

right upper quadrant mass with painless jaundice indicates there is cancer in the head of the pancreas causing biliary obstruction, the palpable mass is due to an enlarged gallbladder buddy.
what is courvoisiers sign and what is it associated with?

the splenic vein and superior and inferior mesenteric veins my friend.
okay, what will the portal of confluence give you?

klatskins tumor
what is a malignant tumor of the common hepatic duct biff called?

right hepatic duct and left hepatic duct will be dilated and don’t forget this for the test buddy.
when you have a malignant tumor at the biff of hepatic duct biff. klatskins tumor, what will be dilated?

bifurcates into the proper hepatic artery and the gastroduodenal artery
as the common hepatic artery courses toward the liver, what does it bifurcate into?

common hepatic artery and splenic artery which is the seagull sign.
the celic axis terminates with the bifurcation of what?

branches of celiac axis are the common hepatic, left gastric and splenic artery.
what are the branches of the celiac axis?

celiac axis and it is located superior to the pancreas
what is the first branch of the aorta and its location?

between the ivc and azygos vein posterior
what is the location of the right crus?

large gallbladder filled with mucous in the wall and measures more than five centimeters and is caused by chronic obstruction of neck or cystic duct of gallbladder by stones or a tumor.
what is hydrops of the gallbladder?

vitelline vein
where did the portal vein come from?

abscess due to poorly defined border is the key, if it was a cyst, it would have well defined borders
patient with a high fever and right upper quadrant pain has a large hemogenous mass with low level echoes and poorly defined borders in right lobe with increased through transmission, what is it?

aortic arch
on the left side the common carotid artery originates from where?

phlegmon
what is a nonencapsulated collection of necrotic and edematous peripancreatic tissue termed?

it will have a normal appearance
most common ultrasound appearance of acute pylonephritis?

hepatitis, increased alt, ast, and conjugated and unconjugated billirubin.
what is a nonobstructive hepatocellular cause of jaundice and what lab values will be elevated?

vesicouterine
what is the space between the bladder and uterus called?

comes from right hepatic artery, is on ardms test for sure so don’t forget it!
the cystic artery supplies blood to the gallbladder and comes from where?

hepatoma, secondary is metastasis.
what is the most common primary tumor of the liver?

splenic and common hepatic artery and don’t forget as on the test
what is the seagull wing made up of?

stomach, left kidney, pancreas and splenic flexure of the colon.
what is located on the visceral surface of the spleen?

splenic, left gastric and common hepatic
what are the branches of the celiac trunk?

superior mesenteric artery
what artery is above the renal arteries?

medial and anterior
fundus of the stomach and lesser sac are in what relation to the spleenic hilum?

histoplasmosis and tb
what are the most common causes of granulomas?

in the liver and lungs
where can granulomas be found?

hydatid cysts, epidermoid cysts, autosomal dominant polycystic disease, hematomas and pancreatic pseudocysts.
what are some examples of cystic lesions found on the spleen?

true cysts lined by the squamous epithelium
what are epidermoid cysts?

erode into spleen, may weaken vessels causing pseudoaneurysms and bleeding into pseudocysts
pancreatic pseudocysts can do what?

general abdominal sepsis
spleenic infection is associated with what?

anterior and medially
when the spleen enlarges it extends in what direction?

a calcified circle in the left upper quadrant
what is a splenic artery aneurysm seen as?

polysplenia and asplenia
what are two classifications of heterotaxia?

symptoms of congestive heart failure or jaundice due to biliary tract abnormalities
presentation of heterotaxia in a neonatal patient?

lesser sac, between the liver, pancreas and stomach. and greater sac the rest of peritoneum cavity, when imaging floating bowel, this is greater sac.
what are the two separate compartments of the peritoneal cavity and their locations?

epiploic foramen
what is the entrance to the lesser sac called?

stomach, liver, spleen, first part of duodenum, kidneys, appendix, cecum, sigmoid colon uterus, ovaries and fallopian tubes, aorta,
What are the retroperitoneal structures? Hint, Neumonic.. (Some live splendid during kings and castles singing under open flowers always)

anterior pararenal space, perirenal space and posterior pararenal space.
two layers of renal fascia divides the retroperitoneum into what three compartments?

central, transitional and peripheral
what are the three zones of the prostate?

corpora amylacea
what are the calcifications in the inner gland of the prostate called?

verumontanum
what are small elevations of urethral crest in which orfices of ejaculatory ducts are located on either side?

pass through the central zone and not the twilight zone like I feel like where I am at studying this stuff. and empty into the urethra
ejaculatory ducts pass through what zone of the prostate and empty into what?

Adenomyomatosis, just remember A for Aschoff buddy.
Rokitansky Aschoff sinuses are associated with what conditon?

superior mesenteric vein
during isonation of the pancreas,you routinely image a vessel coursing anterior to the ucinate process,what is this vessel?

the left renal vein can be seen coursing transversely
what structure can be seen coursiing transversely at the level of the upper pancreatic head?

junction of the cystic duct and common hepatic duct
what forms the common bile duct?

avascular mass with low level echoes
what is the appearance of tumefactive sludge within the gallbladder?

caput medusae
paraumbilical vein drains into a network of varices that can be seen on the abdomen radiating from the umbilicus, these varices are known as what?

greater than three point five
renal aortic stenosis ratios greater than what is detection of stenosis?

superior mesenteric and splenic veins
what is the portal vein formed by, junction of what two veins?

posterior to the body of the pancreas
what is the location of the superior mesentric artery?

at the aortic bifurcation
generally retroperitoneal fibrosis is centered where?

in perirenal space, located superior, anterior and medial aspect of the kidney
what is the location of the adrenal glands?

hypoechoic midline mass
sonographic appearance of retroperitoneal fibrosis?

bilateral ureteral obstruction
what is retroperitoneal fibrosis associated with?

medial and posterior
what is location of crus of diaphragm in relation to the right adrenal gland?

anterior to the aorta, superior to the celiac axis and posterior to the ivc
crus of diaphragm is located what to the aorta, what to the celiac axis and what to the ivc?

kidneys, adrenal glands and proximal ureter
perirenal space is separated by two fascia layers and contains?

splenic vein and the tail of the pancreas
the left pararenal space includes the ———vein and ?

ivc, pancreas and segments of the duodenum.
what is located within the right pararenal space?

lymph nodes, fat, vessels of abdomen and pelvis, ureters and segments of colon
anterior renal space extends into the pelvis and contains?

Renal artery
Crus of diaphragm is seen as a hypoechoic tubular structure posterior to what?

perirenal space, posterior pararenal space and anterior pararenal space.
What are the three segments of the retroperitoneum, ppa as a clue.

hodgkins and non hodgkins
most common sources of lymphadenopathy?

SMA and celic vessels
Enlargement of the lymph nodes in the para aortic region may cause displacement of ??

appears as bulky midline mass and pt presents with abdominal pain, back pain, nausea and vomiting, also hypertension and weight loss.
Retroperitoneal fibrosis appears as and patient presents with what?

splenic artery divides into the common hepatic artery, splenic artery and left gastric artery.
celiac artery divides into the common hepatic artery and the?

left hemidiaphragm
at the superior border of the spleen is what?

posterior superior surface of liver
location of caudate lobe?

renal sinus, pancreas, liver, renal parenchyma
decreasing order of echogenicity?

fold at the gallbladder fundus
a phyrgian cap is a?

dilated intrahepatic ducts
what is associated with Caroli disease?

posterior surface of the pancreatic body and tail
throughout its course, the splenic vein is located on the?

ucinate process
the portion of the pancreas that lies posterior to the superior mesenteric artery and vein is?

retroperitoneum
location of pancreas?

second portion of the duodenum
common bile duct is joined with the pancreatic duct before entering what?

the lesser sac is located anterior and superior to the pancreas
what is the location of the lesser sac?

left renal vein thrombosis
what will displace the superior mesenteric artery anteriorly?

splenic vein, portal vein, right portal vein, emissary veins, ivc, and the right atrium.
what is the correct path of a red blood cell returning to the heart? ( Send Past Right emissary in Rio for a hint)

sickle cell anemia
what disease would result in splenic atrophy ( autosplenectomy)

portal hypertension
what is most common cause of splenomegaly?

multicystic dysplastic kidney
what is the most commonly reported cause of palpable abdominal mass in the newborn?

upper pole hydronephrosis
what is a complication of uretocele?

the liver.
what organ is most likely to be affected in polycystic kidney disease?

infantile polycystic kidneys
bilateral echogenic kidneys in newborns most likely represent?

acute tubular necrosis
most common cause of acute renal failure?

protein, creatinine, and bun
What are the lab values associated with renal failure?

acid phosphatase
what lab value is associated with prostate cancer?

ALT ( alanine amniotransferase)
what lab value is the most sensitive indicator of hepatocellular disease?

increase in direct billirubin and slight increase in indirect billirubin.
choledocholithiasis may result in an increase in what?

a predominant increase in direct billirubin
hemolytic disorders may result in an increase in what?

Pheochromocytoma
A patient presents with hypertension, urinary epinephrine and norepinephrine and an adrenal mass, what is the adrenal mass?

Adrenal Hemmorrhage
what is the most common neonatal adrenal mass?

neuroblastoma
what is the most common childhood adrenal mass?

The Lung
what is the most common primary carcinoma associated with adrenal metastasis?

In the perirenal space
where do the adrenal glands lie?

Chronic adrenal hypofunction
Addison’s disease is a syndrome that results from?

Crus of the diaphragm
what structure lies directly medial to both adrenal glands?

Cushings syndrome
what syndrome is associated with adrenal mass?

Anterior pararenal space
The IVC and Aorta are located in which retroperitoneal space?

No
Is the spleen retroperitoneal?

Posterior parietal peritoneum and posterior abdominal wall muscles.
Define the retroperitoneal space

Malignant
Primary retroperitoneal tumors are generally benign or malignant?

hydronephrosis
If retroperitoneal fibrosis is suspected, what is the most common secondary finding?

splenic vein
what does the inferior mesenteric vein drain into?

Left renal vein
which vessel is imaged between the SMA and Aorta?

Left gonadal veing
which vessel doesn’t drain directly into the IVC?

proximal abdominal IVC to the superior vena cava
what does the azygos vein connect?

Adenomas
The halo sign is defined as a sonolucent rim surrounding a thyroid mass, is is commonly seen in what?

hypercalcemia
what may parathyroid adenoma’s be associated with?

papillary cancer
what is the most common primary thyroid cancer?

Anterior to the trachea
location of thyroglossal duct cysts?

seminoma
what is the most common testicular tumor?

lymphoma
what is the most common primary source of testicular metastasis?

epididymitis
what is the most common cause of an acute scrotum in a postpubertal male?

tunica vaginalis
hydroceles can be defined as serous fluid accumulated between the two layers of?

seminiferous tubules, tubuli recti, rete testes, efferent ductules, epididymis, vas deferens
what is the correct pathway of sperm from the testes?

anterior to the aorta and posterior to the left lobe of the liver.
sonographically the gastroesophageal junction can be visualized where?

Hepatic metastasis
what is most likely the diagnosis in a patient with elevation of CEA, alkaline phosphatase and GGT?

Biliary obstruction
Elevation of both GGT and ALP would most likely represent what?

Laterally
A right adrenal mass will displace the superior pole of the kidney how?

Alanine amniotransferase
SGPT is the same lab value as what

perpheral zone
Largest percent of cancers originate from which area of the prostate?

She is pregnant, at least I hope it is a she…ha ha
the patient has a elevated ALP and normal GGT, what is going on?

Peripherally located hypoechoic lesion
What is classic appearance of prostate cancer?

transition zone
Benign prostatic hypertrophy originates from which zone?

posterior superior surface
the seminal vesicles are located on which surface of the pancreas?

Hepatitis
what is most likely the diagnosis in a patient with elevated AST and ALT?

GREENFIELD FILTER
MOST COMMON IVC FILTER?

RENAL CELL CARCINOMA
MOST COMMON TUMOR TO INVADE IVC?

BETWEEN LIVER, PANCREAS AND STOMACH
LOCATION OF LESSER SAC?

REST OF PERITONEUM CAVITY, WHEN IMAGING ASCITES WITH FLOATING BOWEL THIS IS GREATER SAC
LOCATION OF GREATER SAC?

CALCIFIED CIRCLE IS SEEN IN LEFT UPPER QUAD ON X RAY
TYPICALLY HOW IS A SPLENIC ARTERY ANEURYSM SEEN?

BACTERIAL ENDOCARDITIS AND SPLENIC ARTERY ANEURYSMS
SPLENIC INFARCT ARE COMMON WITH PATIENTS WHO HAVE WHAT?

INFERIOR AND MEDIAL
WHAT POSITION DOES THE LEFT KIDNEY LIE TO THE SPLEEN?

MEDIAL AND ANTERIOR
FUNDUS OF THE STOMACH AND LESSER SAC ARE _ AND ___ TO THE SPLENIC HILUM

PERITONEAL AND BETWEEN STOMACH AND DIAPHRAGM
SPLEEN IS A ____ORGAN LOCATED BETWEEN ______ AND _

12 CM SAG, 8 CM TRANS, 4 CM THICKNESS
NORMAL SPLEEN MEASUREMENTS? SAG, TRANS AND THICKNESS?

SERUM ACID PHOSPHATASE
WHAT IS ELEVATED IN ASSOC WITH PROSTATE CANCER?

VIA INTERNAL PUDENDAL, INF VESICAL AND MIDDLE RECTAL ARTERIES, ARE BRANCHES OF ILIAC ARTERIES
WHAT IS THE ARTERIAL SUPPLY FOR THE PROSTATE?

ADENOMATOID
WHAT IS MOST COMMON EXTRATESTICULAR TUMOR?

VAS DEFERENS, CREMASTERIC, DIFFERENTIAL, TESTICULAR ARTERIES, PAMPINIFORM PLEXUS OF VEINS AND LYMPHATICS AND NERVES
WHAT DOES THE SPERMATIC CORD CONSIST OF?

CAPSULAR AND CENTRIPETAL ARTERIES
TESTICULAR ARTERIES DIVIDE INTO _ AND ____ BRANCHES

SCROTAL RAPHE
DIVISION OF TWO SCROTAL CHAMBERS IS CALLED?

GFR
SERUM CONCENTRATION IS INVERSELY RELATED TO

ILIAC FOSSA
TRANSPLANTED KIDNEY IS USUALLY PLACED WHERE?

SMALL HARD PULSE WHICH RISES AND FALLS SLOWLY
WHAT IS PULSUS PARVIS ET TARDUS?

LESS THAN 3MM DISTENDED AND LESS THAN 5 MM UNDISTENDED
WHAT IS NORMAL BLADDER WALL THICKNESS, DISTENDED AND NON DISTENDED?

ISCHEMIA OF MEDULLARY PYRAMIDS, DIAGNOSED BY SLOUGHED PAPILLA IN THE URINE.
WHAT IS PAPILLARY NECROSIS? AND HOW DIAGNOSED?

ANALGESIC ABUSE
PAILLARY NECROSIS IS ASSOC WITH WHICH MOST COMMON CONDITION?

WILMS(NEPHROBLASTOMA) 3.5 YEARS
MOST COMMON CHILDHOOD RENAL TUMOR?

MULTISYSTEM GENETIC DISEASE WITH BENIGN TUMORS GROWING IN ORGANS, IE: BRAIN, KIDNEYS HEART, EYES AND LUNGS
WHAT IS TUBEROUS SCLEROSIS?

MULTICYSTIC DYSPLASTIC KIDNEY, IS EITHER UNILATERAL OR BILATERAL
MOST COMMON CAUSE OF ABDOMINAL MASS IN NEWBORN?

HYPERTROPHY OF RENAL CORTICAL PARENCHYMA LOCATED BETWEEN 2 MEDULLARY PYRAMIDS
WHAT IS COLUMNS OF BERTIN?

TRIANGULAR HYPERECHOIC AREA ON ANT ASPECT OF UPPER POLE OF RT KIDNEY
JUNCTIONAL PARENCHYMAL DEFECT?

LEFT KIDNEY, LATERAL ASPECT
DROMEDARY HUMP EFFECTS WHICH KIDNEY?

CROSSED FUSED RENAL ECTOPIA, TWO KIDNEYS VISUALIZED ON ONE SIDE OF ABDOMEN
SECOND MOST COMMON RENAL FUSION ANAMOLY?

PELVIC KIDNEY, THEY DIDN’T ARISE INTO ABDOMEN
WHAT DOES ECTOPIC KIDNEY MEAN?

5 SEGMENTAL ARTERIES
AT THE HILUM MAIN RENAL ARTERIES DIVIDE INTO?

GLOMERULUS AND GLOMERULUS CAPSULE, (BOWMANS CAPSULE)
WHAT DOES THE RENAL CORPUSCLE(MALPIGHIAN BODY) CONSIST OF?

MEDULLARY PYRAMIDS
RENAL PAPILLA ARE APEX OF?

RENAL PELVIS
MAJOR CALYCES ARE 3 EXTENSIONS OF THE?

POSTERIOR ASPECT OF KIDNEYS
LOCATION OF QUADRATUS LUMBORUM AND PSOAS MUSCLES IN RELATION TO KIDNEY?

SUPEROMEDIAL
IN RELATION TO THE RIGHT KIDNEY, THE ADRENAL GLAND IS ?

MULTIPLE ENDOCRINE NEOPLASIA IS AN INHERITED CANCER SYNDROME
WHAT IS MULTIPLE ENDOCRINE NEOPLASIA?

GASTRINOMA, FREQ MALIGNANT
SECOND MOST COMMON ISLET CELL TUMOR OF PANCREAS? BENIGN OR MALIGNANT?

INSULINOMA
MOST COMMON ISLET CELL TUMOR?

SERUS CYSTADENOMA-ENIGN AND MUCINOUS CAN BE EITHER MALIGNANT OR BENIGN
ASS WITH VON HIPPEL LINDAU SYNDROME
TWO MOST COMMON CYSTIC NEOPLASMS OF THE PANCREAS? ASS WITH WHAT SYNDROME?

< 2 %
WHAT IS THE FIVE YEAR SURVIVAL RATE % WITH PANCREATIC CARCINOMA?

FAILURE OF PANCREATIC DUCTS TO FUSE DURING DEVELOPMENT
WHAT IS PANCREATIC DIVISUM?

ABDOMINAL TRAUMA
IN CHILDREN WHAT IS THE MOST COMMON REASON FOR PANCREATIC PSEUDOCYST?

ANTERIOR PARARENAL SPACE
WHAT SPACE IS THE PANCREAS LOCATED?

ADULT POLYCYSTIC KIDNEY DISEASE OR VON HIPPEL LINDAU SYNDROME
MULTIPLE PANCREATIC CYSTS ARE ASS WITH?

NO
IS CHRONIC PANCREATITIS REVERSABLE?

STONES WITHIN WIRSUNGS DUCT FROM CHRONIC PANCREATITIS OR STONE AT AMP OF VATER
PANCREATIC DUCT DILATION IS TYPICALLY DUE TO?

2MM OR LESS
NORMAL PANCREATIC DUCT MEASUREMENT?

SUPERIOR MESENTERIC
SPLENIC VEIN JOINS WHAT VEIN TO CREATE PORTAL VEIN?

PROPER HEPATIC ARTERY
RIGHT GASTRIC ARTERY IS A BRANCH OF?

DILATION OF INTRAHEPATIC DUCTS
WHAT IS CAROLI’S DISEASE?

CHOLANGIOCARCINOMA LOCATED AT SPLENIC HILUM, ON INTRAHEPATIC DUCTS DILATE
KLATSKINS TUMOR?, WHAT DILATES?

EXTRAHEPATIC BILIARY OBSTRUCTION DUE TO IMPACTED STONE IN CYSTIC DUCT
WHAT IS MIRIZZI’S SYNDROME?

ONLY PROXIMAL DUCTS
WITH COMMON HEPATIC OBSTRUCTION , WHAT WILL BE DILATED?

DILATED HEPATIC DUCTS ADJACENT TO THE PORTAL VEIN
WHAT DOES THE PARALLEL CHANNEL SIGN OR SHOTGUN SIGN REFER TO?

MUCOCELE OF GB
HYDROPS ALSO KNOWN AS?

LEVEL OF AMPULLA OF VATER
WITH ACUTE CHOLECYSTITIS, AMYLASE INCREASE SUGGESTS OBSTRUCTION IS WHERE?

BILLIRUBIN, ALT, AST, ALKALINE PHOSPHATASE
WITH CHRONIC VIRAL HEPATITIS, WHAT LAB VALUES INCREASE?

CBD AND MAIN PANCREATIC DUCT
WHAT FORMS THE AMPULLA OF VATER?

CYSTIC DUCT AND COMMON HEPATIC
WHAT FORMS THE CBD?

12 X 7X 3
NORMAL SPLEEN MEASUREMENTS

2 X 4 X 3
NORMAL PROSTATE MEASUREMENTS?

???
what forms the ampulla of vater?

Which of the following is not a factor when choosing a high level disinfectant?

a. exposure to sunlight
b. temperature
c. disinfection time
d. ventilation requirements

A
A
The factors to consider when choosing a high level disinfectant are: transducer compatibility, ventilation requirements, disinfection time, rinsing requirements, temperature, re-use period, cycle cost, neutralization requirements.

2
Q
Which type of kidney infection is considered a critical finding?

a. acute pyelonephritis
b. glomerulonephritis
c. emphysematous pyelonephritis
d. chronic pyelonephritis

A
C
Emphysematous Pyelonephritis is considered a critical finding on a renal US. If not treated early, it may lead to fulminant sepsis and carries a high mortality.

3
Q
All of the following are techniques that can be used to assist an Alzheimer’s patient during an US exam, except:

a. allow a family member to accompany them during their exam.
b. keep the exam room door open and the lights on during the exam in case you need to get help quickly.
c. have the patient’s nurse alert you to helpful techniques used with this patient.
d. bring something that is usually familiar to the patient into the exam room.

A
B
There is no need to compromise patient privacy and exam performance due to patient condition. Always consult the patient’s nurse or family member for any special considerations needed while performing the exam.

4
Q
Put the following in order in decreasing echogenicity:

a. pancreas, spleen, liver, renal sinus, renal parenchyma
b. renal parenchyma, spleen, liver, pancreas, renal sinus
c. renal sinus, pancreas, liver, spleen, renal parenchyma
d. renal sinus, renal parenchyma, liver, spleen, pancreas

A
C
The order of echogenicities in the abdomen from most echogenic to least; A P L S P = renal sinus > pancreas > liver > or = spleen > renal parenchyma.

5
Q
Which of the following is a contraindication for a renal biopsy?

a. current Warfarin use
b. current antibiotic treatment
c. AIDS
d. Polycystic kidney disease

A
A

Patients who are currently taking a blood thinner’s cannot have a biopsy unless they can stop the meds for several days.

6
Q
All of the following are usually part of the normal paracentesis procedure, except?

a. post-procedure chest x-ray
b. puncture usually made lateral to the mammary line
c. only a small amount of fluid is removed for a diagnostic procedure
d. removal of up to 6 liters of fluid in one day

A
A

A pre- and post-procedure x-ray are normally performed for a Thoracentesis.

7
Q
Which blood vessel is located lateral to the left lobe of the thyroid and demonstrates an anechoic ovoid shape in a transverse view of the mid thyroid?

a. left external carotid artery
b. left internal jugular vein
c. left CCA
d. superior vena cava

A
B
The internal jugular vein is located lateral to the left lobe of the thyroid and demonstrates an anechoic ovoid shape in a transverse view of the mid thyroid. The carotid artery is also lateral to the thyroid but it should be a circular structure.

8
Q
Which of the following is the least likely sonographic characteristic of the papillary carcinoma of the thyroid?

a. hypervascularity
b. multiple cystic areas within the mass
c. cervical lymphadenopathy
d. microcalcifications

A
B
Sonographic characteristics of Papillary Cancer of the thyroid include heterogeneous, hypo echoic solid mass with microcalcifications. The mass is hypervascular in most cases. It is commonly associated with enlarged cervical lymph nodes due to metastasis. Cystic degeneration of the mass is very rare.

9
Q
Which of the following vessels can be identified as a circular structure immediately posterior to the neck of the pancreas?

a. splenic vein
b. celiac axis
c. SMA
d. IVC

A
C
The splenic vein would be identified as a tubular structure that courses posterior to the neck of the pancreas. The IVC is posterior to the head. The celiac axis is superior to the head of the pancreas.

10
Q
Which liver disorder leads to the sonographic appearance of hepatomegaly with decreased echogenicity and scattered bright portal reflections within the parenchyma?

a. cirrhosis
b. acute hepatitis
c. chronic hepatitis
d. hydatid disease

A
B
Acute hepatitis demonstrated the sonographic appearance of hepatomegaly with decreased echogenicity and scattered bright portal reflections within the parechyma (starry night).

11
Q
You are evaluating an obese patient with a known AAA. While using color doppler to assess flow within the aneurysm, minimal flow is detected in the anechoic lumen. You decrease the color scale and increase the color gain but there is no improvement. Which of the following adjustments will most improve the color display on this exam?

a. switch from a 4MHz curved array to a 4MHz linear array
b. increase color sample size
c. decrease the color threshold
d. increase the color threshold

A
D
Color threshold (AKA priority) determines the limit of when color will be displayed over the gray scale image. Decreasing the threshold would reduce the amount of color displayed in slow flowing vessels. Increasing the color threshold will increase the amount of color displayed. Smaller sample sizes produce a better color display.
12
Q
Which of the following is correct referring to cleaning a transducer as part of a high level disinfection procedure?

a. dampen soft cloth to cleanse probe with mild liquid soap
b. use just running water to remove any debris or gel
c. rinse probe with alcohol
d. dry probe with toilet paper

A
A or B? needs research

You need to remove probe cover and use running water to remove any debris or gel. Dry probe with a soft cloth or paper towel.

13
Q
Which of the following is a correct statement describing the guidelines for wearing a mask?

a. remove gloves, wash hands and then remove mask
b. it is okay to touch the parts of the mask that will touch your face
c. tie upper strings across the top of your head and the lower stings around your neck
d. when finished, remove the gloves first, and then untie the bottom strings, followed by the top strings.

A
D
When finished, remove gloves first, and then untie the bottom strings, followed by the top strings. Do not touch the part of the mask that will touch your face. Tie the upper strings over the ears and towards the back of your head. Discard the mask properly and wash your hands.

14
Q
Which portions of the GB and/or biliary tree are involved in the formation of a Phrygian cap?

a. cystic duct and CBD
b. neck and body
c. neck and cystic duct
d. body and fundus

A
D

A Phrygian cap is when the fundus of the GB folds over the body of the GB.

15
Q
A pregnant woman presents with lateral LUQ pain and a bruit in the same area. What will be the most likely finding?

a. splenic aneurysm
b. splenic varices
c. pancreatic pseudocyst
d. IVC congestion due to uterine compression

A
A
Splenic artery aneurysms are the most common type of abdominal aneurysm. They are seen more in females than males and are associated with pregnancy. The risk of rupture makes it a critical finding requiring immediate intervention. Splenic varices would demonstrate low velocity flow that would not normally lead to a bruit.

16
Q
Which of the following liver disorders will result in increased levels of iron stored in the liver?

a. amyloid disease
b. hemochromatosis
c. glycogen storage disease
d. Wilson disease

A
B
Amyloidosis- increased levels of amyloid deposited in the liver.
Hemochromatosis- increased levels of iron deposited in the liver.
Wilson disease- increased levels of copper stored in the liver.

17
Q
__ is a hemolytic disorder that causes increased indirect bilirubin levels in the blood.

a. Kaposi’s sarcoma
b. acute hepatitis
c. Budd Chiari syndrome
d. Anemia

A
D
Hemolytic anemia refers to the abnormal destruction and reformation of RBC’s. This causes excessive amounts of RBC’s to be destroyed which releases the hemoglobin they should be carrying. The hemoglobin is then converted into bilirubin. The more RBC’s destroyed. the more bilirubin in the blood stream.

18
Q
When scanning a large habitus patient, a possible renal cyst is identified. Which of the following is a technique that would most likely be used to better visualize the finding?

a. switch to a higher frequency probe
b. drink water and rescan in 20-30 minutes
c. scan the patient in the decubitus position
d. use a standoff pad

A
C
The decubitus position allows for transducer placement closer to the kidney than from an anterior approach. Coronal and posterior probe placement can aid in better visualization of the kidney’s, especially in obese patients.

19
Q
If a liver mass is located between the middle hepatic vein and the right portal vein, in what lobe of the liver is the mass located?

a. anterior right lobe
b. medial left lobe
c. caudate lobe
d. posterior right lobe

A
A
The middle hepatic vein separates the right and left lobes. The right portal vein enters the center of the right lobe. A mass between the two structures would be located in the anterior right lobe.

20
Q
An unconscious patient is referred for an abdominal ultrasound through ER after a car accident. All abdominal organs appear normal in structure and echogenicity. There is free fluid in the right and left paracolic gutters with a complex mass of irregular tissue in the inferior midline pelvic cavity and no discernible bladder can be identified. What should you do when the exam is complete?

a. there is a rectus sheath hematoma present which is a non-critical finding
b. you should ask the nurse to insert a foley catheter and attempt to fill the bladder with saline solution
c. there is a possible bladder rupture and the radiologist should review the exam immediately
d. there is a potential bladder rupture and you should notify the referring physician immediately

A
C
The presence of free fluid in the pelvis and an irregular mass of tissue in the lower pelvis are indicators for possible bladder rupture. This is a critical finding and the case should be reviewed by a radiologist immediately. A sonographer should not offer exam results to the ER physician. Attempting to fill the bladder could assist in a pelvic exam but the physician must write an order for foley catheter insertion. If bladder rupture is suspected, it is highly unlikely a foley would be inserted. A rectus sheath hematoma would demonstrate fluid collection within the abdominal wall, not the cavity.

21
Q
A 60 year old presents with a history of sickle cell anemia. What do you expect to find of the US evaluation of the spleen?

a. agenesis of the spleen
b. splenic atrophy
c. splenomegaly
d. splenic artery aneurysm

A
B

Sickle cell anemia causes the spleen to enlarge early in life and then atrophy and shrink later in life.

22
Q
Elevated GGTP and ALP indicates . Elevated GGTP and ALT indicates .

a. hepatocellular disease, biliary obstruction
b. biliary obstruction, hepatocellular disease
c. hepatic carcinoma, hepatic adenoma
d. cirrhosis, hepatitis

A
B
GGTP: gamma-glutamyl transpeptidase is normally found in liver cells and biliary epithelium. It is the most sensitive indicator for alcoholism. A marked increase is seen with liver disease and post hepatic biliary obstruction. A moderate increase is seen with liver damage due to alcohol, drugs and chemotherapy.
Increased GGTP + ALP = biliary obstruction
Increased GGTP + ALT = hepatocellular disease

23
Q
A patient presents with a palpable lump adjacent to the umbilicus. Which of the following transducers would be best to use for this exam?

a. 3.5MHz linear array
b. 7MHz phased array
c. 7MHz linear array
d. 3.5MHz curvilinear array

A
C

High frequency linear arrays are preferred for superficial imaging. 7MHz linear array.

24
Q
All of the following techniques will aid in the visualization of a small renal calculi with questionable shadowing, except?

a. decreasing the dynamic range
b. using multiple scanning planes
c. decreasing the transducer frequency
d. rescanning the patient in the decubitus position

A
C

Increased transducer frequency will improve resolution of smaller objects.

25
Q
Which of the following correctly describes the preferred patient position for a renal biopsy?

a. decubitus with knees drawn to chest
b. supine with a pillow under the back to accentuate the normal curvature of the spine
c. prone with a pillow under the abdomen to accentuate the normal curvature of the spine
d. posterior oblique with ipsilateral arm extended over the head

A
C
The preferred position for the patient undergoing a renal biopsy is prone with a pillow under the abdomen to accentuate the normal curvature of the spine. This approach allows the needle to travel the shortest distance through other tissues before reaching the kidney. It also prevents puncture of bowel and other organs. An anterior approach is least preferable due to the bowel, liver, spleen and vessels that could be punctured in the procedure.

26
Q
A 45 year old patient presents with increased serum BUN and creatinine, a recent history of oliguria and bilateral flank pain. The US exam demonstrates bilateral hydronephrosis and hydroureter but the bladder is nearly empty. Which of the following statements is true regarding the patient and the findings?

a. acute intrinsic renal failure is present that is most likely due to acute tubular necrosis
b. acute post renal failure is present and the radiologist should review the exam immediately
c. the patient has a neurogenic bladder and the radiologist should review the exam immediately
d. the patient will most likely return in 7-10 days after antibiotic treatment

A
B
Bilateral hydronephrosis and hydroureter with increased serum levels of BUN and creatinine and an empty bladder most likely indicates acute renal failure. Acute renal failure caused by a bilateral ureteral obstruction can be treated by removing the obstructions. They parenchyma can be saved if the patient gets immediate treatment. A neurogenic bladder will not empty properly, causing increased bladder volumes pre- and post-void. Antibiotics are used to treat infections, not obstructions.

27
Q
Which of the following operator controls adjusts the beam intensity?

a. dynamic range
b. output
c. overall gain
d. TGC

A
B
The output control will increase/decrease the voltage applied to the transducer which affects the intensity of the beam accordingly. Harmonic imaging is a frequency compounding technique used to improve image resolution of the displayed information. The TGC and overall gain controls are used to amplify the reflected signal.

28
Q
When scanning the pancreas, which of the following techniques will not assist in reducing the gas artifact around the pancreas?

a. NPO 8-12 hours prior to exam
b. ask patient to perform the Valsalva maneuver
c. water ingestion
d. deep inspiration

A
B

The Valsalva maneuver will not usually affect the evaluation of the pancreas.

29
Q
A 50 year old patient presents with acute RUQ pain and elevated alkaline phosphatase and bilirubin. The GB appears normal and the CBD measures 11mm at the porta heptatis. What should be evaluated next for this patient’s exam?

a. pancreas for a mass in the head
b. pancreas for a mass in the tail
c. spleen for varices
d. liver metastasis

A
A
11mm CBD is too large and abnormal. The pancreas head should be evaluated for a reason that the CBD is dilated if the duct is clear.

30
Q
Which of the following is true regarding the difference between the sample obtained from a fine needle aspiration and a core biopsy sample taken from the liver?

a. FNA procedures are more traumatic to the tissues than needle biopsy procedures
b. The needle used for the FNA is usually much larger than the biopsy needle
c. The cytology lab evaluates the sample obtained from the FNA. The histology lab evaluates the sample obtained from the needle biopsy
d. Core needle biopsies are more commonly performed for liver masses than to assess parenchymal liver disease

A
C
The needle used for the FNA is usually much smaller than the biopsy needle. Core needle biopsies are more commonly performed to assess parenchymal disease than for liver mass evaluation. FNA procedures are less traumatic to the tissues than needle biopsy procedures.

31
Q
Which of the following is a contraindication for a liver biopsy?

a. patient with Budd-Chiari syndrome
b. patient with severe liver atrophy caused by cirrhosis
c. patient with severe allergies to iodinated contrast
d. patient with active Alzheimer’s disease and hepatitis

A
D
A patient with Alzheimer’s disease cannot sign the informed consent for the procedure and may be uncooperative.
Contraindications for sonography guided procedures:

  1. coagulopathy- use of blood thinners
  2. unsafe biopsy route- must avoid large vessels, bowel, trachea and adjacent organs.
  3. Uncooperative patient- uncontrolled movement during the procedure can result in laceration or hemorrhage.

32
Q
Which scrotal artery encircles the testicular periphery?

a. cremasteric
b. deferential
c. capsular
d. centripetal

A
C
Capsular artery courses along the testicle periphery and produces branches called centripetal arteries which course through the parenchyma. The deferential artery supplies the vas deferens and epididymis with blood. The cremasteric supplies the scrotal sac with blood.

33
Q
A patient presents with a recent single episode of significant indigestion, increased levels of AST and normal levels of ALT, alkaline phosphatase and bilirubin. Which of the following is a possible explanation of these symptoms?

a. myocardial infarction
b. chronic hepatitis C
c. Schistosomiasis
d. biliary stasis with a partial ductal obstruction due to Klatskin’s tumor

A
A
Increased levels of AST with normal ALT is associated with myocardial infarction, CHF, CNS disease and other non-hepatic disorders. Some patients that experience a myocardial infarction describe the radiating pain as heart burn or intense indigestion.

34
Q
A right renal mass will displace the IVC:

a. Inferiorly
b. Laterally
c. Superiorly
d. Medially

A
D

A right renal mass will cause medial displacement of the IVC.

35
Q
Potential bioeffects of US can be minimized by which of the following?

a. reduce exam time
b. using a lower frequency transducer
c. decrease overall gain
d. increase PRF

A
A
Reducing overall exam time will reduce patient exposure. Adhering to ALARA principle will also reduce exposure by limiting power output increases and using TGC contrast controls instead.

36
Q
While you are performing an echo, the patients nurse comes in to try to draw blood. The nurse dropped the tube and you get a splash of blood on your hand. What should you do?

a. Immediately wash hands with soap and water
b. use alcohol based sanitizer to disinfect hand
c. report to occupational health so they can test the blood on your hand for disease
d. report the clumsy actions of the nurse to her supervisor

A
A
Immediately wash the affected area with soap and water. Notify your supervisor and immediately report to occupational health to file an exposure report.

37
Q
Harmonic imaging:

a. is not recommended for use on fetal exams due to increased beam intensity
b. uses information from reflections created by the non-linear behavior of sound waves to form the image
c. improves spatial resolution but causes a reduction in temporal resolution
d. improves spatial resolution but causes a reduction in contrast resolution

A
B
Harmonic imaging does not affect the beam intensity so there is no restriction of its use. The process is performed by the machine processing more reflections to improve contrast resolution.

38
Q
What modality is commonly used in the characterization of a renal angiomyolipoma?

a. CT scan
b. angiography
c. x-ray
d. nuclear medicine

A
A

CT scan can detect the high fat composition of the mass.

39
Q
What is a potentially life threatening complication of portal HTN?

a. ascites formation
b. ruptured gastroesophageal varices
c. portal embolism
d. pulmonary embolism

A
B

Cavernous transformation causes the formation of varices that can rupture and cause significant hemorrhage, even death.

40
Q
Which of the following is incorrect regarding intraoperative US?

a. soaking the transducer in alcohol for 30 minutes prior to the procedure is sufficient sterilization before surgery
b. sterile gel is used between the transducer and sterile sheath
c. sterile gel is used between the sterile sheath on the transducer and the organ being scanned
d. ethylene oxide gas can be used to sterilize the transducer

A
C
Gel is not used as a coupling agent when scanning intraoperatively. Surface moisture is usually enough to allow sound transmission into the body but sterile saline can be used.

41
Q
A patient presents for a renal US due to high serum levels of anti-diuretic hormone, low urine output and a recent pulmonary embolism. What is the referring doctor most likely looking for on the exam?

a. renal vein thrombosis
b. formation of renal carbuncle
c. obstructive bladder calculi
d. abscess formation in the renal pelvis

A
A
Increased levels of ADH and low urine output indicates the patient is dehydrated. The recent PE indicates there is potential thrombus formation in the IVC or its branches.

42
Q
In cases of acute pancreatitis, which of the enzyme levels increases first?

a. Alkaline Phosphatase
b. Trypsin
c. Amylase
d. Lipase

A
C
Amylase usually increases within the first 3-6 hours of the onset while lipase usually increases within 48 hours of the onset of acute pancreatitis.

43
Q
Retrograde flow in the splenic artery would most likely indicate:

a. left gastric artery occlusion
b. chronic systemic HTN
c. mild portal HTN
d. celiac axis occlusion

A
D
If the celiac artery is occluded, the splenic artery will receive no blood because it is distal to the celiac origin. The lack of blood to the celiac artery will cause a significantly low pressure within the vessel that will allow collateral flow to occur. As the blood enters the empty splenic artery from a point distal to its origin from the celiac, it will flow back to the origin to the point of obstruction. Severe portal HTN could cause increased resistance and flow reversal in the splenic artery but mild portal HTN will have little effect on the splenic artery flow.

44
Q
In the normal fasting adult, blood sugar levels should not exceed _ of blood.

a. 50mg/100ml
b. 100mg/100ml
c. 150mg/100ml
d. 200mg/100ml

A
B

In a normal fasting adult, blood sugar should not exceed 100mg/100ml of blood.

45
Q
Which of the following is not a sonographic characteristic of prostatitis?

a. heterogenous gland
b. increased gland size
c. increased vascularity seen with color doppler
d. solid hypoechoic mass

A
D

Prostatitis is usually a diffuse process that affects the entire gland.

46
Q
Which of the following will improve the color display in the abdominal aorta?

a. increased frame rate
b. increased sample size
c. decrease color threshold
d. decrease color gain

A
A
Increased frame rate will increase temporal resolution that may reduce any visible delay in displayed information. Smaller sample sizes will increase frame rates. Color threshold determines the limit of when color is displayed over the 2D image. Increasing the threshold allows for more color to be displayed.

47
Q
Which 2 gut layers are normally hypoechoic on the US image?

a. submucosa, muscularis propria
b. serosa, superficial mucosa
c. deep mucosa, muscularis propia
d. serosa, muscularis propia

A
C
The gut signature (sonographically):
superficial mucosa- epithelial lining (and lumen); echogenic
deep mucosa- consists of loose connective tissue and muscularis mucosa; hypoechoic
submucosa- echogenic
muscularis propria- inner circular fibers and outer longitudinal fibers; hypoechoic
serosa or adventitia- echogenic

48
Q
What can be used to alter the displayed brightness of the image after it has been stored in the memory?

a. compression
b. rejection
c. pre-processing
d. post-processing

A
D
Post-processing is the only control that will adjust the brightness of the image AFTER the data is stored in the digital memory.

49
Q
The major branches of the celiac axis are:

a. left gastric, splenic, common hepatic artery
b. left gastric, SMA, GDA
c. hepatic, GDA, splenic
d. left gastric, common hepatic, GDA

A
A

The major branches of the celiac axis are the left gastric, splenic, common hepatic arteries.

50
Q
The US exam of a patient’s liver demonstrates a patent umbilical vein. The most common cause for this recanalization is:

a. hepatitis
b. abscess
c. portal HTN
d. ascites

A
C
Increased pressure in the liver and portal system causes them to engorge with blood. In some cases, this pressure is great enough to recanalize the umbilical vein outward to the umbilicus, as blood backs up in the system. The umbilical vein is a remnant of fetal circulation that used to connect the portal system of the liver to the placenta.

51
Q
You are scanning the liver of an average sized patient with known cirrhosis. You complete the exam using a 3.5MHz curvilinear transducer but the doctor would like a better evaluation of the liver surface nodularity. Which of the following transducers would provide the best evaluation of the liver surface?

a. 3.5MHz linear
b. 8MHz curvilinear
c. 12MHz linear
d. 5MHz linear

A
D
In order to balance resolution and penetration a 5-7MHz transducer should work best for liver surface evaluation in an average size patient. The linear array works best due to its large near field and rectangular field of view.

52
Q
A female patient presents with 8 days of abdominal pain, fever and nausea. The symptoms started 2 days after returning from a business trip to Hong Kong. Recent lab values demonstrate increased levels of direct serum bilirubin, alkaline phosphatase and WBC’s. Which of the following biliary abnormalities will most likely be identified on the exam?

a. cholangitis
b. primary GB carcinoma
c. Klatskin’s tumor
d. Pneumobilia

A
A

The fever and increased WBC’s indicate an infection. Cholangitis is common in Asian countries.

53
Q
When evaluating intratesticular flow, which vessel is sampled by doppler?

a. cremasteric
b. gonadal
c. deferential
d. centripetal

A
D
Centripetal arteries course through the parenchyma to deliver blood to the testicular tissues. These arteries sampled with doppler in a standard US evaluation.

54
Q
A patient presents for a post-biopsy scan of the right kidney. A lymphocele is suspected. What will be the appearance of this abnormality on the image?

a. fluid collection with debris and septations located adjacent to the kidney
b. cystic formation with debris levels found within the renal sinus
c. anechoic free fluid beneath the renal capsule that distorts the contour of the parenchyma
d. heterogeneous mass within the renal parenchyma

A
A
A lymphocele is a collection of lymphatic fluid that is usually associated with trauma or surgery. The fluid commonly contains debris and septations. A hematoma will form quickly after the procedure due to the constant blood flow from the heart, while a lymphocele or seroma will usually present after several days, or even weeks, post-biopsy.

55
Q
Which imaging plane can demonstrate the IVC and the aorta on the same image in a normal patient?

a. coronal only
b. transverse only
c. mid sagittal only
d. coronal and transverse

A
D
A transverse view can demonstrate a transverse image of both great vessels on the same image. A coronal view can demonstrate a sagittal view of both great vessels on the same image.

56
Q
Which of the following liver abnormalities can be related to the use of oral contraceptives?

a. hydatid disease and schistosomiasis
b. focal nodular hyperplasia and hepatic adenoma
c. hepatic adenoma and Budd Chiari syndrome
d. Budd Chiari syndrome and hydatid disease

A
C

Both hepatic adenoma and Budd Chiari syndrome have been associated with oral contraceptive use.

57
Q
Which type of transducer would be used evaluate a patient with increased LFT’s and height 5’3” and weight of 275lbs?

a. 3MHz curvilinear phased array
b. 10MHz vector array
c. 5MHz linear
d. 5MHz sector phased array

A
A

Deep abdominal imaging is best performed with a low frequency, curvilinear phased array transducer.

58
Q
A patient presents with an order for a renal US due to increased BUN and creatinine levels in the blood and bilateral decreased renal size seen on CT. What will be the expected findings on the US exam?

a. nephrocalcinosis
b. medical renal disease
c. multicystic kidney disease
d. medullary sponge kidney

A
B
MCKD is a congenital malformation of the kidney with formation of multiple cysts and decreased function. In most cases it causes the unilateral formation of multiple large cysts that consume the parenchyma. Medullary sponge kidney and nephrocalcinosis lead to enlarged kidneys. Medical renal disease leads to decreased renal function and size bilaterally.

59
Q
What system control can reduce the appearance of color ghosting?

a. increasing persistence
b. increasing color threshold
c. decreasing color priority
d. decreasing PRF

A
C
Color priority- adjusts the threshold setting; determines if a pixel is displayed as a shade of gray, black or white, or as a color; lower threshold settings reduce the display of color (lower color priority); higher threshold settings display more color (higher color priority); necessary to minimize color ghosting and improper placement of color on vessel walls.

60
Q
All of the following are true regarding Couinad’s liver segmentation except:

a. Allows for liver resection/surgery without negatively affecting other segments
b. right and left lobes are divided by the branches of the main hepatic vein
c. each of the 8 segments has its own branch of the hepatic vein, portal vein and bile duct
d. divides liver into 8 functional segments

A
B

61
Q
Mucocele of the GB most commonly occurs due to:

a. a stone in the ampulla of Vater
b. hepatitis
c. a stone blocking the cystic duct
d. cirrhosis

A
C

If the GB is the only part of the biliary system that is dilated, the blockage must be isolated to the cystic duct.

62
Q
Which of the following statements is false regarding HCC?

a. multiple tumors form throughout the parenchyma on most cases
b. is is the most common liver malignancy
c. tumor invasion of the portal system can lead to the portal vein thrombosis
d. most cases demonstrate increased levels of serum alpha-fetaprotein

A
B

Liver metastasis is the most common liver malignancy.

63
Q
Which of the following describes the sonographic appearance of the pancreas with chronic cystic fibrosis?

a. Increased size with mottled echogenicity, no definite nodules
b. Decreased size and increased overall echogenicity
c. Increased size with nodule calcification formation
d. Decreased size with hypoechoic nodularity throughout the gland

A
B
As the pancreas begins to produce the mucous substance, the gland becomes fibrotic. These changes lead to an increase in echogenicity and decrease in size.

64
Q
If there is no diastolic flow demonstrated on a Doppler tracing of the arcuate artery of the upper pole of the right kidney, what will the resistive index be?

a. 0
b. 1.0
c. 0.5
d. Unable to calculate RI, not enough information given

A
B

The RI is calculated using the peak and end diastolic velocities measured on a Doppler tracing. PSV-EDV/PSV.

65
Q
Which of the following ligaments separates the medial and lateral left lobes of the liver?

a. Ligamentum teres
b. Coronary ligament
c. Ligamentum venosum
d. Broad ligament

A
A
The left intersegmental fissure divides the left lobe into medial and lateral lobes. The falciform ligament, ligamentum teres and left hepatic vein course within this fissure.

66
Q
Which of the following would normally present as a hypoechoic renal mass?

a. Acute pyelonephritis
b. Angiomyolipoma
c. Mycetoma
d. Oncocytoma

A
D
Oncocytoma would normally present as a hypoechoic renal mass, acute pyelonephritis does not cause mass formation but will present as loss of corticomedullary definition. The other choices will appear as a hyperechoic mass.

67
Q
The loops of Henle are located in which portion of the kidney?

a. Pyramids
b. Medulla
c. Cortex
d. Sinus

A
B
The nephrons are located within the parenchyma. Bowmann’s capsule and the vascular supply of the nephron are located in the cortex layer. The nephron tubules to and from the capsule dive between the pyramids within the medulla area of the parenchyma to connect to the loop of Henle.

68
Q
Which of the following is the most commonly occurring germ call tumor of the testicle?

a. Seminoma
b. Choriocarcinoma
c. Embryonal cell tumor
d. Teratoma

A
A

Seminomas make up to 40-50% of all germ cell tumors identified in the testicle.

69
Q
What causes hepatic flow to have a triphasic waveform?

a. Hepatic congestion
b. Congestive heart failure
c. Portal HTN
d. Right atrial contraction and relaxation

A
D

The flow in the hepatic veins is normally triphasic due to right atrial contraction and relaxation.

70
Q
The IVC and SVC empty in the __.

a. Left ventricle
b. Left atrium
c. Right ventricle
d. Right atrium

A
D

The IVC and SVC empty their blood into the right atrium

71
Q
The most frequently involved organ in metastatic disease is the:

a. Pancreas
b. Spleen
c. GI tract
d. Liver

A
D
The liver filters blood from many abdominal organs, such as the stomach and the colon. This function of the liver makes if more susceptible to metastasis.

72
Q
What type of pancreatic cells compose an adenoma?

a. Alpha only
b. Beta only
c. Islets of Langherhans and alpha
d. Alpha and beta

A
B

A pancreatic adenoma is composed of beta cells.

73
Q
An inflammation of the biliary tree common in HIV pts is called:

a. Mirizzi’s syndrome
b. Pneumobilia
c. Cholangitis
d. Klatskin tumor

A
C

The suffix -itis refers to an infection. A, B and D refer to tumor formation.

74
Q
Which of the following statements is false regarding pancreatic divisum?

a. Sonographically demonstrates a classic V-shaped organ with increased vascularity
b. Abnormal drainage of the pancreatic enzymes
c. Most common congenital pancreatic anomaly
d. Commonly associated with pancreatitis

A
A

Pancreatic divisum is difficult to detect sonographically.

75
Q
Sonographically __ appears to connect the GB neck and the junction of the right and left portal veins.

a. Ligamentum venosum
b. Main lobar fissure
c. Intersegmental fissure
d. Ligamentum teres

A
B
The main lobar fissure courses through the right lobe between the neck of the GB and the IVC fossa. Sonographically, the main lobar fissure appears to connect the GB neck and junction of the right and left portal veins. Ligamentum teres courses through the left lobe and contains the collapsed umbilical vein. Ligamentum venosum separates the left lobe from the caudate lobe. The right and left segmental fissures separates the left and right lobes in 2 portions.

76
Q
The IMV:

a. Courses from the mesentery to join the SMV near the porta hepatis
b. Drains blood from the distal colon
c. Drains blood from the proximal colon
d. All of the above

A
B
The IMV drains blood from the distal colon. The SMV drains the blood from the proximal colon and some small bowel portions. The IMV meets the splenic vein near the portal confluence.

77
Q
All of the following are correct regarding acute pyelonephritis, except:

a. Critical finding that requires immediate nephrectomy to prevent sepsis
b. Most commonly caused by an ascending UTI
c. Focal involvement will demonstrate a hypoechoic wedge shaped segment of parenchyma
d. Diffuse involvement will demonstrate loss of distinction of the renal sinus in affected kidney/segment

A
A

Acute pyelonephritis is most commonly diagnosed clinically with lab testing and is treated with antibiotics.

78
Q
Which of the following retroperitoneal spaces contains the adrenal gland?

a. Posterior pararenal space
b. Retrofascial space
c. Anterior pararenal space
d. Perirenal space
e. Glisson’s capsule

A
D
Perirenal space holds kidneys, perinephric fat, prox ureter, adrenal glands. Renal infection can lead to fluid, abscess or gas within the space which will distort renal fascia.

79
Q
Which of the following is least likely to lead to extrinsic compression of the IVC?

a. Congestive heart failure
b. 8cm mass in the right lobe of the liver
c. 6cm AAA at the level of the renal arteries
d. Budd Chiari syndrome

A
A
CHF indicates poor cardiac function. There will be a back log if blood tries to enter the heart from the IVC causing it to dilate. Bilateral pedal edema is a common sign of CHF.

80
Q
Which of the following correctly lists the structures found in the portal triad?

a. Common hepatic artery, hepatic vein, portal vein
b. Portal vein. Proper hepatic artery and bile duct
c. Hepatic vein, proper hepatic artery and bile duct
d. Portal vein, hepatic vein and bile duct

A
B

The portal triad contains the portal vein, proper hepatic artery and CBD(/CHD?)

81
Q
A pt presents with a history of choledocholithiasis. How will you assess the common bile duct for the presence of a stone?

a. Demonstrate a longitudinal view of the main portal vein and look for the CBD anterior to it
b. Locate the main portal vein and look for the CBD posterior to it
c. Demonstrate a long axis GB image and look for the CBD superior to it
d. Demonstrate a transverse view of the pancreatic head and look for the CBD entering the anterior aspect

A
A

The CBD usually courses anterior to the main portal vein when it exits the liver.

82
Q
The diameter of the normal aorta is greatest at the level of the:

a. Renal arteries
b. Iliac bifurcation
c. SMA
d. Celiac axis

A
D
The aorta tapers in size as it courses inferiorly through the abdomen. The celiac axis is the most superior branch of the aorta therefore the aorta diameter is greatest at the level of its origin.

83
Q
The most common primary malignancy to metastasize to the spleen is ____________.

a. Lung cancer
b. Melanoma
c. Renal cancer
d. Breast cancer

A
B

The most common primary malignancy to metastasize to the spleen is melanoma.

84
Q
All of the following are usually benign liver neoplasms, except:

a. Focal nodular hyperplasia
b. Hemangioma
c. Hepatoma
d. Adenoma

A
C
Focal nodular hyperplasia is a benign liver tumor. An adenoma is a benign liver tumor commonly associated with oral contraceptives. A hemangioma is an abnormal accumulation of vascular tissues that forms as a benign liver mass. A hepatoma is a primary malignant tumor of the liver.

85
Q
A thyroglossal duct cyst is most commonly found:

a. On the midline neck, superior to the thyroid gland
b. Posterior to the ear at the level of the thyroid gland
c. Near the angle of the mandible
d. Within the isthmus of the thyroid gland

A
A
A thyroglossal duct cyst is identified on the midline neck, superior to the thyroid gland. A brachial cleft cyst is located near the angle of the mandible.

86
Q
A pt presents for an abdominal US to evaluate a potential cystadenocarcinoma of the pancreatic head that was identified on a recent CT scan. The US demonstrates a complex mass in the pancreatic head. Also, demonstrated is a 12cm GB with no stones or wall thickening and the CBD is patent measuring 10mm. the pt has a negative murphy’s sign. Which term below can be used to describe the findings?

a. Secondary biliary metastasis
b. Courvoisier GB
c. Mirizzi syndrome
d. Caroli’s disease

A
B

Courvesier GB refers to an enlarged, non-tender GB caused by a distal obstruction.

87
Q
The renal arteries branch from the aorta just below the origin of the __.

a. IMA
b. Gonadal arteries
c. SMA
d. Celiac axis

A
C

The renal arteries branch just below the SMA.

88
Q
Autoregulation of the systemic blood pressure refers to:

a. The release of epinephrine to cause constriction of the convoluted tubules
b. The renin-angiotensin system that controls the level of constriction of the convoluted tubules
c. The release of aldosterone to cause vasoconstriction of the efferent arterioles
d. The renin-angiotensin system that controls the glomerular filtration rate of the kidneys and systemic BP level

A
D
When the BP drops, the GFR drops. The kidneys release renin to prompt the release of angiotensin which constricts flow in the arterioles. This increases BP and GFR.

89
Q
All of the following are congenital anomalies of the spleen, except:

a. Splenunculi
b. Agenesis
c. Splenomegaly
d. Ectopic spleen

A
C
Splenomegaly is an acquired condition. Common causes include infection and portal HTN. Agenesis is a congenital absence of the spleen. An ectopic spleen forms in a different location than in the LUQ adjacent to the left kidney. A splecunculi is an accessory spleen or mass of normal splenic tissue not attached to the main gland.

90
Q
All the following lab values will be increased with cirrhosis, except:

a. AST
b. ALT
c. Alkaline phosphatase
d. bilirubin
e. alpha fetoprotein

A
E
ALT, AST, bilirubin and alkaline phosphatase levels will all increase with cirrhosis. Alpha fetoprotein levels increase with malignancy and pregnancy. If HCC develops with cirrhosis, then AFP levels will rise.

91
Q
What term describes a GB that is filled with bile that is isoechoic to the liver and is difficult to identify on US?

a. Mirizzi syndrome
b. Charcot’s triad
c. Courvoisier GB
d. Hepatization

A
D
Hepatization of the GB refers to a GB filled with sludge that is isoechoic to the liver. This makes the GB difficult to identify on US. Try to identify the GB wall in these cases.

92
Q
A cyst within a cyst (daughter cyst) is a classic sign of:

a. Polycystic liver disease
b. Hydatid disease
c. Echinococcal cyst or pyogenic abscess
d. Hydatid cyst or Echinicoccal cyst

A
D
Echinococcal cysts are also called hydatid cysts and demonstrate a cyst with another cyst inside. They form due to parasitic infection of the liver.

93
Q
Which of the following vessels does not drain into the IVC?

a. Right gonadal vein
b. Right renal vein
c. Left hepatic vein
d. Splenic vein
e. Internal iliac vein

A
D

The splenic vein is a part of the portal venous system. It joins with the SMV at the portal confluence.

94
Q
Which of the following is false regarding pancreatic masses?

a. The location of the mass in the head of the pancreas usually indicates a benign process
b. The location of the mass in the head of the pancreas usually indicates a malignant process
c. A pancreatic tail mass can be confused with splenic hilar pathology
d. A mass with round smooth borders is usually a benign process

A
A
Most tumors with round smooth borders are benign. Most pancreatic tumors located in the head are malignant. Masses found in the tail are usually benign. Because of the proximity of the pancreatic tail to the splenic hilum, tail masses can look like splenic pathology.

95
Q
The __ is formed at the junction of the vas deferens and the seminal vessicals.

a. Rete testes
b. Ejaculatory duct
c. Afferent ducts
d. Efferent ducts

A
C

The ejaculatory duct is formed at the junction of the vas deferens and seminal vesicles.

96
Q
During an abdominal US, a small circular anechoic structure is identified at the anterior portion of the pancreatic head. Color flow is identified in the structure. What is it?

a. Celiac axis
b. CBD
c. GDA
d. IMA

A
C

The GDA is demonstrated as a small circular anechoic structure at the anterior portion of the pancreas head.

97
Q
All of the following correctly describes liver adenomas, except:

a. Encapsulated mass that is more common in women
b. Surgical intervention is usually recommended because of risk of internal hemorrhage
c. Easily distinguished from a focal nodular hyperplasia due to hypervascularity
d. Associated with glycogen storage disease

A
C
FNH and liver adenomas have a similar sonographic appearance and biopsy is usually required to determine the mass origin.

98
Q
A fluid collection between the diaphragm and splenic capsule may represent:

a. Subphrenic abscess
b. Subcapsular hematoma
c. Pericardial effusion
d. Pleural effusion

A
A
A pleural effusion will be located above the diaphragm. A subcapsular hematoma will be located between the splenic tissue and the splenic capsule. A pericardial effusion refers to fluid in the pericardial sac around the heart. Sub- means under and -phrenic means diaphragm. The subphrenic abscess will be located under the diaphragm adjacent to the splenic capsule.

99
Q
The _ is the portion of the prostate that is usually composed of smooth muscle cells and make up approximately 1/3 of the gland.

a. Peripheral zone
b. Central zone
c. Transitional zone
d. Anterior fibromuscular zone

A
D

The anterior fibromuscular zone is composed of mostly smooth muscle and occupies 1/3 of the gland.

100
Q
The spleen produces _ and their main function is to control antibodies and prevent infection.

a. Amylase
b. Aldosterone
c. Mitochondria
d. Platelets
e. Leukocytes

A
E

Leukocytes are WBCs. They are produced in the splenic tissue.

101
Q
Where is the most common location for the occurrence of retroperitoneal fibrosis?

a. Adjacent to the diaphragm surrounding aorta
b. At the lateral borders of the kidneys
c. At the level of the renal arteries
d. At the level of the aortic bifurcation

A
D
Retroperitoneal fibrosis usually occurs at the level of the aortic bifurcation. It refers to idiopathic overgrowth of fibrous tissue. It can be related to drugs, infection, malignancy or cancer therapy. The overgrowth of tissue may lead to ureteral obstruction, causing hydronephrosis.

102
Q
__ is the most common malignant neoplasm of the prostate.

a. Metastasis
b. Central zone sarcoma
c. Transitional zone sarcoma
d. Adenocarcinoma

A
D

Adenocarcinoma is the most common malignant neoplasm found in the prostate.

103
Q
The __ are called intersegmental vessels of the liver.

a. Portal veins
b. Hepatic arteries
c. Hepatic veins
d. Hepatic arteries and veins

A
C
The hepatic veins course between the liver segments (intersegmental). The portal veins and hepatic arteries course into the mid portion of each segment and branch toward the edge (intrasegmental).

104
Q
Acute cholecystitis is associated with wall thickening _.

a. >3mm
b. >2mm
c. >6mm
d. <3mm

A
A

Acute cholecystitis is associated with GB wall thickening >3mm.

105
Q
The renal pyramids are found in the:

a. Medulla
b. Calyces
c. Sinus
d. Cortex

A
A
The renal pyramids are wedge-shaped spaces located in the medulla portion of the renal parenchyma. The base of the triangle is adjacent to the cortex of the parenchyma. The urine is funneled to the papilla, or tip of the wedge, that is connected to the calyce of the renal sinus.

106
Q
All of the following structures are directly medial and anterior to the splenic hilum, except:

a. Fundus of the stomach
b. Morrison’s pouch
c. Lesser sac
d. Pancreatic tail

A
B
Morrison’s pouch is located between the liver and the right kidney. The pancreatic tail, fundus of the stomach and lesser sac are anterior/medial to the spleen

107
Q
Calcitonin is produced by which of the following types of cells?

a. Parathyroid parafollicular
b. Parathyroid follicular
c. Thyroid parafollicular
d. Thyroid follicular

A
C
Calcitonin is produced by in the thyroid in the parafollicular cells. These cells are located in the tissues between the thyroid follicles.

108
Q
All of the following correctly describe papillary thyroid cancer, except:

a. Is highly curable
b. 5% of all thyroid cancers
c. Associated with enlarged cervical lymph nodes
d. Most commonly presents as a hypoechoic mass with calcifications

A
B
Papillary carcinoma is the most common type of thyroid cancer. It is commonly associated with enlarged cervical lymph nodes but is highly curable. Sonographic characteristics include a hypoechoic mass with calcifications.

109
Q
Which of the following correctly describes the pediatric pancreas?

a. It normally increases in echogenicity as the pt reaches adulthood
b. The ducts are dilated at >2mm at birth and decrease in size throughout the first year
c. It is normally isoechoic to the liver
d. Cystic fibrosis leads to the production of mucus and the gland is reduced in echogenicity

A
C
The pancreas is isoechoic as a child and increases in echogenicity as the pt reaches adulthood. Cystic fibrosis causes an increase in echogenicity of the gland.

110
Q
Which of the following is not a congenital anomaly/disorder?

a. Biliary atresia
b. Choledochal cyst
c. Sclerosing cholangitis
d. Caroli’s disease

A
C

Sclerosing cholangitis is an acquired inflammatory disorder of the biliary tree

111
Q
In most pts, the proper hepatic artery bifurcates into the right and left hepatic arteries. In about 10% of pts, the right hepatic artery originates from the and the left hepatic artery originates from the .

a. SMA, left gastric
b. Splenic artery, common hepatic
c. Left renal, right renal
d. Celiac, coronary

A
A
In most pts the proper hepatic artery bifurcates into the right and left hepatic arteries. In about 10% of pts the right hepatic artery originates from the SMA and the left originates from the left gastric artery.

112
Q
Which of the following statements are false regarding liver hemangiomas?

a. This type of liver mass is commonly identified as an incidental finding
b. The mass is composed of a high concentration of vascular tissues
c. This type of mass is rarely malignant
d. This type of liver mass is usually hyperechoic to normal liver tissue
e. Increasing diffuse fatty replacement will cause the mass to increase in echogenicity

A
E
Increasing diffuse fatty replacement will cause the liver tissues to increase in echogenicity and the mass will become more isoechoic or even hypoechoic to the liver tissues

113
Q
Which of the following scrotal structures carries the seminal fluid from the rete testis to the epididymis?

a. Efferent ducts
b. Mediastinum
c. Vas deferens
d. Seminiferous tubules

A
A

The seminiferous tubules empty into the efferent ductules that connect to the epi.

114
Q
What fetal syndrome is associated with hepatoblastoma?

a. Down syndrome
b. Beckwith-Weidemann syndrome
c. Trisomy
d. Budd Chiari syndrome

A
B

Beckwith-Weidemann syndrome is associated with hepatoblastoma

115
Q
Sonographic signs of chronic renal failure include a/an _ in size and a/an in echogenicity.

a. Decrease, increase
b. Decrease, decrease
c. Increase, increase
d. Increase, decrease

A
A
As kidney function decreases the organ atrophies. The cortex thins and becomes increasingly dense leading to an increase in cortical echogenicity

116
Q
Which of the following Doppler criteria is used to evaluate the parenchymal flow in a renal allograft?

a. Resistive index
b. SD ratio
c. Peak systolic velocity
d. Renal aorta ratio

A
A
The resistive index should normally be below 0.7 in a renal allograft. If the RI reaches 0.8-1.0 transplant failure is suspected.

117
Q
All of the following are produced by the exocrine function of the pancreas, except:

a. Trypsin
b. Lipase
c. Insulin
d. Amylase

A
C

Insulin is a hormone produced by the pancreas (endocrine).

118
Q
A histology report indicates that a testicular tumor contains only seminomatous cells. Which of the following best describes the tumor?

a. Mature teratoma
b. Mixed germ cell tumor
c. Metastatic tumor
d. Seminomatous tumor

A
D

Seminomatous tumors are comprised of seminomatous cells.

119
Q
Corpora amylacea is a benign condition of what organ?

a. Prostate
b. Liver
c. Spleen
d. Kidneys

A
A

Corpora amylacea refers to calcifications commonly seen in the inner gland.

120
Q
A pt presents with splenomegaly and multiple, small, tortuous vessels are visualized in the porta hepatis area of the liver. These vessels most likely represent:

a. Cavernous transformation
b. Dilated extrahepatic biliary system
c. Duplicated hepatic artery
d. Portal aneurysm

A
A
Cavernous transformation of the vessels at the porta hepatis refers to the formation of varices from the increased portal pressure with portal HTN. Accessory veins dilate at the porta hepatis due to the backlog of blood flow. These vessels are small and tortuous. Splenomegaly is another sign of portal HTN and a backlog of blood flow in the porta hepatis.

121
Q
Regarding the aorta, fusiform aneurysm formation usually occurs in the while dissecting aneurysms usually occur in the _.

a. Aortic arch, distal abdominal aorta
b. Prox abdominal aorta, thoracic aorta
c. Distal abdominal aorta, aortic arch
d. Distal abdominal aorta, thoracic aortic

A
C
Fusiform aneurysms usually occur in the distal abdominal aorta while dissecting aneurysms usually occur in the aortic arch.

122
Q
Hutch diverticula are identified in the:

a. GB
b. Large bowel
c. Urinary bladder
d. Small bowel

A
C

Hutch diverticula are congenital outpouchings of the bladder wall near the urethral opening.

123
Q
Zenker’s diverticulum can be mistaken for:

a. A thyroid mass
b. A bowel obstruction
c. Dilated CBD
d. The appendix

A
A
Esophageal diverticulum refers to the outpouching of a weak spot in the esophageal wall.
Killian-Jameison diverticulum- protrudes from anterolateral wall of the cervical esophagus.
Zenker’s diverticulum- located at the back of the throat, protrudes posterior or midline.
They can be mistaken for a thyroid mass.

124
Q
Which of the following is least likely to demonstrate an increased RI in the arcuate arteries?

a. Acute renal artery occlusion
b. Renal vein thrombosis
c. Acute renal failure
d. Acute tubular necrosis

A
A
No intrarenal arterial flow will be detected in the kidney with an acute renal arterial occlusion. Acute tubular necrosis, acute renal failure and renal vein thrombosis reflect the kidneys that cannot properly filter blood and cortical tissue resistance is increased.

125
Q
A tardus parvus waveform is described as:

a. Pronounced early systolic pressure and prolonged acceleration time
b. Reduced systolic velocity with diastolic flow reversal
c. Absence of early systolic peak and prolonged acceleration time
d. RI of 1.0 or greater

A
C
Tardus parvus waveform- absence of early systolic peak and prolonged acceleration time. If identified in the native arcuate arteries, significant stenosis is present.

126
Q
All of the following are benign splenic neoplasms, except:

a. Hamartoma
b. Granuloma
c. Hemangioma
d. Lymphoma

A
D
A splenic hamartoma is a rare, benign vascular proliferation that is often found incidentally. A hemangioma is another type of benign vascular malformation. A granuloma forms after infection. Lymphoma is a primary malignancy of the spleen related to the lymphatic tissues.

127
Q
What biliary anomaly is suspected if jaundice persists more than 14 days after birth?

a. Agenesis of the GB
b. Choledochal cyst
c. Biliary atresia
d. Ectopic GB

A
C

The lack of opening of the biliary tree will cause persistent jaundice after birth.

128
Q
What is the most common bilateral malignancy of the testicle?

a. Seminoma
b. Leukemia
c. Metastasis
d. Lymphoma

A
D

The most common BILATERAL malignancy of the testes is lymphoma.

129
Q
Primary testicular carcinoma is usually _ and secondary testicular malignancy is usually _.

a. Heterogeneous, isoechoic
b. Hypoechoic, isoechoic
c. Bilateral, unilateral
d. Unilateral, bilateral
e. Hypervascular, hypovascular

A
D

Unilateral, bilateral

130
Q
Which of the following describes carpal tunnel syndrome?

a. Compression of the median nerve by the carpal sheath causing numbness, weakness and pain
b. Compression of the blood vessels as they enter/exit the wrist causing thrombus formation and emboli
c. Damage to the median nerve by chronic compression between the metacarpal bones of the palm
d. Damage to the cartilage between the carpal bones of the wrist causing swelling and pain with motion

A
A
Carpal tunnel syndrome refers to compression of the median nerve by the carpal sheath causing numbness, weakness and pain in the wrist/hand.

131
Q
Which of the following biliary abnormalities is associated with chronic cholecystitis?

a. Empyema
b. Porcelain GB
c. Strawberry GB
d. Biloma

A
B

Chronic cholecystitis can lead to calcium formation within the wall of the GB.

132
Q
abscesses are most commonly found in countries outside the U.S. abscesses are most commonly seen in immunosuppressed pts.

a. Fungal, amebic
b. Amebic, fungal
c. Fungal, pyogenic
d. Pyogenic, amebic

A
B

133
Q
A pt presents for an abdominal US with a history of increased LFT’s and obesity. The referring doctor would like to rule out hepatomegaly. Which of the following statements regarding hepatomegaly is true?

a. When the right lobe is rounded inferiorly and extends past the upper pole of the right kidney
b. Rounding of the superior liver segments
c. AP measurement exceeds 15.5cm
d. When the right lobe is rounded inferiorly and extends below the lower pole of the right kidney

A
D
Hepatomegaly is diagnosed when the length of the liver exceeds 15.5cm. the AP measurement will also increase but the length is most accurate.

134
Q
Which of the following pts would benefit from a FAST exam?

a. Pt with abnormal bruit, weight loss and post prandial pain
b. Pt with suspected prostate cancer
c. Pt with a history of abdominal pain after falling from a ladder yesterday
d. Pt with a history of RUQ pain after eating

A
C
FAST evaluation- Focused Assessment with Sonography in Trauma; used to evaluate the abdomen for fluid or blood in trauma pts; check the 4 quadrants, midline and Morrison’s pouch

135
Q
How many layers are identified in the normal gut?

a. 5
b. 3
c. 8
d. 2
e. 6

A
A

Serosa, muscularis, submucosa, deep mucosa, superficial mucosa/lumen.

136
Q
Increased thyroid volume is associated with all of the following, except:

a. Iodine deficiency
b. Chronic hepatitis
c. Increasing age
d. Chronic renal failure

A
B
Thyroid volume increases normally with age and body weight. Iodine deficiency, acute hepatitis and chronic renal failure can also cause increased gland volume. Chronic hepatitis, thyroxine treatment and radioactive iodine treatment can cause decreased gland volume.

137
Q
Which of the following is least likely to be used to treat a liver mass formed due to HCC?

a. Acetic acid injection
b. Thrombin injection
c. Radiofrequency ablation
d. Ethanol injection

A
B
Percutaneous ethanol injection is used to cause tumor necrosis in thyroid adenomas and cysts, parathyroid adenomas, HCC and liver metastasis. Acetic acid is another type of injectable used for liver tumor treatment. Radiofrequency ablation of liver tumors induces thermal injury to the target tissues. Thrombin is used in the treatment of pseudoaneurysms.

138
Q
A FNA biopsy of a thyroid nodule is not recommended for nodules __.

a. <20mm b. >50mm
c. <1mm
d. <10mm

A
D

139
Q
Which of the following is incorrect in reference to sterile technique?

a. Sterile fields or packages are opened as close as possible to time of actual use
b. Personnel with colds should avoid working while ill or apply a double mask
c. Gowns are considered sterile only in the front
d. The only person who must be sterile within the sterile field is the physician performing the procedure

A
D
Sterile field:
All items and personnel in a sterile field must be sterile
Sterile packages or fields are opened or created as close as possible to the time of actual use.
Moist areas are not considered sterile.
Contaminated items must be removed immediately from the sterile field.
Gowns are considered sterile only in the front, from chest to waste and from the hands to slightly above the elbow.
Personnel with colds should avoid working while ill or apply a double mask.
Edges of sterile areas are not considered sterile.

140
Q
Which of the following describes the best pt position for a thoracentesis?

a. Left lateral decubitus
b. Seated leaning forward over a table
c. Semi-erect with the knees raised toward the chest
d. Right lateral decubitus

A
B
Thoracentesis is performed for removal of fluid from the pleural space in the thoracic cavity. It is performed for diagnostic or therapeutic purposes. A chest x-ray is performed before and after the procedure to evaluate fluid removal and for the presence of a potential pneumothorax caused by the procedure. Most pts are placed in the semi-erect position and leaning forward over a table.

141
Q
Which lab value is associated with jaundice?

a. Bilirubin
b. WBC
c. PT
d. INR

A
A

Jaundice is caused by an increase in bilirubin in the blood. This can be due to ductal obstruction or liver malfunction.

142
Q
Which of the following will help the radiologist select the correct biopsy needle size/length for the biopsy of a mass in the posterior right liver?

a. Measuring the depth of the mass within the liver
b. Measuring the width of the mass
c. Using color Doppler to locate the surrounding vascularity
d. Usually the sonographer opens multiple sizes of core biopsy needle and places them on the sterile procedure tray in case they are needed

A
A
The depth of the mass can affect the length of needle required for the procedure. Normally only one biopsy device is opened and used for the procedure.

143
Q
A pt presents for a liver US with contrast to evaluate a tumor. Which of the following lists the correct machine settings needed for the exam?

a. Low mechanical index, harmonics on, focal zone placed at the area of interest
b. Mechanical index over 1.5, harmonics off, focal zone at the chest wall
c. Highest output power settings available, harmonics on, at least 3 focal zones surrounding the area of interest
d. Low mechanical index, harmonics off, focal zone at the chest wall

A
A
The mechanical index should be low, 1.0 or less, to decrease cavitation and decrease the potential to burst the microbubbles. Harmonics will assist in visualizing the contrast in the microvascularity. The focal zone should always be placed at or just below the area of interest.

144
Q
Which type of transducer would be used to evaluate a palpable lump in the neck?

a. 3MHz curvilinear sequential array
b. 12MHz sector phased array
c. 12MHz linear phased array
d. 12MHz curvilinear phased array

A
C
Superficial imaging is best performed with linear transducers. The large near field and rectangular FOV provide improved imaging of superficial structures. Higher frequencies also provide the best resolution.

145
Q
All the following are pt positioning techniques that can assist in viewing the GB, except:

a. Prone
b. Supine
c. LLD
d. Erect

A
A
Due to the anatomic position of the GB anf the limited imaging windows from the pt’s back, the prone position will not provide additional imaging views of the GB.

146
Q
When the prostate is enlarged, what other structures should be evaluated for related findings?

a. Renal arteries for thrombosis
b. Kidneys for hydronephrosis
c. Portal vein for thrombosis
d. Renal veins for nutcracker syndrome

A
B

An enlarged prostate can cause urine retention in the bladder and associated hydronephrosis.

147
Q
Overgaining on an US exam can cause _.

a. Overestimation of Doppler velocity
b. Blood to appear anechoic
c. Increased posterior shadowing
d. Underestimation of arterial stenosis

A
A

148
Q
Which of the following machine adjustments will produce a more black and white 2D image?

a. Decreased overall gain
b. Increased dynamic range
c. Decreased monitor contrast
d. Increased log compression

A
D
As log compression increases, the dynamic range decreases. The lower the dynamic range, the smaller number of shades of gray that are displayed on the image. Overall gain would adjust the brightness of all shades on the screen and will not affect contrast.

149
Q
What type of precautions are required for a pt with MRSA?

a. Reverse protective isolation
b. Contact precautions
c. Enteric precautions
d. Strict isolation

A
B
Wound-skin precautions- AKA contact precautions; used to protect others from germs in pt’s wounds or any other heavily contaminated areas; require gown and gloves for any type of pt contact; MRSA, scabies, VRE, Impetigo.

150
Q
What is the twinkle artifact?

a. Varied brightness of vascular calcifications causes a twinkling effect on 2D
b. Thin line of reverberation artifact that is caused by calcium deposits in the renal parenchyma
c. Mosaic color pattern displayed in a linear pattern posterior to a dense object, such as a renal stone
d. Describes the color display seen as urine is entering the bladder during real-time imaging

A
C

151
Q
Which image plane can produce a longitudinal image of the IVC and the aorta on the same image?

a. Coronal only
b. Coronal and transverse
c. Mid sagittal only
d. Transverse only

A
A

152
Q
A pt presents for a post biopsy scan of the right kidney. An extracapsular hematoma is suspected. What will be the appearance of this abnormality on the image?

a. Free fluid with debris levels found within Morrison’s pouch
b. Free fluid beneath the renal capsule that distorts the contour of the parenchyma
c. Free fluid with debris levels found within Murphy’s pouch
d. Cystic formation with debris levels found within the renal sinus

A
A
Blood that collects between the renal capsule and the cortex is an intracapsular hemorrhage. Blood that collects outside the capsule. Between the capsule and the liver tissue is an extracapsular hemorrhage. An intracapsular hemorrhage will distort the renal cortex while an extra capsular hemorrhage will not.

153
Q
A pt presents after a recent MVA. He complains of several new palpable masses on the anterior abdomen. Sonographically they appear as rounded complex masses with some through transmission. These findings most likely represent:

a. Rectus sheath hematoma
b. Peritoneal fibrosis
c. Lipomas
d. Sebaceous cysts in the rectus sheath abdominus muscles

A
A
The recent history of an MVA and acute presentation of the complex masses should lead to a diagnosis of a hematoma. Fibrosis would appear as dense formations within the muscular tissues. A sebaceous cyst would present in a much more superficial location as they are related to the oil glands of the skin. Lipomas are composed of fatty tissue and usually present as a round or oval homogeneous, hyperechoic mass related to the surrounding muscle tissue.

154
Q
Which of the following is NOT accurate when describing standard precautions?

a. Gowns are worn if soiling of clothing is likely from blood or body fluid
b. When using cloth sheets, each one can be used twice as long as it is flipped over to the clean side after the first pt
c. Hands should be washed after removing gloves
d. Standard precautions should apply to all body fluids from all pts

A
B

155
Q
The _ region is defined by the orifices of the two ureters and the urethral outlet.

a. Trigone
b. Fundus
c. Dome
d. Base

A
A

156
Q
The right lobe of the liver is divided into _ segments, while the left lobe is divided into __ segments.

a. Anterior and posterior, medial and lateral
b. Caudate and anterior, quadrate and posterior
c. Medial and lateral, anterior and posterior
d. Caudate and posterior, quadrate and anterior

A
A

157
Q
Which of the following probe frequencies should be used when trying to evaluate the pancreas in an obese pt?

a. 3MHz linear
b. 5MHz phased
c. 3MHz curvilinear
d. 5MHz curvilinear

A
C

158
Q
A 6yr old presents with a history of sickle cell anemia. What do you expect to find on the US evaluation of the spleen?

a. Splenomegaly
b. Splenic atrophy
c. Agenesis of the spleen
d. Splenic artery aneurysm

A
A

Sickle cell anemia causes the spleen to enlarge early in life and then atrophy later.

159
Q
If the entire image is too bright, what console adjustment should you make first?

a. Adjust far field TGC
b. Adjust near field TGC
c. Decrease output power
d. Decrease the overall gain

Study These Flashcards
A
C

If the image is too bright, always decrease the output power first to decrease pt exposure. Remember, ALARA.

160
Q
Which of the following organs should be evaluated when looking for a tumor of the endocrine system?

a. Pancreas and kidney
b. Adrenal glands and spleen
c. Salivary glands and adrenal glands
d. Liver, spleen and pancreas
e. Pancreas and adrenal glands

Study These Flashcards
A
E
The endocrine system includes the pancreas, pituitary gland, hypothalamus gland, pineal gland, thyroid, parathyroid, adrenal glands, ovaries and testes.

161
Q
What type of precautions is required for a pt that had a renal transplant yesterday?

a. Reverse protective isolation
b. Enteric precautions
c. Strict isolation
d. Contact precautions

Study These Flashcards
A
A
Reverse protective isolation- AKA protective environment precautions; protects pts from the germs of others; it is used with pts who have extremely impaired immune systems or recent organ transplant.

162
Q
All of the following can cause artifactual debris within a simple cyst, except:

a. Increased rejection settings
b. Improper gain settings
c. Superficial location of the cyst
d. Small size of the cyst

Study These Flashcards
A
A
Increased gain settings can cause fill in anechoic structures. Small cysts may demonstrate artifactual echoes from surrounding tissues. A very superficial cyst can demonstrate range resolution artifact in the anterior cyst. Increased attenuation of the sound beam would actually help to remove artifactual echoes because it will lead to a decreased amount of reflections that reach the probe.

163
Q
Leukocytosis means:

a. Increased WBC’s
b. Increased platelets
c. Increased RBC count
d. Decreased WBC count

Study These Flashcards
A
A

164
Q
What type of precautions is required for a pt with the mumps?

a. Droplet precautions
b. Strict isolation
c. Contact precautions
d. Reverse protective isolation

Study These Flashcards
A
A
Respiratory isolation- AKA droplets; used to protect others from germs
In the pt’s nose, mouth, throat and lungs. It is used for diseases that are airborne.

165
Q
If a pt has a non-reactive Thompson test, what problem do they have?

a. Fungal abscess of the liver
b. Baker’s cyst in the popliteal
c. Complete tear of the Achilles tendon
d. Hydatid cyst of the liver

Study These Flashcards
A
C
Thompson test- pt placed in the prone position and the calf muscle is squeezed. If the foot flexes in response to the squeeze the tendon is not completely torn. If the foot is non-reactive to the squeeze, the tendon is completely torn.

166
Q
Anemia is least likely to be related to an abnormality of which of the following organs?

a. Thyroid
b. Aorta
c. Esophagus
d. Spleen

Study These Flashcards
A
A

If there is a small bleed in the AA or the GI, anemia can be an indicator for these problems.

167
Q
Increasing the size of the field of view will _ the frame rate.

a. Decrease
b. Cause a 50% decrease
c. Have no effect on
d. Increase

Study These Flashcards
A
A
Increasing the area to be scanned will require an increase in time to process the increased # of reflections for display causing a decreased frame rate.

168
Q
A pt presents for an abdominal US due to suspected displacement of the greenfield filter. Where should you look to find the filter in the normal position?

a. In the renal artery at the renal hilum
b. In the IVC below the level of the renal veins
c. In the renal vein at the renal hilum
d. In the IVC just below the level of the hepatic veins

Study These Flashcards
A
B

169
Q
A 40yr old male presents for an abdominal US exam with a history of anxiety, headaches and excessive sweating. His current BP is 175/90mmHg with a pulse of 102bpm. What is the referring doctor expecting to find?

a. Aortic aneurysm
b. Adenoma
c. Pheochromocytoma
d. Budd Chiari syndrome

Study These Flashcards
A
C
Pheochromocytoma cause increased secretion of dopamine, epinephrine and norepinephrine. This leads to symptoms that include HTN, anxiety, headache, nausea, vomiting, sweating and tachycardia. Tumors originate in medulla tissue and grow to average size of 5-6cm.

170
Q
Which imaging plane is most commonly used to obtain the length of the spleen during an abdominal US?

a. Axial
b. Coronal
c. Transverse
d. Sagittal

Study These Flashcards
A
B
A left lateral approach is commonly used to obtain a longitudinal image of the spleen. This coronal view is used to assess the length of the spleen.

171
Q
The fibromuscular stroma of the prostate is called the __ zone.

a. Anterior
b. Central
c. Transitional
d. Peripheral

Study These Flashcards
A
A

172
Q
All the following are clinical signs of an abscess. Except:

a. Increased WBC
b. Fever
c. Pain
d. Anemia

Study These Flashcards
A
D

173
Q
What is the most common primary renal malignancy?

a. RCC
b. TCC
c. Wilm’s tumor
d. Cystadenocarcinoma

Study These Flashcards
A
A

174
Q
While performing a Doppler evaluation of the main renal arteries, multiple pt positions may be required in order to maintain a Doppler angle of _ degrees or less.

a. 0
b. 30
c. 60
d. 90

Study These Flashcards
A
C

175
Q
The splenic artery distributes blood to the _.

a. Stomach
b. Pancreas
c. Spleen
d. All of the above

Study These Flashcards
A
D

176
Q
A pt presents for a renal US with a history of increased serum BUN and creatinine, HTN. Right renal artery occlusion was documented on an US performed last year. What are the expected findings on the US exam today?

a. The right kidney will be consumed by cyst formation with a normal left kidney
b. The right kidney will be atrophied and hyperechoic and the left kidney will be within normal limits for size
c. The right kidney will be atrophied and surrounded by ascites due to decreased renal function. The left kidney will appear normal
d. The right kidney will be enlarged and edematous with decreased corticomedullary definition, the left kidney will appear normal

Study These Flashcards
A
B
When the arterial supply to the kidney is obstructed, the kidney atrophies and does not perform the proper filtration of the blood. This leads to increased serum levels of BUN and creatinine.

177
Q
All the following are characteristics of power Doppler, except:

a. Demonstrates organ perfusion better than color Doppler
b. Demonstrates presence or absence of flow
c. More sensitive than color Doppler
d. Demonstrates velocity of flow within the organ

Study These Flashcards
A
D
Pulsed wave Doppler demonstrates the range of peak flow velocities and flow direction of the blood vessel. Color Doppler demonstrates the average flow velocity of the blood in a vessel and flow direction. Power Doppler usually does not demonstrate flow direction but does demonstrate the presence/absence of flow.

178
Q
A pt presents for an abdominal US to verify findings on a CT exam. The CT report indicates numerous cystic structures throughout the liver that do not communicate with the biliary tree or the vascular system. Which of the following best describes the expected US findings?

a. Choledochal cysts
b. Caroli’s disease
c. Budd Chiari syndrome
d. Polycystic disease

Study These Flashcards
A
D
Cystic structures related to Caroli’s disease and choledochal cysts will communicate with the biliary tree. Cysts that form with PCLD do not.

179
Q
Most pts with pancreatic adenocarcinoma demonstrate:

a. Normal levels of amylase and lipase
b. Increased lipase with normal amylase
c. Variable levels of amylase with increased lipase
d. Increased amylase and lipase

Study These Flashcards
A
A

Most cases of pancreatic adenocarcinoma will demonstrate normal enzyme levels.

180
Q
Which of the following sonographic characteristics is least likely to be identified in a pt with renal vein thrombosis?

a. Increased diastolic flow in the renal arteries
b. Enlarged kidneys with mottled echogenicity
c. Lack of venous flow within the kidney
d. Decreased corticomedullary definition

Study These Flashcards
A
A
Renal vein thrombosis will demonstrate increased kidney size, hypoechoic renal cortex, decreased corticomedullary differentiation, mottled echogenicity, dilated renal vein and no intrarenal venous flow and increased resistance in renal arteries.

181
Q
The _ arteries are the vessels that course between the renal pyramids.

a. Lobular
b. Interlobar
c. Segmental
d. Arcuate

Study These Flashcards
A
B

182
Q
Which of the following is false regarding hydrocele formation?

a. Most hydroceles are congenital
b. Usually contains serous fluid but may see minimal cellular debris
c. Most common cause of painless swelling
d. Accumulation of serous fluid between the two layers of the tunica albuginea

Study These Flashcards
A
D
A hydrocele is the accumulation of serous fluid between the two layers of the tunica vaginalis. They most commonly occur in newborns but are also seen in adults associated with infection/inflammation.

183
Q
All of the following are correct regarding renal angiomyolipomas, except:

a. Benign renal tumor
b. Most commonly presents in the left kidney
c. Large masses can cause propagation speed artifact
d. CT used to confirm diagnosis

Study These Flashcards
A
B

The majority of angiomyolipomas form in the right kidney.

184
Q
A hematoma of the rectus abdominus muscle that occurs below the arcuate line can extend into:

a. Morrison’s pouch
b. Pouch of douglas
c. Space of retzius
d. The umbilicus

Study These Flashcards
A
C
An acute hematoma will be anechoic or mildly hypoechoic to surrounding tissues. The linea alba stops the blood from crossing the midline, above the level of the arcuate line. Blood is not confined within the sheath below the arcuate line which allows the blood to extend into the pelvis.

185
Q
An average sized pt presents with a fever, acute RLQ pain and vomiting. The ER physician wants to rule out appendicitis in this pt. which transducer would best be suited for the exam?

a. 12MHz linear
b. 5MHz curvilinear
c. 3MHz curvilinear
d. 5MHz linear

Study These Flashcards
A
D
In the average sized pt, 5-7MHz linear probe will usually provide the appropriate level of penetration for the exam. The linear array is preferred for its footprint and large near field.

186
Q
A 55yr old male pt presents with a script that states “pelvic US; post biopsy, gleason score of 8”. What are you most likely looking for on the exam?

a. Lymphadenopathy and metastasis related to prostate cancer
b. Inguinal hernia caused by recent bladder wall biopsy
c. Lymphadenopathy and metastasis related to TCC of the bladder
d. Bladder puncture caused by recent biopsy of the seminal vesicle

Study These Flashcards
A
A
Gleason score is used to classify prostate cancer; 1-10 with 10 as the most aggressive. A score of 8 indicates a moderately aggressive prostate tumor. The pelvis should be evaluated for lymphadenopathy and metastasis related to prostate cancer.

187
Q
A right adrenal mass will displace the IVC:

a. Inferiorly
b. Posteriorly
c. Anteriorly
d. Superiorly

Study These Flashcards
A
C

The right adrenal gland is posterior to the IVC. Tumor invasion can displace the IVC more anteriorly.

188
Q
A pt presents with a history of Reidel’s lobe. What are the expected findings on the US exam?

a. The left lobe will extend across the midline to touch the upper pole of the spleen
b. The liver will demonstrate diffuse fatty infiltration of the right lobe only
c. The caudate lobe will be at least 2X greater in size than the left lobe
d. The right lobe of the liver will have a tongue-like extension of the right lobe that extends over the lower pole of the right kidney

Study These Flashcards
A
D

189
Q
In a pt with primary hyperparathyroidism, levels of parathyroid hormone will be increased and levels will be .

a. T3, increased
b. Serum calcium, decreased
c. Serum calcium, increased
d. T4, increased

Study These Flashcards
A
C

190
Q
Which of the following can be treated using the lithotripsy techniques?

a. GB sludge
b. Renal stenosis
c. Staghorn calculus
d. Budd Chiari syndrome
e. Lymphoma

Study These Flashcards
A
C

Lithotripsy is used to shatter kidney stones into “manageable” sized pieces to allow the pts to pass them normally.

191
Q
In a normal adult pt, the common hepatic duct lumen measures less than or equal to __.

a. 6mm
b. 2mm
c. 2cm
d. 4mm

Study These Flashcards
A
D

192
Q
Decreased hematocrit levels are associated with all the following, except:

a. Sickle cell anemia
b. Esophageal varices
c. Cavernous hemangioma of the spleen
d. Hemorrhagic cyst of the liver

Study These Flashcards
A
C
A low hematocrit means the percentage of RBC’s is below the lower limits of normal. Causes include internal bleeding (varices), RBC destruction (anemia) and malnutrition.

193
Q
You are scanning through the liver and note the caudate lobe is nearly the same size as the right lobe. The surface of the liver appears lobulated or nodular. Which of the following is the most likely diagnosis of these findings?

a. HCC
b. Focal nodular hyperplasia
c. Cirrhosis
d. Hepatitis

Study These Flashcards
A
C
The caudate enlarges as the larger right and left lobes shut down due to cirrhosis. A caudate to right lobe ratio >0.65 is said to be related to cirrhotic atrophy. The caudate lobe os drained by the emissary veins which drain into the IVC.

194
Q
The hepatic veins divide the liver longitudinally into:

a. Inferior right and left lobe segments
b. Posterior right and left lobe segments
c. Superior right and left lobe segments
d. Anterior right and left lobe segments

Study These Flashcards
A
C

The hepatic veins divide the liver longitudinally into the superior right and left lobe segments.

195
Q
All of the following correctly describe TCC, except:

a. Can cause unilateral or bilateral hydronephrosis
b. Painless hematuria is the most common symptom
c. Most commonly presents as diffuse bladder wall thickening with multiple masses within the wall
d. Most common bladder mass

Study These Flashcards
A
C
TCC forms in the transitional cells that line the renal pelvis, ureters and bladder. It is the most common bladder neoplasm. The most common symptom is painless macroscopic hematuria. Mass or focal wall thickening of the bladder wall are strong indicators of TCC. Sonographic characteristics include solitary or multiple hypoechoic mass formation with potential invasion into surrounding tissues. Unilateral hydronephrosis can be present with single ureter involvement, bilateral hydro seen with bilateral ureter involvement.

196
Q
The epididymis is normally __ to the teste.

a. Moderately hypoechoic
b. Moderately hyperechoic
c. Isoechoic to mildly hypoechoic
d. Isoechoic to mildly hyperechoic

Study These Flashcards
A
D
The sonographic appearance of the normal testes is an oval structure filled with homogeneous, uniform, medium level echoes. The mediastinum testis (AKA rete testes) is seen as a linear echogenic band within the testicle. The tunica albuginea is not normally visualized sonographically. The epididymis is normally isoechoic or mildly hyperechoic to the testicle. Doppler waveforms demonstrate high levels of antegrade, monophasic flow.

197
Q
Which of the following is associated with echogenic debris/calcification in the renal pyramids?

a. Angiomyolipomas
b. Pyelonephritis
c. Papillary necrosis
d. Hyperthyroidism

Study These Flashcards
A
C
Papillary necrosis refers to ischemia of the medullary pyramids. Sloughed papilla are identified in the urine and can cause urinary obstruction. The most common cause is analgesic abuse. Sonographic characteristics include swollen pyramids, clubbed calix, echogenic material and calcifications with shadowing within the collecting system and triangular cystic areas in the cortex.

198
Q
The splenic artery courses across the _ surface of the pancreas.

a. Anterior, inferior
b. Posterior superior
c. Posterior inferior
d. None of the above

Study These Flashcards
A
B
The head of the pancreas is usually anterior to the IVC, while the aorta is posterior to the neck. The celiac axis is a branch of the aorta that originates superior to the neck of the pancreas.

199
Q
A 43yr old female with a history of Hep-C presents with jaundice, increased abdominal girth and pain. The most probable US finding in the liver will be:

a. Metastasis from GI tract
b. Fatty infiltration
c. schistosomiasis
d. Cirrhosis and ascites

Study These Flashcards
A
D
The jaundice indicates chronic liver malfunction. Hep-C is associated with liver dysfunction and cirrhosis. Chronically decreased liver function usually leads to ascites formation in the abdomen.

200
Q
Increased levels of _ will reduce urine output and increase blood volume in the body.

a. Thyroxine
b. Angiotensis
c. Renin
d. Aldosterone

Study These Flashcards
A
D
Aldosterone secreted by the adrenal cortex increases the rate of sodium resorption and potassium excretion in the kidneys; adjusts urine output to increase or decrease blood volume.

201
Q
Which of the following is true regarding the anatomic position of the left renal gland?

a. Anterior to the stomach
b. Anterior to the pancreas tail
c. Lateral to the left diaphragmatic crura
d. Anterior to the IVC
e. Superiolateral to the left kidney

Study These Flashcards
A
C
Left adrenal gland- medial to the spleen, posterior to the pancreas tail and stomach, lateral to the aorta and left diaphragmatic crura.

202
Q
If you are trying to evaluate the cystic duct for a possible obstruction, where will you look?

a. Superior to the GB neck
b. Inferior to the GB neck
c. Anterior to the main portal vein, adjacent to the CBD
d. At the junction of the right and left hepatic ducts, just inside the liver

Study These Flashcards
A
A

203
Q
Branches of which of the following vessels supply the pancreas with blood?

a. SMA and right gastric
b. Proper hepatic and SMA
c. GDA and proper hepatic
d. SMA and GDA

Study These Flashcards
A
D

204
Q
The gastroesophageal junction is best seen in the __ plane. Anterior to the aorta and posterior to the left lobe of the liver.

a. Coronal
b. Radial
c. Longitudinal
d. Transverse

Study These Flashcards
A
C

205
Q
The renal cortex contains which of the following?

a. Calyces
b. Pyramids
c. Loops of Henle
d. Bowman’s capsule

Study These Flashcards
A
D
The pyramids and loops of Henle are located in the medulla portion of the parenchyma of the kidney. The calyces are a part of the collecting system in the renal sinus. Bowman’s capsules are located in the cortex portion of the parenchyma and are responsible for blood filtration.

206
Q
The most common benign, solid lesion of the thyroid is __.

a. Cyst
b. Adenoma
c. Goiter
d. Hematoma

Study These Flashcards
A
B
A cyst and a hematoma are not considered solid masses. A goiter is a diffuse condition. The adenoma is the most common solid, benign thyroid mass.

207
Q
On an US exam, the normal adrenal cortex appears:

a. As a hypoechoic ring around the echogenic medulla
b. As an anechoic ring around the echogenic medulla
c. As a hyperechoic ring around the hypoechoic medulla
d. Isoechoic to the medulla

Study These Flashcards
A
A

208
Q
Ascites may appear complex and contain debris/septations if there is associated __.

a. Internal bleeding
b. Bacterial infection
c. Viral infection
d. All of the above

Study These Flashcards
A
D

209
Q
A vascular condition of the liver frequently seen in women who take oral contraceptives is:

a. Portal aneurysm
b. Hereditary telangiectasia
c. Budd Chiari syndrome
d. Liver adenoma

Study These Flashcards
A
C

The other 3 choices are malignancies.

210
Q
Which type of liver neoplasm can be stimulated to grow in a pregnant pt?

a. Schistosomiasis
b. Granuloma
c. Hemangioma
d. Hepatoma

Study These Flashcards
A
C

Estrogen replacement therapy and pregnancy can cause a hemangioma to increase in size.

211
Q
The formation of Hartmann’s pouch usually occurs in what portion of the GB?

a. Neck
b. Body
c. Fundus
d. Cystic duct

Study These Flashcards
A
A

212
Q
Which of the following is associated with the production of milk of calcium bile?

a. Acute cholecystitis
b. Chronic cholecystitis
c. Hepatoma
d. Cirrhosis

Study These Flashcards
A
B

213
Q
Ascites can be identified in all of the following spaces/areas except:

a. Paracolic gutters
b. Between the coronary ligaments within the bare area of the liver
c. Subhepatic space below the right lobe
d. Morrison’s pouch

Study These Flashcards
A
B

Because the liver is in contact with the diaphragm in the bare area, fluid con not accumulate there.

214
Q
Most malignant tumors will demonstrate _ flow on Doppler evaluation.

a. Absent
b. Low resistance
c. High resistance
d. Increased diastolic flow

Study These Flashcards
A
B
Most malignancies are constantly growing and “invading” so this requires constant flow throughout the cardia cycle. Doppler will demonstrate increased diastolic flow consistent with a low resistance vascular bed.

215
Q
The right CCA is a branch of which artery?

a. Right subclavian
b. Right vertebral
c. Innominate
d. Aortic arch

Study These Flashcards
A
C

216
Q
A Reidel’s lobe is a normal liver variant defined as:

a. Tongue-like extension of the right lobe
b. Elongated left lobe that comes in contact with the spleen
c. Elongated right lobe that comes in contact with the spleen
d. Tongue-like extension of the caudate lobe

Study These Flashcards
A
A

217
Q
Asplenia is associated with all of the following, except:

a. Absence of the spleen
b. Aorta of the right side of the body
c. Congenital heart defects
d. Liver on the right side, GB on the left

Study These Flashcards
A
D
Asplenia is a type of heterotaxia that causes double right-sidedness. It involves absence of the spleen, right-sided aorta, left sided IVC, midline liver and GB, horseshoe kidneys and congenital heart defects are common.

218
Q
They thyroid normally produces the most of which of the following hormones?

a. Thyrotropin
b. TSH
c. T3
d. T4

Study These Flashcards
A
D

219
Q
Portal thrombosis may lead to an accumulation of fluid in the abdomen called:

a. Ascites
b. Retroperitoneal effusion
c. Pleural effusion
d. Pericardial effusion

Study These Flashcards
A
A

220
Q
Billroth’s cord in the red pulp of the spleen are responsible for _ destruction. Malphigian’s corpuscles in the white pulp of the spleen are responsible for production.

a. Platelet, lymphocyte
b. Lymphocyte, platelet
c. Lymphocele, RBC
d. RBC, lymphocyte

Study These Flashcards
A
D

221
Q
A 4-month old presents for an abdominal US following a recent Kasai procedure. Which of the following US findings would indicate the procedure was a success?

a. Echogenic mesh in the abdominal wall used to close an umbilical hernia
b. Echogenic mesh in the abdominal wall used to close an inguinal hernia
c. Normal liver texture with normal biliary diameter
d. Echogenic graft connecting the hepatic and portal venous systems in the liver with flow velocity greater than 40cm/s

Study These Flashcards
A
C
The Kasai procedure is performed to treat biliary atresia. A segment of the ductal system is directly connected to a portion of the GI tract. Biliary tree dilation should reduce and the expected homogeneous liver appearance should return to normal.

222
Q
Which of the following will cause an increase in alkaline phosphatase values?

a. Glycogen storage disease
b. Adenoma
c. Focal nodular hyperplasia
d. HCC
e. Hepatoblastoma

Study These Flashcards
A
D

223
Q
Which of the following describes the most common sonographic appearance of hepatoma?

a. Solid mass that may be more or less echogenic than liver parenchyma
b. Usually causes multiple diffuse solid mass formation
c. Most commonly demonstrates diffuse coarse liver texture in the affected lobe
d. Echogenic mass with a central scar

Study These Flashcards
A
A

224
Q
Which of the following best describes the US appearance of nephrocalcinosis?

a. Hyperechoic renal pyramids that may shadow
b. A large calculus within the renal pelvis causing hydronephrosis
c. Multiple linear calcifications within the renal parenchyma
d. 3 or more renal calculi identified in each pole of the kidney

Study These Flashcards
A
A

225
Q
On a longitudinal scan, this vessel is seen in cross section, posterior to the IVC:

a. Right renal vein
b. Right renal artery
c. Left renal vein
d. Left renal artery

Study These Flashcards
A
B

226
Q
When trying to distinguish free fluid in the abdomen from the IVC, which of the following is true?

a. The IVC will not demonstrate respiratory changes in diameter, while fluid pockets will vary with respiration
b. Color Doppler is ineffective in differentiating the IVC from free fluid
c. The IVC is located between the caudate lobe and ligamentum venosum within the liver
d. The IVC courses posterior to the caudate lobe

Study These Flashcards
A
D

227
Q
A pt is referred for an abdominal US to rule out a post-transplant fluid collection. Which of the following is least likely to be a complication of a liver transplant?

a. Biloma
b. Hematoma
c. Seroma
d. Urinoma
e. Abscess

Study These Flashcards
A
D

All other options are related to the liver a urinoma is not.

228
Q
While scanning the abdomen, you note the GB is moderately overdistended. Which of the following is a potential cause for the finding?

a. Right hepatic duct stone
b. Gastric mass
c. Portal HTN
d. Mass in the ampulla of Vater

Study These Flashcards
A
D
The ampulla of vater connects to the duodenum and is formed by the junction of the CBD and the pancreatic duct. A mass in this location will lead to dilated biliary tree and pancreatic ducts.

229
Q
The _ is a connective sheath that is connected to the large intestine that provides structure and support, along with encasing/protecting blood vessels.

a. Lesser omentum
b. Mesocolon
c. Mesentery
d. Greater omentum

Study These Flashcards
A
B

230
Q
Normally, there are _ parathyroid glands in the adult neck.

a. 2
b. 8
c. 6
d. 4

Study These Flashcards
A
D

There are usually 2 on each side of the trachea. One set is usually more superior than the other.

231
Q
What is the most common cause of cholangitis?

a. Klatskin’s tumor
b. Choledocolithiasis
c. Adenomyomatosis
d. AIDS

Study These Flashcards
A
B

232
Q
The _ separates the caudate lobe from the left lobe of the liver.

a. Falciform ligament
b. Main lobar fissure
c. Ligamentum teres
d. Ligamentum venosum

Study These Flashcards
A
D

233
Q
US demonstrates an enlarged GB with a rounded, fluffy, non-shadowing, mobile mass in the lumen. Which of the following is the correct diagnosis?

a. GB polyp
b. GB metastasis
c. Sludge ball
d. Cholelithiasis

Study These Flashcards
A
C

The key words are non-shadowing and mobile.

234
Q
Which testicular tumor is composed of bone, cartilage and smooth muscle fibers?

a. Choriocarcinoma
b. Seminoma
c. Teratoma
d. Yolk sac tumor

Study These Flashcards
A
C

235
Q
Acute cholecystitis is associated with all of the following, except:

a. Fever
b. McBurney’s point
c. RUQ pain
d. Positive Murphy’s sign

Study These Flashcards
A
B

McBurney’s point is the most common location of the appendix.

236
Q
Which of the following describes the appearance of the liver in a pt in the later stages of cirrhosis caused by viral hepatitis?

a. Multiple nodules usually 1-5cm in size
b. Diffuse coarse liver texture with no nodule formation
c. Significant hepatomegaly with diffuse decrease in hepatic texture
d. Multiple nodules usually <1cm in size

Study These Flashcards
A
A
Micronodular cirrhosis- multiple nodule <1cm; usually associate with alcoholism
Macronodular cirrhosis- multiple nodules usually 1-5cm in size; usually associated with chronic viral hepatitis.

237
Q
Which of the following is a characteristics of a normal lymph node?

a. Rounded shape
b. Thin hypoechoic rim with hyperechoic central hilum
c. Posterior enhancement
d. All of the above

Study These Flashcards
A
B
Normal lymph nodes are ovoid is shape. They do not demonstrate posterior enhancement. As they enlarge they become rounded in shape.

238
Q
The aorta bifurcates:

a. 3-5cm below umbilicus
b. At the level of the sacrum
c. 3-5cm below the SMA origin
d. At the level of the L4 vertebral body

Study These Flashcards
A
D

239
Q
Which muscle group is located posterior to the thyroid gland?

a. Tracheal muscles
b. Sternocleidomastoid
c. Longus coli
d. Strap muscles

Study These Flashcards
A
C

240
Q
Which of the following renal pathologies will appear the same sonographically as focal nodular hyperplasia?

a. Renal infarct
b. Pyonephritis
c. Angiomyolipoma
d. Renal adenoma

Study These Flashcards
A
A

Both will demonstrate normal renal size with a focal, indistinct, hypoechoic wedge shaped segment of parenchyma.

241
Q
Which scrotal arteries penetrate the testicular parenchyma to supply oxygenated blood.

a. Centripetal
b. Capsular
c. Cremasteric
d. Deferential

Study These Flashcards
A
A
Capsular courses along the testicle periphery and produces branches called centripetal arteries which course through the parenchyma. The deferential supplies epi and vas deferens. The cremasteric supplies scrotal sac.

242
Q
A pt presents with a history of AIDS and recent pneumocystis carinii infection. What changes to the liver do you expect to see on the US exam.

a. Diffuse increase in echogenicity with ascites
b. Gas-producing abscess near porta hepatis
c. Formation of multiple abscesses within the liver
d. Diffuse echogenic foci throughout the liver

Study These Flashcards
A
D

243
Q
The most common secondary malignancy of the testicles is:

a. Lymphoma
b. Leukemia
c. Hypernephroma
d. Myeloma

Study These Flashcards
A
A

244
Q
Splenic atrophy in adults is often associated with:

a. Sickle cell anemia
b. Splenitis
c. Typhoid fever
d. TB

Study These Flashcards
A
A
Sickle cell anemia is determined by abnormally shaped RBC’s that carry abnormal forms of hemoglobin. These abnormal cells can cause pooling of the blood in the spleen. The spleen will become enlarged and painful from the increase in blood volume early in life. After repeated episodes of splenic sequestration, the spleen becomes scarred and permanently damaged causing atrophy.

245
Q
A pt presents with a recent diagnosis of AIDS. Which of the following statements is true regarding the sonographic appearance of the spleen?

a. Multiple hypoechoic tumors will be present causing a marked increase in size
b. The spleen will be severely enlarged with numerous puncate calcifications
c. The splenic hilum should be evaluated for varices
d. The spleen will be mild-moderately enlarged with a homogeneous texture

Study These Flashcards
A
D
Leukemia, lymphoma, myelofibrosis= severe splenomegaly
AIDS, portal HTN, infection= mild-moderate splenomegaly

246
Q
Steatosis refers to:

a. Fatty liver disease
b. Pneumocystitis carinii
c. Chronic hepatitis infection
d. Glycogen storage disease

Study These Flashcards
A
A

247
Q
Acute pain in the RLQ is commonly associated with , while acute pain in the LLQ is commonly associated with _.

a. Appendicitis, irritable bowel syndrome
b. Appendicitis, cecal colitis
c. Appendicitis, diverticulitis
d. Diverticulitis, irritable bowel syndrome

Study These Flashcards
A
C

248
Q
A mass is identified in the left testicle. It is rounded with well-defined borders. There are internal rings of hyper and hypoechoic tissues giving the appearance of an onion. No color flow is demonstrated within the mass. Which of the following most likely describes this finding?

a. Sperm granuloma
b. Testicular infarct
c. Dilated rete testes
d. Epidermoid cyst

Study These Flashcards
A
D
Epidermoid cysts have fibrous walls that may calcify. They are described as having an onion-like appearance due to alternating rings of hyper and hypoechoic tissues. These cysts contain thick keratin and are avascular.

249
Q
Which zone of the prostate has the highest risk for developing BPH?

a. Central
b. Transitional
c. Peripheral
d. Stromal

Study These Flashcards
A
B

95% of BPH occurs in the transitional zone.

250
Q
The most common cause of splenomegaly is __.

a. Malignant HTN
b. AIDS
c. Mononucleosis infection
d. Portal HTN

Study These Flashcards
A
D

251
Q
All of the following will demonstrate normal values on liver function tests, except:

a. Glycogen storage disease
b. Polycystic disease of the liver
c. Focal nodular hyperplasia
d. Adenoma
e. Fungal abscess

Study These Flashcards
A
E

252
Q
All of the following are true regarding a pancreatic pseudocyst, except:

a. Pt will present with a fever and leukocytosis
b. They are usually found in the tail of the pancreas
c. There are usually multiple pseudocysts present when the abnormality is found
d. They can be mistaken for a splenic or renal mass depending on their location

Study These Flashcards
A
C
A pseudocyst forms as an accumulation of digestive enzymes form and the body attempts to wall off the accumulation of the enzymes by forming a reservoir. In most cases, only one pseudocystic area forms as the fluid builds up.

253
Q
If the SMA is identified to the right of the SMV in a pt with acute abdomen symptoms. What does this suggest?

a. Midgut malrotation
b. Normal anatomy
c. Intussusception
d. Pyloric stenosis

Study These Flashcards
A
A
Midgut malrotation refers to any variation in the normal rotation and fixation of the GI tract during development. It is usually associated with malposition of the SMA and SMV.

254
Q
Leukopenia means:

a. Decreased WBC count
b. Increased lymph production
c. Increased platelets
d. Increased WBC count

Study These Flashcards
A
A

255
Q
Chronic renal disease is least likely to be associated with which of the following?

a. Small kidneys
b. Enlarged kidneys
c. Unilateral hydro
d. Bilateral hydro

Study These Flashcards
A
B

Chronic renal disease leads to renal atrophy and a decrease in overall size.

256
Q
The primary indication for a liver transplant in an adult is _ and in children _ is the most common indication of liver transplant.

a. Hep C, biliary atresia
b. Cirrhosis, hep C
c. Hep C, cirrhosis
d. Biliary atresia, hep C

Study These Flashcards
A
A

257
Q
Which of the following is a common cause of cystic fibrosis?

a. AAA
b. Meconium ileus
c. Portal HTN
d. Renal calculi

Study These Flashcards
A
B
The pancreas is isoechoic to the liver as a child and increases in echogenicity as the pt reaches adulthood. Cystic fibrosis causes an increase in echogenicity of the gland with mucus production and gland fibrosis. The mucus can cause meconium ileus.

258
Q
Which of the following statements best describes the appearance of color Doppler evaluation of a testicular abscess?

a. There will be focal areas of increased vascularity in the central abscess and limited color flow demonstrated at the periphery of the abscess
b. There will be a significant increase in vascularity throughout the area of the abscess due to inflammation
c. There will be no flow demonstrated centrally with increased color flow displayed around the periphery of the abscess
d. Due to fluid accumulation with inflammation, color Doppler does not usually demonstrate flow around or within an abscess

Study These Flashcards
A
C
The central portion of an abscess is fluid and debris with no vascularity. The surrounding tissues are inflamed which leads to increased flow.

259
Q
The most common malignant renal tumor in adults is:

a. Nephroblastoma
b. RCC
c. Wilm’s tumor
d. TCC

Study These Flashcards
A
B
Nephroblastom (AKA Wilm’s tumor) is the most common renal malignancy in children. TCC is malignancy of the transitional cells of the ureters, bladder and renal pelvis.

260
Q
Pts with _ have a significantly increased risk of developing a seminoma.

a. Diabetes
b. Cryptorchidism
c. Varicocele
d. Hydrocele

Study These Flashcards
A
B

261
Q
A 35yr old male pt presents for a RUQ US due to pain, increased alkaline phosphatase and conjugated bilirubin. You identify dilated intrahepatic ducts in both lobes of the liver, while the CBD and GB appear within normal limits. Which of the following describes the cause for these findings?

a. Stone in the right hepatic duct
b. Stone in the left hepatic duct
c. Biliary atresia
d. Stone in the CHD

Study These Flashcards
A
D
The lab indicates biliary obstruction. The presence of dilated INTRAhepatic ducts in both lobes indicates that the right and left hepatic ducts are obstructed=CHD.

262
Q
Liver transplant evaluation requires PW Doppler evaluation of all the following anastomosis sites, except:

a. Hepatic arterial
b. IVC
c. Biliary
d. Portal

Study These Flashcards
A
C

The biliary anastomosis site is not vascular therefore there is no need or even ability to use PW Doppler at this site.

263
Q
The most common cause of small bowel obstruction in adults is . The most common cause of small bowel obstruction in an infant is _.

a. Bezoars, pyloric stenosis
b. Diverticulitis, pyloric stenosis
c. Extrinsic compression by a pelvic mass, intussusception
d. Adhesions, intussusception

Study These Flashcards
A
D

264
Q
Which of the following describes a non-infectious cause of hepatitis?

a. Anabolic steroids
b. Bacteria
c. Fungus
d. Virus

Study These Flashcards
A
A

265
Q
While scanning the urinary bladder using color Doppler, the left urinary jets is detected but is partially obstructed by a thin membrane that balloons into the bladder lumen. What does this finding most likely represent?

a. Posterior urethral valves
b. Urethrocele
c. UPJ obstruction
d. Ureterocele

Study These Flashcards
A
D
Ureteroceles are thin membranes that cover the junction of the ureter and bladder. Posterior urethral valves are abnormal structures in the urethral tube causing a backlog pf urine starting in the bladder and extending to the kidneys.

266
Q
Which of the following can lead to a false positive diagnosis of stones in the biliary system?

a. Shadowing from surgical clip in porta hepatis
b. Improperly decreased TGC settings
c. Decreased color Doppler gain
d. Rejection setting too high

Study These Flashcards
A
A
Surgical clips are highly echogenic and cause posterior shadowing which can cause a false positive diagnosis of a ductal stone. Rejection removes low level echoes from the image. If this setting is too high, the image will be overly grainy. This could remove reflections from small stones leading to a false negative. Decreased gain could also remove important information from reflectors in the GB leading to a false negative for stones.

267
Q
A pt presents with a small, palpable extratesticular mass on the left side. The US exam demonstrates a mobile, hyperechoic mass with posterior shadowing. Which of the following best describes the findings?

a. Microlithiasis
b. Scrotal pearl
c. Adenomatoid tumor
d. Abscess

Study These Flashcards
A
B

A scrotal pearl refers to an extratesticular calcification.

268
Q
A 28yr old male pt presents with jaundice, increased AST & ALT, decreased albumin and elevated serum copper levels. The liver appears diffusely echogenic with periportal thickening. Which of the following is the most likely cause for these findings?

a. Alcoholic cirrhosis
b. Chronic hepatitis
c. Wilson disease
d. Glycogen storage disease

Study These Flashcards
A
C

Elevated serum copper level is indicative of Wilson disease.

269
Q
A pt presents for an abdominal US due to a history of median arcuate ligament syndrome. What structure(s) should be closely evaluated for related findings?

a. Portal system
b. Celiac axis
c. Splenic artery and vein
d. Renal arteries
e. Left renal vein

Study These Flashcards
A
B
The celiac axis can be compressed by the median arcuate ligament near the diaphragm. Symptoms may be intermittent with respiration. Symptoms appear with expiration and resolve with inspiration.

270
Q
The greatest advantage of US guided percutaneous needle biopsy over biopsy performed with CT guidance is:

a. Lack of radiation exposure
b. Ability to view the biopsy needle passing into the mass in real time
c. Decreased cost to the pt
d. Ability to take multiple samples during one procedure

Study These Flashcards
A
B
Advantages of US guided biopsy:

1 advantage is real time visualization of the needle massing through the area of interest to confirm an accurate sample is taken. Accurate diagnosis in 95% of cases. If you can see it on US you can biopsy it by US guidance. Minimally invasive. No radiation. Cost effective. Prevents surgical removal of tissue for analysis. Allows for multiple pt positions and approaches.

271
Q
An obese pt has a known hepatoma in the posterior aspect of the right lobe of the liver. Which probe would you use to assist the radiologist in performing a biopsy?

a. 2.25MHz, short focus
b. 3.5MHz, long focus
c. 5MHz, short focus
d. 2.25MHz, long focus

Study These Flashcards
A
D
The lower the probe frequency, the better the penetration of the beam. The longer the focus, the better the resolution of deeper structures.

272
Q
A pt presents with LUQ pain and a history of MVA 3 days ago, a cystic structure is seen in the spleen with layered debris within it. This most likely represents:

a. Carcinoma
b. Hydatid cyst
c. Hemangioma
d. Hematoma

Study These Flashcards
A
D
The recent history and the cystic structure with debris indicate a possible hematoma. A hemangioma is a benign formation of vascular tissues. A hydatid cyst is a parasitic infection that leads to cyst formation most commonly in the liver. Carcinoma formation usually presents as a solid, irregular mass.

273
Q
Which of the following would improve the visualization of the thyroid isthmus?

a. Stand off pad
b. Color Doppler
c. Extended field of view
d. Compound imaging

Study These Flashcards
A
A

A stand off pad eliminates the dead zone on the image. This is helpful for very superficial structures.

274
Q
Increased serum levels of __ usually provide the first indication of an extrahepatic biliary obstruction.

a. Aldosterone
b. Direct bilirubin
c. Alkaline phosphatase (ALP)
d. Indirect bilirubin

Study These Flashcards
A
C
Increased serum levels of ALP usually provide the first indication of an extrahepatic biliary obstruction. The levels elevate quickly and usually to a level that is 3 or more times the normal value.

275
Q
Which of the following is true regarding hand washing recommendations in the standard precautions guidelines?

a. If gloves are used during an exam, there is no need to wash your hands with soap and water when finished
b. Hands should always be washed after assisting with any type of interventional procedure
c. 5-10 seconds of lathering and rinsing I sufficient for killing most germs
d. Hands should be cleaned in cidex following any procedure involving blood or body fluids

Study These Flashcards
A
B

276
Q
A pt presents for a renal/adrenal scan due to a history of Cushing’s syndrome. What lab value will be increased in this pt?

a. Cortisol only
b. Aldosterone only
c. Adrenocorticotropic hormone only
d. Adrenocorticotropic hormone and cortisol

Study These Flashcards
A
D
Excessive production of adrenocorticotropic hormone (ACTH) by the pituitary gland will cause the adrenal gland to produce excessive cortisol.

277
Q
Which of the following liver vessels course within the main lobar fissure?

a. Common Hepatic artery
b. Middle hepatic vein
c. Main portal vein
d. Right hepatic vein

Study These Flashcards
A
B

The main lobar fissure connects the GB neck to the right portal vein.

278
Q
A pt presents with mildly increased ALT and significantly increased levels of AST. Which of the following is an expected finding on the US exam?

a. Acute hepatitis
b. Mirizzi’s syndrome
c. Liver metastasis
d. Choledocholithiasis

Study These Flashcards
A
C
Comparison of the AST and ALT levels is used to diagnose the associated liver abnormality. Significant increases in AST with mild increase in ALT indicates cirrhosis or metastatic disease is present. Significant increase in ALT with mild increase in AST indicates acute hepatitis or a benign hepatic obstruction is present.

279
Q
A 56yr old male presents with a 4yr history of cirrhosis. When performing the US exam, the size of which of the following structures should be assessed to evaluate the effects of cirrhosis?

a. Caudate lobe and hepatic artery
b. Portal vain, hepatic vein, hepatic artery
c. Left lobe and caudate lobe
d. Right lobe, portal vein, caudate lobe

Study These Flashcards
A
D
The caudate lobe is normally sparred from the atrophy associated with cirrhosis. Comparing the size of the caudate lobe to the size of the right lobe aids in assessment of liver atrophy seen with cirrhosis. The portal vein will dilate as the cirrhosis progresses. A portal vein diameter of >13mm is abnormal.

280
Q
A pt’s chart indicates that they have a history of cavernous transformation. Where does this abnormality form?

a. GB fossa
b. Porta hepatis
c. Pancreatic tail
d. Morrison’s pouch

Study These Flashcards
A
B
Cavernous transformation refers to the dilation of the portal system varices at the entrance of the liver. Commonly caused by portal HTN.

281
Q
Internal rotation of the arm is preferred for evaluation of the , while external rotation of the arm is preferred for evaluation of the .

a. Long bicep tendon, short bicep tendon
b. Flexor tendon, biceps tendon
c. Subscapularis, supraspinatus
d. Supraspinatus, subscapularis

Study These Flashcards
A
D

282
Q
Harmonic imaging leads to:

a. Decreased lobe artifact
b. Improved axial but degraded lateral resolution
c. Increased image noise
d. Reduced signal to noise ratio

Study These Flashcards
A
A
Tissue harmonic imaging reduces noise and clutter by improving signal to noise ratio and resolution. As an US wave propagates through the media, a change occurs in the shape and frequency of the transmitted signal. The change is due to the normal resistance of tissues/particles to propagate sound energy. This resistance and the resulting signal change is called harmonic oscillation. The input frequency doubles the output frequency in harmonic frequency. EX: transmit frequency of 4MHz for maximum penetration with a return frequency of 8MHz for improved resolution. The returning higher frequency signal has to only travel one direction to the probe. The advantages of high frequency imaging and the one-way travel effects are decreased reverberation, beam aberration, and side lobes, as well as increased resolution.

283
Q
The radiologist asks you to set up the US room for a thyroid biopsy. Which probe should be covered with a sterile probe cover?

a. Curvilinear 8MHz
b. Linear 10MHz
c. Linear 5MHz
d. Vector 10MHz

Study These Flashcards
A
B

284
Q
Which of the following is not a technique utilized for a 6yr old pt that is anxious about their renal exam?

a. Show them the probe and gel. Demonstrate how the machine works on your arm
b. Allow mom to come into the room during the exam
c. Turn on a DVD of cartoons
d. Show them an article on the importance of renal US in children

Study These Flashcards
A
D

285
Q
All the following would be associated with an adrenal mass, except?

a. Pheochromocytoma
b. Conn’s disease
c. Nephroblastoma
d. Cushing’s syndrome

Study These Flashcards
A
C
A nephroblastoma is a primary renal malignancy. Adrenal tumors are commonly seen with cushing’s syndrome and conn’s disease. A pheochromocytoma primarily presents as a benign tumor of the adrenal gland that causes symptoms similar to anxiety.

286
Q
The right and left hepatic ducts come together to form the common hepatic duct:

a. Inside the liver, near the porta hepatis
b. At the pancreas head
c. Deep within the liver parenchyma
d. At the level of the Amplulla of Vater

Study These Flashcards
A
A
The right and left hepatic ducts join together inside the liver, near the porta hepatis. The cystic duct joins the CHD at the porta hepatis to form the CBD as it exits the liver.

287
Q
Which of the following are true regarding paracentesis?

a. The preferred entry point for fluid removal is on the right side at the level of Morrison’s pouch
b. Because the diagnostic paracentesis is performed with a long thin needle instead of a catheter, a sterile field is not required
c. An informed consent is not required when a diagnostic paracentesis is performed, only for a therapeutic paracentesis
d. Paracentesis requires the use of a sterile field technique

Study These Flashcards
A
D

288
Q
Which of the following biliary abnormalities would be an acute complication caused by a bacterial infection of the GB?

a. Porcelain GB
b. Adenomyomatosis
c. Strawberry GB
d. Emphysematous cholecystitis

Study These Flashcards
A
D
Emphysematous cholecystitis is an acute complication caused by a bacterial infection of the GB. It is a critical finding that should be reported to the radiologist immediately. Porcelain GB may develop after chronic infection of the wall. The other 2 choices are chronic conditions that affect the GB.

289
Q
Which of the following lists an advantage of fine needle capillary technique over a fine needle aspiration technique?

a. FNC aspirates a smaller but more concentrated sample of cells than FNA
b. FNC collects a tissue core while FNA collects a small amount of cells
c. FNC is less traumatic than FNA
d. FNC does not require local anesthetic and FNA does

Study These Flashcards
A
A
FNC aspirates a smaller but more concentrated sample of cells than FNA. Both procedures use local anesthetic to numb the area around the puncture site. FNC and FNA procedures are used to collect cells from a mass.

290
Q
You are scanning the GB and there appears to be sludge within the lumen. Which of the following will aid in differentiating the sludge from artifact?

a. Apply color Doppler
b. Ask the pt to fast for 3-4 more hours and rescan the GB for changes
c. Ask the pt to perform the Valsalva maneuver
d. Increase the transducer frequency

Study These Flashcards
A
D

291
Q
A large __ could lead to propagation speed artifact.

a. Complex cyst
b. Simple cyst
c. Cavernous hemangioma
d. Renal angiomyolipoma

Study These Flashcards
A
D
The speed of sound in fatty tissue is slower than in soft tissue. A large fatty tumor could lead to structures posterior to the mass being displayed deeper on the image than they actually are in the body.

292
Q
An average sized pt presents with a fever, acute RLQ pain and vomiting. The ER physician wants to rule out appendicitis. Which of the following techniques would be used in this exam?

a. Use a stand off pad to better visualize the superficial bowel loops
b. Drink 32oz of water to fill the bladder prior to the exam
c. Apply graded transducer compression over the area of interest
d. Ask the pt to perform the Valsalva maneuver while scanning

Study These Flashcards
A
C
Graded compression should demonstrate pliable loops of bowel with normal peristalsis. Rebound pain is a strong indicator of appendicitis.

293
Q
All of the following are examples of high level disinfection chemicals, except:

a. Ortho-phathaldehyde (OPA)
b. 70% ethanol
c. Glutaraldehyde
d. Hydrogen peroxide

Study These Flashcards
A
B

294
Q
A pt presents with mild scrotal swelling and pain on the left side that started 2-3 days ago. The pt awoke this morning with a slight fever and increased swelling. Which of the following is not an expected finding on the US exam?

a. Orchitis
b. Epididymitis
c. Hydrocele
d. Intratesticular abscess
e. Testicular infarct

Study These Flashcards
A
E
Acute pain and swelling, not associated with trauma, are usually caused by infect. Scrotal infections can lead to hydrocele formation that will present as swelling. A testicular infarct would not be related to fever.

295
Q
Which of the following statements regarding insulinomas is false?

a. They commonly occur in the body or tail of the pancreas
b. Pts are usually hypoglycemic
c. They usually occur in pts 40-60yrs of age
d. Most pts require an insulin pump to counteract the effects of the tumor on blood sugar levels

Study These Flashcards
A
D
An insulinoma causes an increase in insulin in the blood stream leading to hypoglycemia. An insulin pump is for those who do not produce enough insulin.

296
Q
The most common location for a urinary tract obstruction caused by a renal calculi is in the _.

a. Distal urethra
b. Proximal ureter
c. Trigone of the bladder
d. Distal ureter

Study These Flashcards
A
D

297
Q
_ resolution is defined as the ability of the US system to differentiate structures on an image based on variations of brightness of the echo/pixel.

a. Elevational
b. Lateral
c. Contrast
d. Axial

Study These Flashcards
A
C

298
Q
Which of the following is normally identified in the urine?

a. Creatinine
b. Minimal amounts of RBCs
c. Glucose
d. Protein

Study These Flashcards
A
A
Substances NOT normally found in the urine: sugar (glucose), RBCs, WBCs, protein and acid keytones. Creatinine is normally filtered from the blood and expelled in the urine.

299
Q
Which of the following describes reverse protective isolation?

a. Used to protect others from germs in the pt’s bowels, bladder and stomach
b. Pt is separated from others to prevent the spread, by contact or airborne transmission, of their highly contagious diseases
c. Used to protect others from germs in the pt’s nose, mouth, throat and lungs. It is used for diseases that are airborne
d. Pt is separated from others to protect them from other pt’s germs and is used primarily for pts who have extremely impaired immune systems

Study These Flashcards
A
D

300
Q
The most common indication of renal artery stenosis is:

a. Small kidneys
b. Thrombus in the renal veins
c. Benign HTN
d. Malignant HTN

Study These Flashcards
A
D
Malignant HTN is a significant increase in systemic BP that is unaffected by medication (AKA uncontrolled HTN). When the kidneys do not receive the necessary amount of blood, they activate the renin-angiotensin system. The hormones in this system cause the BP to rise until the kidneys are satisfied. If there is a stenosis in the renal artery, the BP will rise to a critical level and still not satisfy the kidney. Meds will not affect the BP due to the cause of the increase.

301
Q
Which of the following techniques will lead to degraded lateral resolution?

a. Increased probe frequency
b. Increased # of focal zones
c. Trapezoidal field of view on linear array
d. None of the above

Study These Flashcards
A
C
Increasing the size of the field of view by steering the beam into a trapezoidal shape will lead to increased beam divergence. Lateral resolution is reduced by wider beams.

302
Q
All of the following correctly describe Xanthogranulomatous pyelonephritis, except:

a. Most commonly bilateral
b. Prevalent in females and diabetic pts
c. Most cases demonstrate a staghorn calculus and thinned renal cortex
d. Caused by recurrent infections with obstruction

Study These Flashcards
A
A
Xanthogranulomatous pyelonephritis is most commonly a unilateral finding. Recurrent infections cause chronic obstruction to renal urine output. Stone formation is a commonly associated finding with most presenting with staghorn calculus formation. Females and diabetic pts have a higher risk of developing this abnormality.

303
Q
Which of the following is the most likely cause for renal vein thrombosis?

a. Decreased serum BUN and creatinine
b. Dehydration
c. Pulmonary embolism
d. Ascending urinary tract infection

Study These Flashcards
A
B
Dehydration will cause blood to thicken and the kidneys will slow their filtration of blood. These 2 factors can lead to thrombus formation in the renal vein.

304
Q
Which of the following is true regarding a liver hemangioma?

a. Color Doppler is not usually helpful in evaluating this highly vascular mass
b. A hemangioma is composed of an abnormal concentration of smooth muscle and kupfer cells within the liver
c. Fluid and debris levels are commonly present in a hemangioma
d. Most hemangiomas demonstrate some level of posterior shadowing

Study These Flashcards
A
A
Most hemangiomas demonstrate some level of posterior enhancement. Flow within the hemangioma is usually very slow and color Doppler is not an effective evaluation tool.

305
Q
A pt presents with a history of angiomyolipoma. Which of the following is most commonly affected by this neoplasm?

a. Spleen
b. Aorta
c. Kidney
d. Liver

Study These Flashcards
A
C
Angiomyolipomas are a hyperechoic mass found within the renal parenchyma or attached peripherally. They are most commonly associated with tuberous sclerosis. These masses can be found in the liver and spleen too but are much less common in these organs.

306
Q
What is the most common symptom of RCC?

a. Increased AFP
b. HTN
c. Hematuria
d. Proteinuria

Study These Flashcards
A
C

307
Q
Which of the following is least likely to demonstrate a hyperechoic liver mass on US?

a. Focal fatty sparing
b. Liver metastasis
c. Hepatic lipoma
d. Hepatic hemangioma

Study These Flashcards
A
A

308
Q
A pt presents for a post biopsy scan of the right kidney. A subcapsular hematoma is suspected. What will be the appearance of this abnormality on the image?

a. Free fluid beneath the renal capsule that distorts the contour of the parenchyma
b. Cystic formation with debris levels found within the renal sinus
c. Free fluid with debris levels found within Murphy’s pouch
d. Free fluid with debris levels found within Morrison’s pouch

Study These Flashcards
A
A
Blood that collects between the renal capsule and the renal cortex is an intracapsular hemorrhage. Blood that collects outside the capsule, between the capsule and the liver tissue is as extacapsular hemorrhage. An intracapsular hemorrhage will distort the renal cortex while and extracapsular hemorrhage does not.

309
Q
Which of the following correctly describes the ducts of Luschka?

a. Only visible in obese pts
b. Located within the 3rd segment of the duodenum
c. Bile thickening occurs within them
d. Associated with porcelain GB, and GB carcinoma

Study These Flashcards
A
C
Ducts of luschka: located in the GB wall, bile thickening occurs there, can result in the stasis of bile/debris which leads to inflammatory disease.

310
Q
Which of the following will not improve the color Doppler visualization of flow in the arcuate arteries?

a. Increased color gain
b. Increased PRF
c. Increased frequency
d. Larger packet size

Study These Flashcards
A
B
Packet size refers to the number of lines of information used to create the color displayed on the image. The more lines of data used, the larger the packet size, the better the color display. Decreasing PRF or scale will assist in visualization of slower flow. Low PRF are used to examine low velocity flow. The longer interval between pulses allows the scanner a better chance of identifying slow flow.

311
Q
Renal function lab testing include serum levels of:

a. WBC
b. BUN
c. Lipase
d. ALT

Study These Flashcards
A
B
BUN (blood urea nitrogen) levels are a direct assessment of the filtration function of the kidneys. WBC levels are related to the presence of infection. ALT is a liver enzyme that is assessed for liver function. Lipase is an enzyme produced by the pancreas for digestion.
312
Q
Which of the following correctly describes why the arcuate arteries are not easily or accurately evaluated by PW Doppler?

a. Because the blood flows parallel to the Doppler sound beam
b. Because the blood flows perpendicular to the Doppler sound beam
c. The renal capsule attenuates the Doppler signal so only limited information is reflected to the probe
d. Because the flow is very low resistance

Study These Flashcards
A
B

313
Q
Which acoustic artifact will be seen with hydrocele formation?

a. Comet tail artifact
b. Edge shadowing
c. Posterior shadowing
d. Posterior enhancement

Study These Flashcards
A
D

314
Q
The majority of the liver is covered by a thick capsule composed of fibrous and elastic elements called _.

a. Bowman’s capsule
b. Morrison’s pouch
c. Gerota’s fascia
d. Glisson’s capsule

Study These Flashcards
A
D
Bowman’s capsule is part of the renal nephron. Morrison’s pouch is the right renal fossa, the area between the right kidney and liver. Gerota’s fascia covers the kidney like glisson’s capsule covers the liver.

315
Q
Which of the following best describes the probe position used to evaluate the long axis of the pancreas?

a. TRV oblique, at the midline with the notch side of the probe rotated slightly superior to the side of the probe without the notch
b. SAG just to the left of midline with the notch rotated slightly to the left side of the pt
c. SAG just to the left of midline with the notch side of the probe slightly to the right of the pt
d. TRV oblique at the midline with the notch side of the probe rotated slightly inferior than the side without the notch

Study These Flashcards
A
D
The pancreas lies across the abdomen with its long axis best demonstrated in a TRV plane. The tail lies slightly superior to the head and body which requires an oblique TRV probe position.

316
Q
Which probe can best evaluate the surface nodularity of the liver in a pt with cirrhosis?

a. Curved 8MHz
b. Vector 10MHz
c. Curved 5MHz
d. Linear 7.5 MHz

Study These Flashcards
A
D

A linear probe provides the largest near field.

317
Q
All the following will be used to describe the normal sonographic appearance of the spleen, except:

a. Prominent internal vascularity
b. Homogeneous echotexture
c. Mottled appearance
d. Smooth borders

Study These Flashcards
A
C

318
Q
Most US systems send their exams to a digital image storage server called PACS. What does this stand for?

a. Pt archival and charting system
b. Picture accessibility and communication structure
c. Picture archiving and communication system
d. Privacy administration and charting system

Study These Flashcards
A
C

319
Q
Which of the following will reduce anisotropy artifact?

a. Dual screen imaging
b. Rocking the transducer
c. Turn off harmonic imaging
d. Reduce color Doppler gain

Study These Flashcards
A
B
Anisotropy artifact refers to falsely hypoechoic areas of a tendon. This occurs when the transducer is not completely parallel to the tendon. It can be reduced by rocking the transducer more parallel to the tendon. Curved tendons can be straightened by muscular contraction to make the tendon more parallel to the transducer.

320
Q
Adrenal myelolipoma is least likely to be associated with:

a. Comet tail artifacts
b. Attenuation artifacts
c. Increased attenuation
d. Propagation speed artifacts

Study These Flashcards
A
A
Adrenal myelolipomas are composed of fatty and bony elements that cause the mass to be echogenic an US. Fatty tissue causes increased attenuation. Diaphragm disruption due to propagation speed artifact may be seen. The decrease in the speed of sound in fatty tissue can cause the portion of the diaphragm that is posterior to the mass to be placed more posterior so the diaphragm looks ‘broken’. Comet tail artifacts are caused by repetitive reflections at a media boundary between very different tissues.

321
Q
Increasing the wall filter setting could reduce the display of all of the following, except:

a. Clutter artifact
b. Aliasing artifact
c. Cross talk artifact
d. Beam width artifact

Study These Flashcards
A
B
Beam width, clutter and cross talk all involve the incorrect display of lower level Doppler shifts. Increasing the wall filter can reduce the appearance of the weaker Doppler shifts on the image.

322
Q
The largest abdominal tributaries of the IVC are the:

a. Main portal vein
b. Gonadal veins
c. Renal veins
d. Hepatic veins

Study These Flashcards
A
D

323
Q
Which of the following statements is false regarding germ cell tumors of the testicle?

a. Seminomas are the most common type of germ cell tumor of the testes
b. Embryonal cell tumors commonly demonstrate coarse calcifications
c. Germ cell tumors are associated with cryptorchidism
d. Seminomas commonly demonstrate multiple cystic areas due to tumor necrosis

Study These Flashcards
A
D
Non-seminomatous germ cell tumors commonly demonstrate multiple cystic area due to tumor necrosis. Seminomas normally remain solid yet heterogeneous.

324
Q
B-color can be helpful in the evaluation of:

a. Anechoic fluid/cysts
b. Low velocity blood flow
c. High velocity blood flow
d. Subtle differences in tissue

Study These Flashcards
A
D
b-color is the colorize function on the 2D image. The different shades of a single color are used to display the reflected information on the image. Changing from gray scale to shades of color can change the contrast resolution.

325
Q
The _ is a fibrous sheath that is connected to the superior aspect lesser curvature of the stomach that provides structure and support, along with encasing/protecting the blood vessels.

a. Lesser omentum
b. Lesser mesentery
c. Greater omentum
d. Greater mesentery

Study These Flashcards
A
A

326
Q
Which of the following prostate abnormalities most significantly elevates the amount of PSA in lab testing?

a. BPH
b. Abscess
c. Carcinoma
d. Prostatitis

Study These Flashcards
A
C
BPH causes increased levels of PSA in the blood. Carcinoma also causes increased PSA (>10ng/ml) but usually much more significantly that BPH

327
Q
Which of the following vessels follow a retroaortic course in the abdomen?

a. Portal confluence
b. Left renal artery
c. Right renal artery
d. None of the above

Study These Flashcards
A
D

328
Q
Which metabolic disorder is associated with hepatic adenoma formation and fatty infiltration of the liver?

a. Hemochromatosis
b. Wilson disease
c. Glycogen storage disease
d. Amyloidosis

Study These Flashcards
A
C
Glycogen storage disease is an inherited disorder of glycogen metabolism. It usually affects either the liver or skeletal muscle. Excessive storage of glycogen occurs within the hepatocytes. It is associated with hepatomegaly, increased echogenicity similar to fatty liver and hepatic adenoma formation.

329
Q
Which of the following vessels is not connected to the IVC?

a. Right suprarenal
b. Right gonadal
c. Left gonadal
d. Right renal
e. Left hepatic

Study These Flashcards
A
C

330
Q
How can you differentiate a ganglion cyst from a giant cell tumor of the hand?

a. Giant cell tumor will demonstrate posterior enhancement, ganglion cyst will not
b. Ganglion cyst will demonstrate enhancement, giant cell tumor will demonstrate shadowing
c. Giant cell tumors will move along with the adjacent tendon, ganglion cyst will not
d. Ganglion cyst will demonstrate enhancement, giant cell tumor will not

Study These Flashcards
A
D
Because giant cell tumors are soft tissue masses, they will not demonstrate posterior enhancement. Ganglion cysts have the classic signs of a simple cyst on US, including enhancement. Giant cell tumors do not move with the adjacent tendon but neither do ganglion cysts.

331
Q
Applying color Doppler to the distended urinary bladder identifies 2 ureteral jets at the trigone of the bladder that are directed toward the opposing sides of the bladder. The color demonstrates the flow from the 2 ureters as an “X” within the bladder lumen. Which of the following statements best characterize these findings?

a. Bilateral posterior urethral valves are most likely present
b. The ureters area abnormally inserted into the bladder causing eccentric ureteral jet formation
c. This is a normal finding
d. The pt most likely has cystitis with ureteral inflammation leading to eccentric urine jet formation

Study These Flashcards
A
C
The ureters join the posterior inferior aspect of the bladder. The urine usually flows into the bladder at an angle toward the anterior wall. Color Doppler can be used to evaluate ureter patency and visualize the urinary jet into the bladder. If urine flows into the bladder from both jets at the same time, the jets can form an “X” pattern on color Doppler.

332
Q
The segmental anatomy of the liver is determined by:

a. Branching of the hepatic veins
b. Branching of the portal veins
c. Course of the fissures through the tissue
d. Course of the biliary vessels through the tissue

Study These Flashcards
A
B
The right and left lobes are divided by the branches of the main portal vein; right and left portal veins, right and left lobes. These lobes are further divided according to portal branches; right anterior and posterior, left medial and lateral.

333
Q
A 64yr old female presents with recent weight loss and mild jaundice. Lab values demonstrate normal WBC levels but increased levels of direct bilirubin, alkaline phosphatase and prothrombin time. The GB demonstrates a small wall mass that is isoechoic to the wall tissues. The wall measures 4.5mm in maximum thickness. Multiple stones are present within the lumen. Which of the following best describes the findings on the image?

a. Acute cholecystitis, polyp formation and cholelithiasis
b. Primary GB carcinoma and cholelithiasis
c. Acute cholecytitis and cholelithiasis
d. Adenomyomatosis

Study These Flashcards
A
B
Primary GB carcinoma most commonly presents as diffusely thickened GB wall with cholelithiasis. Increased levels of direct bilirubin, alkaline phosphatase and prothrombin time along with normal WBC count indicates no inflammation.

334
Q
A pt presents with increasing RUQ pain and vomiting after a recent liver biopsy. An irregular mass with internal debris and an overall heterogeneous appearance is noted in the right lobe. The mass also demonstrates several echogenic foci with ring down artifact. What is the most likely diagnosis of this finding?

a. Pseudoaneurysm
b. Lymphocele
c. Hematoma
d. Abscess
e. Seroma

Study These Flashcards
A
D

335
Q
Which blood vessel is located lateral to the right lobe of the thyroid and demonstrates an anechoic circular shape in a TRV view of the thyroid?

a. Right CCA
b. Right IJV
c. Left CCA
d. Superior vena cava

Study These Flashcards
A
A

336
Q
What vessel separates the right and left lobes of the liver?

a. Right portal vein
b. Middle hepatic vein
c. Common hepatic vein
d. Left hepatic vein

Study These Flashcards
A
B

337
Q
A 3yr old presents for a renal scan, complaining of left sided pain. No renal abnormality was identified on the exam but a common anatomical variant was. What is the term for a single cortical bulge off of the lateral border of the left kidney?

a. Column of Bertin
b. Fetal lobulation
c. Dromedary hump
d. Cortical hyperplasia

Study These Flashcards
A
C

338
Q
Which lobe of the liver receives blood from both the left and right portal veins?

a. Medial left lobe
b. Anterior right lobe
c. Posterior right lobe
d. Caudate lobe

Study These Flashcards
A
D

339
Q
A top normal thyroid isthmus measurement would be:

a. 6mm
b. 6cm
c. 4mm
d. 4cm

Study These Flashcards
A
A

340
Q
Malignant ascites is described as:

a. Anechoic fluid with free-floating bowel loops and septations
b. Anechoic fluid with free-floating bowel loops
c. Hyperechoic fluid with septations
d. Complex fluid collection with matted bowel loops

Study These Flashcards
A
D

341
Q
Which of the following structures is not included in the spermatic cord?

a. Centripetal artery
b. Cremasteric artery
c. Gonadal artery
d. Vas deferens

Study These Flashcards
A
A

342
Q
The left renal vein courses laterally between:

a. The aorta and the celiac
b. The aorta and the SMA
c. The IVC and the portal veins
d. The IVC and the hepatic veins

Study These Flashcards
A
B

343
Q
A pt presents for a post-transplant evaluation of the pancreas. Where is the most common location for an allograft?

a. Right iliac fossa
b. Periumbilical area
c. Epigastric area
d. LLQ

Study These Flashcards
A
A

344
Q
The pancreatic duct should normally be < __ in diameter in children and young adults.

a. 0.5mm
b. 1mm
c. 2mm
d. 4mm

Study These Flashcards
A
C

345
Q
Where is the dartos muscle layer?

a. In the abdominal wall
b. In the cystic duct
c. In the urinary bladder
d. In the scrotal sac

Study These Flashcards
A
D

346
Q
_ transplant rejection appears as swollen kidney, loss of corticomedullary definition, enlarged renal pyramids, compression of the renal sinus and decreased blood flow with an increased RI value.

a. Chronic
b. Acute
c. Chronic or acute, stages of rejection are not able to be differentiated with US
d. Rejection of a renal allograft is not usually evaluated by US

Study These Flashcards
A
B

347
Q
The spleen’s normal echogenicity is:

a. Homogeneous with portal echoes seen throughout
b. Homogeneous and hyperechoic compared the renal sinus
c. Homogeneous throughout
d. Mildly inhomogeneous

Study These Flashcards
A
C

348
Q
What is the name of the calcium based hormone produced by the parathyroid?

a. PTH
b. TSH
c. Calcitonin
d. None of the above

Study These Flashcards
A
D
Calcitonin is produced by the thyroid and the parathyroid produces parathyroid hormone. Hyperparathyroidism usually leads to increased PTH production which decreases the calcitonin production. This decrease in calcitonin and related calcium metabolism increases the calcium resorption in the intestines and decreases renal filtration of calcium from the blood.

349
Q
The renal arteries arise from the lateral walls of the aorta _.

a. 1-2cm above the SMA
b. 2-3cm below the SMA
c. 2-3cm below the IMA
d. At the level of the SMA

Study These Flashcards
A
B

350
Q
The bare area of the liver is in contact with:

a. Diaphragm
b. Right kidney
c. IVC
d. Right portal vein

Study These Flashcards
A
A

351
Q
On a TRV sonogram, the CBD enters the _ aspect of the head of the pancreas and lays ___ to the IVC.

a. Posterior, posterior
b. Anterior, medial
c. Posterior, anterior
d. None of the above

Study These Flashcards
A
C

352
Q
Which of the following statements regarding the dome of the liver is false?

a. The hepatic veins increase in size as they near the dome
b. Intercostal scanning is required to evaluate this portion of the liver
c. Deep inspiration is required to evaluate this portion of the liver
d. The dome of the liver is in contact with the diaphragm

Study These Flashcards
A
B

353
Q
The renal collecting system is dilated and contains mildly echogenic material with dirty shadowing. This most likely represents:

a. Staghorn calculus
b. Pyelonephritis
c. Hydronephrosis
d. Pyonephrosis

Study These Flashcards
A
D
The debris and dirty shadowing correlate with an infection causing pus accumulation (pyonephrosis). The gas produced by the bacteria in the infection causes dirty shadowing. A staghorn calculus would be highly reflective and exhibit clean shadowing. Hydronephrosis is defined as a backup of urine into the renal pelvis and calyces which would appear anechoic.

354
Q
The _ zone is the prostate zone most commonly affected by malignancy.

a. Peripheral
b. Transitional
c. Central
d. Paraprostatic

Study These Flashcards
A
A

355
Q
Which of the following is the least likely complication associated with chronic hydronephrosis?

a. Pulmonary embolism
b. Sepsis
c. Renal failure
d. Systemic HTN

Study These Flashcards
A
A
Pulmonary embolism is a vascular abnormality that occurs with thrombus formation in the IVC or its tributaries. Chronic renal hydronephrosis can result in renal failure. The kidney function will decrease and the BP will rise to try to increase the filtration rate. A chronic obstruction usually leads to infection and can develop into sepsis.

356
Q
Which of the following holds the psoas muscle?

a. Perirenal space
b. Posterior perirenal space
c. Anterior perirenal space
d. Retrofascial space
e. None of the above

Study These Flashcards
A
D

357
Q
A biloma is:

a. Associated with malignancy of the biliary tree
b. A possible complication of renal surgery
c. A collection of bile in the peritoneal cavity
d. A congenital malformation of the biliary tree

Study These Flashcards
A
C

358
Q
A 1yr old presents for a testicular US due to a palpable lump. The lab work in the chart indicates high levels of AFP. A mostly solid intratesticular mass is identified in the right testicle. There is a small area of possible necrosis centrally. Which of the following most likely describes these findings?

a. Epidermoid cyst
b. Yolk sac tumor
c. Lymphoma
d. Seminoma

Study These Flashcards
A
B
Yolk sac tumor is the most common testicular tumor in pts <2yrs old. They produce AFP (alpha fetoprotein). Sonographically they usually have areas of hemorrhage or necrosis in the mass causing a complex appearance.

359
Q
What vessels drain the blood from the caudate lobe?

a. Umbilical vein
b. Emissary vein
c. Left portal vein
d. Coronary vein

Study These Flashcards
A
B

360
Q
The term “splanchnic arteries” describes which of the following?

a. Celiac, SMA and IMA
b. Hepatic, renal, splenic, and celiac
c. Hepatic, splenic, and gastric
d. All the arteries of the abdominal aorta from the diaphragm to the umbilicus

Study These Flashcards
A
A

361
Q
A pt presents for an US of the RUQ due to a recent diagnosis of lung cancer. If liver metastasis is present, what are the expected US findings?

a. Multiple target lesions
b. Multiple hyperechoic, hypervascular lesions
c. Multiple complex cystic lesions
d. Large solitary hypoechoic lesion

Study These Flashcards
A
A
Hyperechoic lesion- metastasis from GI tract
Hyperechoic lesions with hypervascularity-metastasis from RCC
Hypoechoic lesions- metastasis from lymphoma
Cystic lesions-metastasis from sarcoma
Bull’s eye (target) lesions- metastasis from lungs

362
Q
How does a junctional parenchymal defect appear on an US exam?

a. Triangular, hyperechoic area on the anterior aspect of the upper pole of the right kidney
b. Thicker cortex on the lateral aspect of the left kidney
c. Narrowing or obstruction of the junction of the renal pelvis and ureter
d. Prominent renal cortical parenchyma located between 2 medullary pyramids

Study These Flashcards
A
A

363
Q
The pancreatic tail is anterior to all of the following, except:

a. Upper pole of the left kidney
b. Lesser omental sac
c. Splenic hilum
d. Left adrenal gland

Study These Flashcards
A
B

364
Q
All of the following are located posterior to the kidney, except:

a. Diaphragm
b. Levator ani
c. Psoas muscle
d. Quadrus lumborum

Study These Flashcards
A
B

The levator ani is one of the muscles of the pelvic floor.

365
Q
Which of the following statements is correct regarding testicular seminoma?

a. Spreads to liver first with metastasis
b. Most common in African-American males
c. Associated with cryptorchidism, trisomy 21, klinefelter syndrome and smoking
d. Unresponsive to radiation therapy

Study These Flashcards
A
C
Seminomatous tumors are the most common “pure” germ cell tumor of the testes. Most common in white males age 40-50yrs. It has been associated with cryptorchidism, trisomy 21 (down syndrome), klinefelter syndrome, and smoking. When it metastasizes, it spreads to the retroperitoneal lymph nodes first. They have the best prognosis of all germ cell tumors because they are very responsive to radiation and chemotherapy.

366
Q
GB polyps measuring > _ in diameter are highly suspicious for malignancy.

a. 5mm
b. 10mm
c. 15mm
d. 20mm

Study These Flashcards
A
B

367
Q
_ can lead to air within the portal venous system and _ can lead to air in the biliary tree.

a. An ERCP, ulcerative colitis
b. Ulcerative colitis, appendicitis
c. Ulcerative colitis, an ERCP
d. Diverticulitis, ulcerative colitis

Study These Flashcards
A
C
Ulcerative colitis allows bacteria to invade the blood stream through the portal venous system. This bacteria enters the liver where it produces gas that is identified within the portal system. An ERCP (endoscopic cholangiopancreatogram) is used to evaluate the head of the pancreas and ampulla of Vater. The procedure may allow a small amount of air to enter the ductal system through the ampulla of Vater.

368
Q
A normal portal vein will:

a. Decrease in diameter after eating
b. Increase in diameter by more than 20% with deep inspiration
c. Increase in diameter after exercise
d. Increase in diameter by placing pt in the upright position

Study These Flashcards
A
B

369
Q
Which of the following correctly describes the location of the right hepatic vein?

a. Courses inferiorly through falciform ligament
b. Courses superiorly though the main lobar fissure
c. Courses superiorly through the right intersegmental fissure
d. Courses inferiorly through the right intersegmental fissure

Study These Flashcards
A
C

370
Q
The most common cause of acute scrotal pain in postpubertal men is:

a. Acute epididymitis
b. Hydrocele
c. Torsion
d. Orchitis

Study These Flashcards
A
A

371
Q
A pt presents with abdominal pain for 3 months following a missionary trip to India. Lab values demonstrate normal LFT’s. The US exam demonstrates a 3cm cyst with septations in the right lobe of the liver. Which of the following best describes the findings on the exam?

a. Schistosomiasis
b. Candidiasis
c. Hydatid disease
d. Histoplasmosis infection

Study These Flashcards
A
C
Hydatid infection is common in countries with lots of cows and sheep. The parasite infests the liver parenchyma and a cyst with numerous septations forms.

372
Q
Which of the following is least likely to be associated with an RI over 0.8 in the common hepatic artery?

a. Budd Chiari syndrome
b. Cirrhosis
c. Focal nodular hyperplasia
d. Transplant rejection

Study These Flashcards
A
C
Flow in the hepatic artery is normally low resistance with an RI of 0.55-0.8. Flow resistance increases to RI >0.8 and portal HTN, hepatic congestion, transplant rejection or chronic HCC is suspected.

373
Q
Which of the following hormones is responsible for causing the GB to contract?

a. Trypsin
b. Insulin
c. Lipase
d. Cholecystokinin

Study These Flashcards
A
D

374
Q
Which of the following intrahepatic structures aids in locating the neck of the GB?

a. Main lobar fissure
b. Falciform ligament
c. Ligamentum teres
d. Ligamentum venosum

Study These Flashcards
A
A

375
Q
A pt presents for an abdominal US due to nausea and vomiting. While scanning the GB, the pt asks you to take a break due to the pain caused by the transducer pressure. How should you report this to the radiologist?

a. Positive Homan’s sign
b. Positive Murphy’s sign
c. Pt uncooperative, unable to efficiently perform the exam
d. Positive Morrison’s sign

Study These Flashcards
A
B

376
Q
What renal abnormality is suspected when sloughed papilla are identified in the urine?

a. Acute pyelonephritis
b. Papillary necrosis
c. Nephrocalcinosis
d. Hydronephrosis

Study These Flashcards
A
B
Renal papillary necrosis refers to ischemic necrosis of the renal papilla. The necrosis leads to the presence of sloughed papilla in the urine. It is most commonly caused by analgesic abuse.

377
Q
Which of the following best describes the normal Doppler tracing from the renal arteries?

a. Low resistance with increased diastolic flow reversal
b. Low resistance with increased diastolic flow
c. Triphasic with increased diastolic flow reversal
d. High resistance with increased diastolic flow reversal
e. Low resistance with respiratory phasicity

Study These Flashcards
A
B

378
Q
When scanning a large habitus pt, a possible renal cyst is identified. Which of the following is not a characteristic of a simple renal cyst?

a. Isoechoic
b. Well defined borders
c. Round or oval
d. Posterior enhancement
e. Smooth walls

Study These Flashcards
A
A

379
Q
When a AAA ruptures, which retroperitoneal space will fill with blood first?

a. Anterior pararenal
b. Omental bursa
c. Perirenal
d. Posterior perirenal

Study These Flashcards
A
C

Why not “A” since the aorta is located in the anterior pararenal space?????

380
Q
Alcohol abuse is the most common cause for:
a. Renal failure
b. Cholecystitis
c. Chronic pancreatitis
d. Acute pancreatitis
Study These Flashcards
A
C

381
Q
Which type of liver cyst is caused by a parasite?

a. Pyogenic
b. Fungal
c. Amebic
d. Viral

Study These Flashcards
A
C

382
Q
All of the following correctly describe biliary hamertomas, except:

a. Usually demonstrates a single hyperechoic mass >3cm
b. Focal developmental lesions of the liver
c. Associated with congenital hepatic fibrosis, autosomal dominant polycystic disease and cholangiocarcinoma
d. AKA von Meyenburg complexes

Study These Flashcards
A
A
Biliary hamartomas AKA Meyenburg complexes. They are focal developmental lesions of the liver that are composed of groups of dilated intrahepatic ducts within dense stroma. US appearance includes multiple well defined hypoechoic solid nodules <1cm. numerous echoegenic foci with ringdown may also be identified throughout the liver. Biliary hamartomas have been associated with congenital hepatic fibrosis, autosomal dominant polycystic disease and cholangiocarcinoma.

383
Q
The renal anomaly where the kidneys demonstrate congenital duplication of the renal pelvis/calyces is called:

a. Ectopic kidney
b. Horseshoe kidney
c. Renal agenesis
d. Duplicated collecting system

Study These Flashcards
A
D

384
Q
All of the following are associated with celiac artery occlusion, except:

a. Flow reversal in the splenic artery
b. Flow reversal in the hepatic artery
c. Increased systolic velocity of flow in the celiac artery
d. All the above are characteristics of a celiac artery obstruction

Study These Flashcards
A
C
If the vessel is occluded there will be no flow detected! Celiac artery occlusion will cause a reversal flow of its branches.

385
Q
An abdominal US is ordered for a pt with bilateral pedal edema. The abdomen demonstrates mild ascites but otherwise normal organs. The IVC appears prominent with a constant diameter of 2.8cm. A continuous venous waveform is demonstrated on Doppler evaluation. Which of the following is the most likely cause for the US exam findings?

a. Portal HTN
b. Malignant HTN
c. Congestive heart failure
d. All of the above

Study These Flashcards
A
C
CHF indicates poor cardiac function. Blood flow returns to the heart from the lower extremities via the IVC. If the heart is not ejecting enough blood during systole, the venous return will backlog in the VIC. The IVC should normally collapse with respiration and demonstrate triphasic, pulsatile waveform on PW Doppler.

386
Q
A pt is referred for an abdominal US after a CT demonstrated a possible mass in the posterior right lobe of the liver. The US demonstrates a hyperechoic mass with smooth borders and posterior enhancement. Which of the following abnormalities corresponds most with the findings on the image?

a. Hepatic adenoma
b. Cavernous hemangioma
c. Focal fatty sparing
d. HCC

Study These Flashcards
A
B

387
Q
Primary sclerosing cholangitis significantly increases the risk of developing:

a. Cholangiocarcinoma
b. Klatskin tumor
c. Hepatic lipoma formation
d. Pancreatic carcinoma

Study These Flashcards
A
A

388
Q
What changes occur on the US image when scanning breast tissue affected by acute mastitis?

a. Decreased penetration due to tissue edema
b. Posterior enhancement
c. Improved visualization of the ductal system due to tissue edema
d. Dilated ductal system within normal breast tissues

Study These Flashcards
A
A

389
Q
The most common extra-testicular neoplasm is:

a. AV malformation
b. Lipoma
c. Adenomatoid tumor
d. Hamartoma

Study These Flashcards
A
C

390
Q
A normal evaluation of a pt with transjugular intrahepatic shunt would include all of the following, except:

a. Spleen length <12cm in average pt
b. Hepatofugal flow in the main portal flow
c. Continuous flow pattern within the shunt
d. Diminished flow in the right and left portal branches

Study These Flashcards
A
B
In a properly functioning shunt, portal flow should move easily into the hepatic venous system with hepatopetal flow direction. Flow in the intrahepatic portal branches will be diminished as the main portal flow is shunted directly into the hepatic veins. Signs of portal HTN should diminish, reduced spleen size, reduced appearance of gastric and portal varicosities, resolution of ascites.

391
Q
While scanning the liver, you notice a small dilated tubular structure with stellate branches coursing anterior to the right portal vein. Which of the following best describes the finding?

a. Dilated cystic duct
b. Dilated right hepatic duct
c. Duplicated right portal vein
d. Budd Chiari malformation

Study These Flashcards
A
B

The right and left hepatic ducts follow the course of the portal veins to merge into the common hepatic duct.

392
Q
Pyloric stenosis is diagnosed when the muscle wall thickness exceeds:

a. 6mm
b. 3mm
c. 2mm
d. 5mm

Study These Flashcards
A
B

393
Q
Which of the following terms can be used to describe the normal flow in the portal vein?

a. Continuous
b. Biphasic
c. Hepatofugal
d. Pulsatile

Study These Flashcards
A
A

394
Q
Which of the following biliary conditions is associated with the formation of Rokitansky-Aschoff sinuses?

a. Choledochal cyst
b. Adenomyomatosis
c. Acute cholecystitis
d. Emphysematous cholecystitis
e. Pneumobilia

Leave a Comment

Scroll to Top